Você está na página 1de 192

Kendriya Vidyalaya Sangathan

18, Institutional Area, Shaheed Jeet Singh Marg,


New Delhi - 110 60
DELHI REGION
STUDY MATERIAL
CLASS X (ENGLISH)
2010-2011
Preface
1! An "utline o# Sylla$us and Mar%ing S&he'e
i( )ints
! S*+,I"N A - -*ADIN.
ii( ,hree Sol/ed 0assages 12a&tual, Dis&ursi/e and 3iterary(
iii( ,hree 4nsol/ed 0assages #or 0ra&ti&e 12a&tual, Dis&ursi/e
and 3iterary(
5! S*+,I"N 6 7 8-I,IN.
i( )ints a$out a to9i&, Di/ision o# Mar%s, Sol/ed :uestions and
0ra&ti&e :uestions
ii( 6iogra9hi&al S%et&h; 2a&tual Des&ri9tion 10eo9le, 0la&es,
*/ents(; Data Inter9retation;Dialogue 8riting
iii( In#or'al 3etter, 2or'al 3etter, * 'ail
i/( Arti&le, S9ee&h, Diary *ntry, Story and De$ate
<! S*+,I"N + 7 .-AMMA-
i. *diting - )ints, Sol/ed :uestions = 0ra&ti&e :uestions
ii. "'ission - )ints, Sol/ed :uestions = 0ra&ti&e :uestions
iii. +lo>e .a9-2illing - )ints, Sol/ed :uestions = 0ra&ti&e
:uestions
iv. Senten&e -e-ordering - )ints, Sol/ed :uestions = 0ra&ti&e
:uestions
v. Dialogue +o'9letion - )ints, Sol/ed :uestions = 0ra&ti&e
:uestions
vi. Senten&e ,rans#or'ation 1-e9orted S9ee&h, 0assi/e
2or's and News9a9er )eadlines( - )ints, Sol/ed
:uestions = 0ra&ti&e :uestions
?! S*+,I"N D 7 3I,*-A,4-*
i( 2i&tion 7 2our lessons with Main 0oints, *@tra&ts #or
+o'9rehension, Multi9le +hoi&e :uestions, Short
Answer :uestions and 3ong Answer :uestions
ii( 0oetry 7 2i/e 0oe's with Main 0oints, *@tra&ts #or
+o'9rehension, Multi9le +hoi&e :uestions,, Short
Answer :uestions and 3ong Answer :uestions
iii( Dra'a - ,wo 0lays with Main 0oints, *@tra&ts #or
+o'9rehension and 3ong Answer :uestions
6! Sa'9le :uestion 0a9er Su''ati/e Assess'ent -1 1Sol/ed(
Su''ati/e Assess'ent -
Foreword
8ith the introdu&tion o# +ontinuous and +o'9rehensi/e */aluation 1++*(
syste' o# s&hool-$ased e/aluation o# students, the ai's o# tea&hing-learning
ha/e $roadened to a 9oint that &o/ers all as9e&ts o# studentsA de/elo9'ent!
It is a de/elo9'ental 9ro&ess o# assess'ent whi&h e'9hasi>es on two #old
o$Be&ti/es! ,hese o$Be&ti/es are &ontinuity in e/aluation and assess'ent o#
$road $ased learning and $eha/ioural out&o'es! ,he ter' A&ontinuousC is
'eant to e'9hasise that e/aluation o# identi#ied as9e&ts o# studentsA growth
and de/elo9'ent is a &ontinuous 9ro&ess rather than an e/ent, $uilt into the
total tea&hing-learning 9ro&ess and s9read o/er the entire s9an o# a&ade'i&
session! It 'eans regularity o# assess'ent, #reDuen&y o# unit testing, diagnosis
o# learning ga9s, use o# &orre&ti/e 'easures, retesting and #eed$a&% o# e/iden&e
to tea&hers and students #or their sel# e/aluation!
,he se&ond ter' A&o'9rehensi/eC 'eans that the s&he'e atte'9ts to
&o/er $oth the s&holasti& and the &o-s&holasti& as9e&ts o# studentsC growth
and de/elo9'ent! Sin&e a$ilities, attitudes and a9titudes &an 'ani#est
the'sel/es in #or's other then the written word, the ter' re#ers to a99li&ation o#
/ariety o# tools and te&hniDues 1$oth testing and non-testing( and ai's at
assessing a learnerCs de/elo9'ent in di##erent areas o# learning!
,he ++* assess'ent in&ludes $oth s&holasti& and &o-s&holasti& assess'ent!
,he desira$le $eha/iour related to the learnerCs %nowledge, understanding,
a99li&ation, analysis, e/aluation, synthesis, and the a$ility to a99ly it in an
un#a'iliar situation are so'e o# the o$Be&ti/es in s&holasti& do'ain! ,he
desira$le $eha/iour related to learnerCs 3i#e S%ills, attitudes, interests, /alues, &o-
&urri&ular a&ti/ities and 9hysi&al health are des&ri$ed as s%ills to $e a&Duired in
&o-s&holasti& do'ain!
6oth S&holasti& and &o-s&holasti& do'ains should $e assessed in two ways-
#or'ati/e assess'ent and su''ati/e assess'ent! 2or'ati/e assess'ent is
&arried out during a &ourse o# instru&tion #or 9ro/iding &ontinuous #eed$a&% to
$oth the tea&hers and the learners #or ta%ing de&isions regarding
a99ro9riate 'odi#i&ations in the transa&tional 9ro&edures and learning
a&ti/ities! ,he tools #or #or'ati/e assess'ent in s&holasti& do'ain are oral
Duestions, assign'ents, &on/ersation s%ills, 9roBe&ts, Dui>>es, grou9 wor% et&!

Su''ati/e assess'ent is &arried out at the end o# a &ourse o# learning! It
'easures or Csu's-u9C how 'u&h a student has learned #ro' the &ourse! ,he
tools #or Su''ati/e assess'ent in s&holasti& do'ain are 'ulti9le &hoi&e
Duestions, short answer ty9e and long answer ty9e des&ri9ti/e Duestions at ter'
end!
,he study 'aterial is a s'all atte'9t to e@9ose students to a /ariety o# test ty9es
and 9ra&ti&e 'aterial #or all the #our se&tions /i> -eading, 8riting, .ra''ar and
3iterature! So'e Duestions and e@er&ises are sol/ed while others are #or
students to gra99le with a#ter they &o'9lete their understanding o# the di##erent
units! ,ea&hers will need to gi/e students &onsistent 9ra&ti&e in handling
M+:s 7a &o'9arati/ely new way o# testing! Sin&e three answers are distra&ters,
students are li%ely to get &on#used! It is i'9ortant #or the' to underline the
i'9ortan&e o# &on&e9t &onsolidation rather than rote 'e'ory! ,he gra''ar and
literature se&tion also 9ro/ide a'9le o99ortunity #or 9ra&ti&ing new Duestions!
+are has $een ta%en to &lari#y &on&e9ts in di##i&ult units o# 3iterature -eader $y
9ro/iding a su''ary and an e@9lanation o# di##i&ult &on&e9ts!
I &ongratulate Dr! S!0! ,ha%ur, 0rin&i9al, Kendriya Vidyalaya Se&tor 8, -! K!
0ura', New Delhi and his tea' 'e'$ers na'ely Mrs! Vi''y Singh, 0.,
1*nglish(, K! V! Se&tor 8, -! K! 0ura', New DelhiE Mrs! Ka/ita -a%hara, 0.,
1*nglish(, K! V! MasBid MothE Mrs! 8il'a D! Ku'ar, 0., 1*nglish(, K! V! Delhi
+antt! E Mrs! Madhu$ala Singh, 0., 1*nglish(, K! V! -aBo%ariE Mrs! M! .iriBa,
,., 1*nglish(, MasBid MothE Mrs! Niru9a'a, ,., 1*nglish(, K! V! Delhi +antt!
No!E Mrs! Ira Joshi, ,., 1*nglish(, K! N! NoidaE Mrs! Veena +hawala, ,.,
1*nglish(, K! V! AndrewsgunBE Mrs! 0urni'a Ka9oor, ,., 1*nglish(, Delhi +antt!
No!1 and Mrs! Durgesh Shar'a, ,., 1*nglish(, K, V, Jana%9uri #or de/elo9ing
the Study Material o# +lass F 1*nglish( in a /ery short s9an o# ti'e!
It is ho9ed that $oth tea&hers and students will #ind the 'aterial use#ul!
V! K! Sri/asta/a
Asstt! +o''issioner
Delhi -egion
Cla X (ENGLISH COMMUNICATI!E)
+ode No! 101
SECTION A: READING 20 Marks
:s 1-< 2our unseen reading 9assages o# ? 'ar%s ea&h! *a&h reading 9assage will ha/e
? su$- 9arts, ea&h o# 1 'ar%! All Duestions will $e 'ulti9le &hoi&e Duestions! ,he
9assages will $e e@tra&ts #ro' 9oe's;#a&tual;des&ri9ti/e;literary;dis&ursi/e 9assages!
:uestions will test in#eren&e, e/aluation and /o&a$ulary! ,here will $e at least 0< 'ar%s
#or assessing /o&a$ulary s%ills! ,he total length o# the < 9assages will $e $etween 6?0
and 800 words!
SECTION B: WRITING 20 Marks
,he writing se&tion &o'9rises o# three writing tas%s as indi&ated $elowG
: ? A short answer Duestion o# u9to 100 words in the #or' o# a 6iogra9hi&al S%et&h
1e@9ansion o# notes on an indi/idualCs li#e or a&hie/e'ents into a short 9aragra9h(;Data
Inter9retation ; Dialogue 8riting or Des&ri9tion 19eo9le, 9la&es, e/ents(!
,he Duestion will assess studentsC s%ill o# e@9ressing ideas in &lear and gra''ati&ally
&orre&t *nglish, 9resenting ideas &oherently and &on&isely, writing a &lear des&ri9tion, a
&lear a&&ount o# e/ents, e@9anding notes into a 9ie&e o# writing, or trans&oding
in#or'ation #ro' one #or' to another!
" Mar#
: 6 A long answer Duestion 1'ini'u' 1?0 words( in the #or' o# a #or'al letter;in#or'al
letter or an e'ail! ,he out9ut would $e a long 9ie&e o# writing and will assess the use o#
a99ro9riate style, language, &ontent and e@9ression!
$ Mar#
: H A long answer Duestion 1'ini'u' 180 words( in the #or' o# a %&ar' e()r', ar)&cle,
*eec+, )-r' -r %e.a)e! StudentsC s%ills in e@9ression o# ideas in &lear and
gra''ati&ally &orre&t *nglish, 9lanning, organising
and 9resenting ideas &oherently $y introdu&ing, de/elo9ing and &on&luding a to9i&,
&o'9aring and &ontrasting ideas and arri/ing at a &on&lusion, 9resenting an argu'ent
with su99orting e@a'9les, using an a99ro9riate style and #or'at and e@9anding notes
into longer 9ie&es o# writing and &reati/e e@9ression o# ideas will $e
assessed! $ Mar#
I'9ortant Note on 2or'at and 8ord li'itG
2or'at will not &arry any se9arate 'ar%s and in 'ost &ases, #or'at will $e gi/en
in the Duestion 9a9er!
,he word li'it gi/en is the suggested 'ini'u' word li'it! No &andidate 'ay $e
9enalised #or writing 'ore or less than the suggested word li'it! Stress should $e
on &ontent, e@9ression, &oheren&e and rele/an&e o# the &ontent 9resented!
SECTION C: GRAMMAR 20 Marks
,his se&tion will assess .ra''ar ite's in &onte@t #or 0 Mar%s! It will &arry ? Duestions
o# < 'ar%s ea&h!
,ests ite's will $e Multi9le +hoi&e :uestions and test /arious gra''ati&al ite's in
&onte@t!
: 8!-: 1 will test gra''ar ite's whi&h ha/e $een dealt with in &lass F! Di##erent
stru&tures su&h as /er$ #or's, senten&e stru&ture, &onne&tors, deter'iners, 9ronouns,
9re9ositions, &lauses, 9hrases, et&!, &an $e tested through #or'ati/e assess'ent o/er a
9eriod o# ti'e! As #ar as the su''ati/e assess'ent is &on&erned, it will re&y&le
gra''ar ite's learnt o/er a 9eriod o# ti'e and will test the' in &onte@t through Multi9le
+hoi&e :uestion #or'at!
,ests ty9es used will in&lude ga9-#illing, &lo>e 1ga9 #illing e@er&ise with $lan%s at regular
inter/als(, senten&e &o'9letion, reordering word grou9s into senten&es, editing,
dialogue-&o'9letion and senten&e-trans#or'ation!
,he gra''ar sylla$us will $e sa'9led ea&h year, with 'ar%s allotted #orG Ver$ #or's
Senten&e stru&tures other areas
NoteG Ju'$led words in reordering e@er&ise to test synta@ will in/ol/e senten&es in a
&onte@t! *a&h senten&e will $e s9lit into sense grou9s 1not ne&essarily into single words(
and Bu'$led u9!
SECTION D: LITERATRE 20 Marks
: 15 ,wo e@tra&ts out o# three #ro' 9rose, 9oetry or 9lays in the #or' o# Multi9le +hoi&e
:uestions $ased on re#eren&e to &onte@t! *a&h e@tra&t will &arry 5 'ar%s! 1/-r% l&0&)
20-10 2-r%( 13145Mar#
: 1< 2our out o# #i/e short answer ty9e Duestions $ased on 9rose, 9oetry and 9lay o#
'ar%s ea&h! ,he Duestions will not test re&all $ut in#eren&e and e/aluation! 18ord li'itG
50-<0 words ea&h( $ Mar#
: 1? "ne out o# two long answer ty9e Duestions to assess 9ersonal res9onse to te@t $y
going $eyond the te@t;9oe';story or e@tra&t! +reati/ity, i'agination and e@tra9olation
$eyond the te@t and a&ross two te@ts will also $e
assessed! 5 0ar#
0res&ri$ed 6oo%s;Materials
1! Intera&t in *nglish 7 F Main +ourse 6oo% -e/ised edition
! Intera&t in *nglish 7 F 3iterature -eader -e/ised edition
0u$lished $y +6S*
5! Intera&t in *nglish 7 F 8or%$oo% -e/ised edition Delhi-1100I
Rea%&(6 Sec)&-(7
-eading #or &o'9rehension, &riti&al e/aluation, in#eren&e and analysis is a s%ill to $e
tested #or'ati/ely as well as su''ati/ely! ,here will $e no di/ision o# 9assages #or this
se&tion, howe/er #or reading 9ur9oses, the Intera&t in *nglish Main +ourse 6oo% will $e
read in two ter's i!e! ,er' I 1A9ril -Se9te'$er( and ,er' II 1"&to$er 7 Mar&h(!
/r&)&(6 Sec)&-(7
All ty9es o# short and e@tended writing tas%s will $e dealt with in $oth I and II ,er'
Su''ati/e as well as in 2or'ati/e Assess'ent! 2or 9ur9oses o# assess'ent all the'es
dealt with in Main +ourse 6oo% and other the'es 'ay $e used!
Gra00ar7
.ra''ar ite's 'entioned in the sylla$us will $e taught and assessed su''ati/ely as
well as #or'ati/ely o/er a 9eriod o# ti'e! ,here will $e no di/ision o# sylla$us #or
.ra''ar in the su''ati/e or #or'ati/e assess'ents #or the two ter's!
S'lla.8 f-r Ter0
S.No. Te!" Books Firs" Ter# $A%ri& '
Se%"e#(er)
Se*o+d Ter# $O*"o(er '
Mar*,)
FA - -0 FA2 -0 SA I 20 FA. -0 FA/ -0 SA II /0
Li"era"0re Reader
1ROSE
- T,e Tri(0"e
2 C0"ie 1ie
. T,e Le""er
/ T,e &"i#a"e Sa2ari
1OETR3
- Ni4," o2
",e S*or%io+

Te!" Books
2 Ode "o ",e Wes" Wi+d
. T,e Fro4 a+d T,e
Ni4,"i+4a&e

/ Mirror
5 T,e Ri#e o2 ",e
A+*ie+" Mari+er

DRAMA
- A C,ris"#as Caro&
2 60&i0s Caesar
Mai+ Co0rse Book FA - FA 2 SA I FA . FA / SA II
- 7ea&", a+d Medi*i+e
2 Ed0*a"io+
. S*ie+*e
/ E+viro+#e+"
5 Trave& a+d To0ris#
1! 2or'ati/e Assess'ent is assess'ent J#orC learning! ,hus s&hools 'ay ada9t the
a$o/e $rea%u9 as 9er their &on/enien&e!
2. All a&ti/ities related to 2or'ati/e Assess'ent su&h as 3anguage ga'es,
Dui>>es, 9roBe&ts, role 9lays, dra'atisation , s&ri9t writing et& 'ust $e done as Jin
&lassC and Jin s&hoolC a&ti/ities! In &ase, a #ield sur/ey or /isit is ta%en u9 it 'ust
$e under the dire&t su9er/ision o# the tea&her!
SECTION A: READING
8ESTIONS -9/
Reading Section is designed to test the comprehension. Questions are framed to test
inference, evaluation and vocabulary. All questions will be multiple choice questions. The
passages will be extracts from factualdiscursiveliterary passages. The total length of the
four passages will be between !"#$%## words.
TI1S ON COM1RE7ENDING NSEEN 1ASSAGES
&. S'imming( Read the passage carefully to 'now what the passage is about.
). Scanning( Read the passage again and identify the main points.
*. Read the question one by one and loo' for the correct option.
+. ,rite the correct option number and the option in the answer sheet.
&. -n the basis of reading of the above poem, answer the questions that follow by
choosing the correct option.
I 98&l% /all
I .8&l% 2all7
/all )+a) *r-)ec),
/all )+a) +&el%,
/all )+a) a' I +all (-) '&el%
Or re:eal
/+- I a0 -r +-2 I feel;
I .8&l% 2all7
/all )+a) +&%e,
/all )+a) c-:er 2+a)< &(&%e,
/all )+a) )are -r 0&le -r l--# a2a',
S&le() l&e,
/all )+a) e:e( .l-c# 0' e'e
=r-0 )+e )ear I 0&6+) +a:e cr&e%;
I .8&l% 2all7
/all )+a) (e:er le) 0e
Tr8l' )-8c+
T+-e I l-:e - :er' 08c+;
/all )+a) (ee% )- fall>
/all 0ea() )- .e f-r)ree
a. The poet builds walls because they
i. /rotect him from inclement weather
ii( 0on1t let any intruder come in
iii( 0efine the boundary line of his house
i/( 2elp him in hiding his identity and feelings
b. The walls that the poet ma'es are made of
i( 3ric's and cement
ii( 4reen hedge
iii( ,ire
i/( 3arriers of his own emotions
c. The line that shows that the poet doesn1t always thin' that walls are good is
i( ,alls that say 56 shall not yield7
ii( ,alls that stare or smile
iii( ,alls that need to fall8
i/( ,alls that cover what is inside
d. The line 5,alls that never let me truly touch . those that 6 love so very much7
means these walls don1t let him
i( 9eet his near and dear ones
ii( :xpress his feelings to his loved ones
iii( 3e in touch with his close relatives
i/( Allow his loved ones to come to him
e. The poetic device used for walls in this poem is
i( Simile
ii( 9etaphor
iii( /ersonification
i/( Alliteration
ANSWERS
A. ;d. 2elp him in hiding his identity and feelings
3. ;d. 3arriers of his own emotions
<. ;c. ,alls that need to fall
0. ;b. :xpress his feelings to his loved ones
:. ;b. 9etaphor
1assa4e 2 $Fa*"0a&9So&ved)
Air(0s Crisis Over
Airbus says it has turned the corner after a crisis connected to production problems and
turmoil in the boardroom at its A*%# super$=umbo pro=ect that has gone on for the past
year. Spea'ing at the /aris air show, >ouis 4allois, <:- of the :uropean plane ma'er,
said, ?Airbus is bac'.?
Airbus, which announced a raft of orders on the first day of the show, is competing with
3oeing, its American rival, for the title of the largest plane ma'er in the world.
3oeing is expected to reveal the numbers of orders for its @%@ 0ream liner soon. Airbus
orders unveiled on 9onday included Qatar Airways confirming a A&!bn order for %#
A*"# Airbus planes and ordering three A*%# super$=umbos for about A@"#m.
3oeing and Airbus are also competing for orders from aircraft leasing firms. -rders from
these companies $ who ran' highly among the biggest global buyers of aircraft $ are often
regarded as an indication of how successful a model will be in the long term. Airbus also
secured orders from BS Airways that are worth A&#bn for )) of its A*"# =ets, !# A*)#s
and ten of its A**#$)## wide$body planes.
A few months ago, Airbus unveiled a ma=or cost$cutting programme aiming to reduce the
wor'force in :urope by &#,###, as well as announcing a group restructuring. ?6 can tell
you with full confidence that Airbus is bac' and fully bac', as you have started noting
yesterday as demonstrated by our first day announcements,? said 9r 4allois on the
second day of the air show. 2owever, 3oeing also announced a deal with 4eneral :lectric
;4:. on the showCs first day. 4:Cs commercial aviation services placed an order for six
@@@ 3oeing freighters valued at around A&.+bn, to be delivered in the last quarter of )##%.
A ,all Street Dournal website report, quoting the 0elta operating chief yesterday said that
0elta Air >ines were on the verge of ordering as many as &)" 3oeing @%@ =etliners by the
end of this year. 2owever, a spo'esman for 0elta later said that it had been having
conversations ?with several aircraft ma'ers? and that ?no final decision? had been made
on future fleet purchases.

a. The problems at Airbus
i. have been resolved completely.
ii. are well on their way to being sorted out.
iii. are far from resolved.
iv. will never be resolved
b. Airbus announced
i. a large number of orders on the first day of the show.
ii. some orders on the first day of the show.
iii. a few orders on the first day of the show.
iv. that 3oeing is the largest airplane ma'er of the world
c. Qatar Airways ordered
i. %* planes on 9onday.
ii. %# planes on 9onday.
iii. * planes on 9onday.
iv. &## planes on 9onday
d. BS Airways
i. placed an order for the new super$=umbo.
ii. didnCt place an order for the new super$=umbo.
iii. may have placed an order for the new super$=umbo.
iv. placed orders worth A& bn
e. 3oeing
i. announced sales of the 0reamliner.
ii. may sell some 0reamliners to 4eneral :lectric.
iii. may sell some 0reamliners to 0elta Air >ines.
iv. may sell 3oeing @@@
ANSWERS
a( ;a. have been resolved completely
$( ;a. a large number of orders on the first day of the show
&( ;a. %* planes on 9onday
d( ;b. didn1t place an order for the new super$=umbo
e( ;d. may sell 3oeing @@@
1assa4e .$Dis*0rsive9So&ved)
Rea&i": Te&evisio+
Reality television is a genre of television programming which, it is claimed, presents
unscripted dramatic or humorous situations, documents actual events, and features
ordinary people rather than professional actors. 6t could be described as a form of
artificial or ?heightened? documentary. Although the genre has existed in some form or
another since the early years of television, the current explosion of popularity dates from
around )###.
Reality television covers a wide range of television programming formats, from game or
quiE shows which resemble the frantic, often demeaning programmes produced in Dapan
in the &F%#s and &FF#s ;a modern example is 4a'i no tsu'ai., to surveillance$ or
voyeurism$ focused productions such as 3ig 3rother.
<ritics say that the term ?reality television? is somewhat of a misnomer and that such
shows frequently portray a modified and highly influenced form of reality, with
participants put in exotic locations or abnormal situations, sometimes coached to act in
certain ways by off$screen handlers, and with events on screen manipulated through
editing and other post$production techniques.
/art of reality televisionCs appeal is due to its ability to place ordinary people in
extraordinary situations. Gor example, on the A3< show, The 3achelor, an eligible male
dates a doEen women simultaneously, travelling on extraordinary dates to scenic locales.
Reality television also has the potential to turn its participants into national celebrities,
outwardly in talent and performance programs such as /op 6dol, though frequently
Survivor and 3ig 3rother participants also reach some degree of celebrity.
Some commentators have said that the name ?reality television? is an inaccurate
description for several styles of program included in the genre. 6n competition$based
programs such as 3ig 3rother and Survivor, and other special$living$environment shows
li'e The Real ,orld, the producers design the format of the show and control the day$to$
day activities and the environment, creating a completely fabricated world in which the
competition plays out. /roducers specifically select the participants, and use carefully
designed scenarios, challenges, events, and settings to encourage particular behaviours
and conflicts. 9ar' 3urnett, creator of Survivor and other reality shows, has agreed with
this assessment, and avoids the word ?reality? to describe his showsH he has said, ?6 tell
good stories. 6t really is not reality TI. 6t really is unscripted drama.?
a. 6n the first line, the writer says Cit is claimedC because
i. they agree with the statement.
ii. everyone agrees with the statement.
iii. no one agrees with the statement.
iv. they want to distance themselves from the statement.
b. Reality television has
i. always been this popular.
ii. has been popular since well before )###.
iii. has only been popular since )###.
iv. has been popular since approximately )###.
c. Dapan
i. is the only place to produce demeaning TI shows.
ii. has produced demeaning TI shows copied elsewhere.
iii. produced 3ig 3rother.
iv. invented surveillance focused productions.
d. /eople have criticised reality television because
i. it is demeaning.
ii. it uses exotic locations.
iii. the name is inaccurate.
iv.it shows reality.
e. Reality TI appeals to some because
i. it shows eligible males dating women.
ii. it uses exotic locations.
iii. it shows average people in exceptional circumstances.
iv. it can turn ordinary people into celebrities.
ANSWERS
a( ;b. every one agrees with the statement
$( ;d. has been popular since around )###
&( ;b. has produced demeaning TI shows copied from elsewhere
d( J the name is inaccurate
e( ;d. it can turn ordinary people into celebrities
1assa4e / $+so&ved)
6a%a+;s Mos" Fa#o0s Do4
6n front of the enormous Shibuya train station in To'yo, there is a life$siEe bronEe statue
of a dog. :ven though the statue is very small when compared to the huge neon signs
flashing, it isnCt difficult to find. 6t has been used as a meeting point since &F*+ and today
you will find hundreds of people waiting there for their friends to arrive.
2achi'o, an A'ita dog,was born in &F)* and brought to To'yo in &F)+. 2is owner,
/rofessor :isaburo Byeno and he were inseparable friends right from the start. :ach day
2achi'o would accompany his owner, a professor at the 6mperial Bniversity, to Shibuya
train station when he left for wor'. ,hen he came bac', the professor would always find
the dog patiently waiting for him. Sadly, the professor died suddenly at wor' in &F)"
before he could return home.
Although 2achi'o was still a young dog, the bond between him and his owner was very
strong and he continued to wait at the station every day. Sometimes, he would stay there
for days at a time, though some believe that he 'ept returning because of the food he was
given by street vendors. 2e became a familiar sight to commuters over time. 6n &F*+, a
statue of him was put outside the station. 6n &F*", 2achi'o died at the place he last saw
his friend alive.
A $ The statue of 2achi'o is
a. enormous
b. tiny
c. life$siEe
d. big
B$ The statue is difficult to find because
a. there are so many people there
b. there are huge neon signs
c. there are a lot of dogs
d. there are too many trains
C K The owner of 2achi'o was
a. station master
b. nursery teacher
c. university teacher
d. high school teacher
D$ The professor died in
a. &F)*
b. &F)+
c. &F)"
d. &F*+
E $ The dog waited every day at the station
a. for food
b. for his master to return
c. for people to pat him
d. for commuters to loo' at him
1assa4e 5 $+so&ved)
B0&&: 2or :o0
The ma'ers of a controversial computer game about bullying have decided to go ahead
and launch it despite calls for it to be banned. 6n the game, players ta'e on the role of new
students at a school and have to fight the bullies, by punching them or hitting them with a
baseball bat.
<ritics have said that the game encourages violence, but the ma'ers deny this and say
that, while there is violence in the game, it is =ust an amusing loo' at school life, besides
which, the violence in the game is directed against the bullies to protect pupils who are
being bullied. The ma'ers also say that players will learn to stand up to bullies.
A 3ritish politician, a former minister, has called for it to be banned as it might affect the
way young people perceive violence.
Anti$bullying charities have said that the game might ma'e people respond violently to
bullies, which might ma'e things more complicated and result in in=uries.
A 9 6n the game, the player ta'es on the role
a. of a bully
b. of a victim
c. of a bas'etball player
d. of a minister
B 9 The game is set in
a. a university
b. a college
c. a school
d. a computer institute
C 9 :veryone agrees that the game encourages
a. good will
b. peace
c. 'indness
d. violence
D 9 The politician used to be
a. a /rime 9inister
b. a minister
c. a statesman
d. a football player
E 9 The anti$bullying charity thin's the game is good
a. because it might ma'e pupils stand up to bullies
b. because it might not ma'e pupils stand up to bullies
c. because it might ma'e people submissive
d. because it might change the bullies
1assa4e <$+so&ved) $1oe#)
C,ari":
6 can hear them
And they1re no longer there,
<rying for a hand
That 6 couldn1t spare.
6 can see them,
Though 6 will not loo',
Reaching for the time
That 6 never too'.
6 can feel them
/ulling at my sleeve,
As'ing me to stay,
Lnowing that 61d leave.
And there1s no way
That 6 could help them all.
3ut can 6 even say,
,hen all is said and done,
That when 6 had a way
6 stopped to help =ust oneM
A. 5<harity7 means
a. an act of humility
b. an act of generosity
c. an act of stupidity
d. an act of meanness
3. 5<rying for hand7 means
a. as'ing for a hand
b. as'ing for help
c. as'ing for a favour
d. as'ing for money
<. ,ho is 5them7 referred to in the poemM
a. the needy
b. the rich
c. the givers
d. the general public
d. ,hat has the narrator doneM
a. 2e gave his hand
b. 2e gave time
c. 2e didn1t wait for them
d. 2e helped one person
:. 2ow does the narrator feelM
a. feels happy
b. feels sad
c. feels helpless
d. feels left out
SECTION B (WRITING)
Fo0r T:%es o2 =0es"io+s o+ wri"i+4 ski&&s90/ #arks
&. 3iographical S'etch
). Gactual 0escription
*. 0ata interpretation
+. 0ialogue completion
-. BIOGRA17ICAL S>ETC7
A biographical s'etch, sometimes referred to as a professional profile, is a brief narrative
that presents you in the best possible light to prospective employers, clients and the
general public. The biographical s'etch summariEes who you are, what matters most to
you, and how you add value to the people and organiEations with which you interact.
;Bsually the hints are given to write the 3iographical S'etches but in case if it is not
present, then the following points need to be stressed..
7i+"s:
&. 6t should be written in third person,
) The points included must be those which are the most significant or impressive.
*. 9ention any special awards or recognitions received by the person concerned.
+. 3e descriptive.
". ,ord limit $ %# words.
!. 3iographical S'etch carries 02 #arks 2or *o+"e+" a+d 02 #arks 2or e!%ressio+ i.e.
F&0e+*: a+d A**0ra*:.
Bio4ra%,i*a& For#a" i+*&0des:
Name and age
9ain /ersonality traits
Special interests
:ducation and training
Special <ontribution or research, if any
-ther honours, awards etc.
,hy do people li'e himherM
.
1ra*"i*e 80es"io+s:
-. Give+ (e&ow is a s,or" (io4ra%,: a+d 2a*"s a(o0" ",e &i2e o2 Sri+ivasa
Ra#a+0?a#@ ",e 2a#o0s I+dia+ #a",e#a"i*ia+. Wri"e a Bio4ra%,i*a& Ske"*, o2 S.
Ra#a+0?a# i+ a(o0" A0 words.
Na#e: Srinivasa Ramanu=am
A*,ieve#e+": Ra#a+0?a# a mathematician par excellence. ,idely believed to be the
greatest mathematician of the )#th <entury.
Significant contribution to the analytical theory of numbers and wor'ed on elliptic
functions,
Ed0*a"io+: too' admission in 4overnment <ollege in Lumba'onam which he entered in
&F#+. Neglected his studies in other sub=ects, dropped out of college
<ontinued to study fractions and divergent series in &F#%
Special <ontribution or research( in &F#F, he got his first paper published on 3ernoulli1s
numbers
6n a =oint paper with 2ardy, Ramanu=an gave an asymptotic formula for p ;n.. 6t had the
remar'able property that it appeared to give the correct value of p ;n.,
2. O+ ",e (asis o2 ",e va&0e %oi+"s 4ive+ (e&ow@ wri"e a (io4ra%,i*a& ske"*, i+ a(o0"
A0 words.
7i+"s:
Na#e: Sachin Ramesh Tendul'ar
1ro2essio+a& a*,ieve#e+": -ne of the greatest batsmen in the history of 6ndian <ric'et
>eading run$scorer and century ma'er in the Test and -ne 0ay <ric'et
-nly player to score a double century in the history of -06 cric'et
6n )##), ran'ed as the second greatest test batsman
And ",e se*o+d greatest one day international ;-06. batsman of all time
Ni*k+a#e( referred to as Little Master or Master Blaster.
Scored *#,### runs in international cric'et on )# November )##F,
Awards( honoured with /adma 3hushan award, 6ndiaCs second highest civilian
Award, and the Ra=iv 4andhi Lhel Ratna award, 6ndiaCs highest sporting honour.
*. -n the basis of the given hints, write a 3iographical s'etch in about %# words.
2ints(
Na#e: 9alini 3hattacharya
1ro2essio+ 2istory Teacher
E!%erie+*e: )" Oears as /4T
80a&i2i*a"io+s: 9.A. in 2istory with a degree in teaching
S%e*ia& areas o2 e!%er"ise( excellent teacher, talented in dramas, wor'ing for an N4- in
her leisure for underprivileged children
O",ers: head of department, senior most teacher,
W,: is s,e %o%0&arB( calm, composed and helpful nature
2. Fa*"0a& Des*ri%"io+

Gactual description simply means describing a person, place and an event factually. A
factual description involves developing a complete mental picture of a person, place or
event. 6t must not include opinions or speculationH it should be descriptive and detailed.
Fa*"0a& Des*ri%"io+ o2 a 1erso+9
/hysical appearance
habits
profession
good qualities
special traits
Active /assive voice is used
Simple present tense to be used
6llustrate your description with concrete examples and anecdotes
So&ved E!a#%&e
Fa*"0a& Des*ri%"io+ o2 a 1os"#a+
Na#e: Adarsh, always can be seen at wor' in the society
7is work: to deliver the letters and parcels, sometimes wor's for eight hours a day six
days a wee',
2as to wor' in all weathers, has to be very cautious in delivering the letters, comes on a
bicycle from his place, sometimes can find no time to eat his food, does a strenuous =ob.
1o%0&ari":( /leasant countenance, friendly with children and helpful to old people and
women.
A 1os"#a+
Adars, is a #0*, k+ow+ 2a*e +ear o0r ,o0se a+d is o2"e+ see+ i+ ",e so*ie":. 7e is a
%os"#a+@ w,ose d0": is "o de&iver ",e &e""ers "o a&& ",e ,o0ses. 7e wears a >,aki
0+i2or# wi", a *a% a+d a (a4 "o *arr: &e""ers. 7e works 2or ei4," ,o0rs a da: a+d
si! da:s i+ a week. 7e ,as "o work i+ a&& ki+ds o2 wea",ers. 7is d0": (e*o#es "o04,
i+ ,o" seaso+ a+d rai+: seaso+. 7e ,as "o (e ver: *a0"io0s o2 doi+4 ,is d0": wi",
dedi*a"io+. 7e *o#es o+ a (i*:*&e "o rea*, ,is %&a*e o2 work. 7e is ver: %o%0&ar
a#o+4s" ",e &o*a& reside+"s (e*a0se ,e is ver: ,o+es" a+d ,e&%20& "o a&&. I+deed@ ",e
?o( o2 a %os"#a+ is ver: s"re+0o0s.
1ra*"i*e 80es"io+s:
&. ,rite a factual description of a new girl boy who has ta'en admission in your school.
). 0escribe a suspicious character whom you have seen hanging around in your colony
many times in the last wee'.
*. 4ive a detailed description of your 4randmother.
Fa*"0a& Des*ri%"io+ o2 a 1&a*e
6f describing a home, school, library, then first of all, mention the location,
dimension, facilities,
0escription of a city $ its geographical location, i.e. the province and the country,
the people, their dress, food habits, social and cultural customs, occupations etc.
Gollowed by remar'able feature li'e weather etc.
6t is usually written in Simple /resent and /assive forms.
So&ved e!a#%&e
Fa*"0a& Des*ri%"io+ o2 a C,i&dre+CsC ,o#e
Na#e: Nirmal 2riday, an integrated and well 'nitted <hildren1s1 home in 0elhi
Fa*i&i"ies: <omfortable rooms with bun'$beds, good mess, and indoor games for
recreation, a well maintained par' with swings, round$abouts, nurses, and mid$wives for
little children, doctors, and some volunteers visit it on Saturdays and Sundays
Ai#: to ensure that destitute and orphans can find a home and can find a place in
the society
Fa*"0a& Des*ri%"io+ o2 a C,i&dre+Cs 7o#e
6 had been to one of the <hildren1s 2ome KNirmal 2riday last Sunday on my
grandmother1s birthday to distribute sweets and clothes to the orphan children. This is a
big building in the heart of 0elhi with red walls. 6t is spread in an area of )### square
feet. There are forty spacious rooms to accommodate near about "## underprivileged
children. 6t has big, airy, well ventilated rooms with bun' beds and cupboards arranged
nicely for every child. There is a big recreation Kroom for the children to en=oy indoor
games. -n the right side of the building, there is a par' with swings and round$ abouts.
They have the facilities of doctors, nurses and mid$wives to ta'e care of any health
problems. Sunday is a day for the volunteers to come and spend a few moments with the
children. Nirmal 2riday really loo'ed li'e a place, which could provide a home to the
homeless.
1ra*"i*e 80es"io+s9
&. 0escribe your school to your /en friend.
). 0escribe the library of your school.
*. 4ive the description of your new house to your cousin, who is abroad.
Fa*"0a& Des*ri%"io+ o2 a+ Eve+"
&. 6f describing an incident or event in the past, /ast tense must be used
). /assive voice is used in scientific and technical writing or lab reports
*. Also 'nown as report writing

So&ved E!a#%&e9
Fa*"0a& Des*ri%"io+ o2 A++0a& Da: F0+*"io+
O**asio+$ Annual 0ay Gunction
I#%or"a+*e9 Achievements of the Iidyalaya are ac'nowledged, the meritorious students
in Academics and other fields are honoured,
Da"e@ "i#e$)#
th
April, )#&#at ! p.m. at Siri Gort Auditorium
C,ie2 G0es"$ 9r. I.L. Sharma, Senior Gaculty at N<:RT
7i4,&i4,"s: ,elcome Address, <ultural /rogrammes, /riEe 0istribution <eremony,
scintillating performance by young children
B&essi+4s by the <hief 4uest
Do"e o2 T,a+ks( 9r. R.N. 4upta, vice$ /rincipal
Fa*"0a& Des*ri%"io+
A++0a& Da: F0+*"io+
The )"
th
Annual 0ay Gunction of -xford /ublic School was held on )#
th
April
)#&# at ! p.m. at Siri Gort Auditorium, New 0elhi .The <hief 4uest of the occasion was
9r. I.L. Sharma, /rofessor at N<:RT, New 0elhi. The /rincipal of the School delivered
the ,elcome address, which was followed by the presentation of Annual Report. A
colourful <ultural /rogramme was presented by the School <hildren. The s'it P4eeta La
Savera Pwas appreciated by everyone. /riEes were given to the meritorious students in the
field of Academics, Sports, and other activities. -verall winners and Runners up trophy
were awarded to the best and second best house respectively in <. <. A and Sports. The
<hief 4uest 9r. /andey lauded the efforts of the students and the staff for putting up a
good show. 6n the end, the Iote of Than's was proposed by the School Iice$principal,
9r. R.N. 4upta.
1ra*"i*e 80es"io+s:
1. ,rite a report in about %# K&## words of the P33< master 9ind QuiE <ontest1
held in New 0elhi in which you represented your school.
;2ints( -rganisers of the show$ the 2T /ace, Number of schools participating, number of
students, host of the show, the response of the audience, some questions were as'ed from
the audience, awards were given to the best slogan and the best cheering team , ,inning
team and the runners$up team .
.
2. Oou are Aarushi 3hatt, a reporter wor'ing for The Tribune. Oesterday you
saw a ric'shaw full of school children overturn when it was hit by a speeding car.
As a result many children besides the ric'shaw puller were in=ured. ,rite a Gactual
0escription of the event in about %# K &## words.
.. DATA INTER1RETATION
This form of writing is used to interpret the facts presented in the form of a table, graph,
or chart. A graph or chart is a visual stimulus to compare the data. 0ata interpretation
requires an analysis and comparison of the given facts and drawing conclusions based on
the given data.
7ow "o do ",ese ":%es o2 =0es"io+s9
Study the charts and graphs carefully.
Read the title, axis labels and 'eys to understand the figure more accurately.
Refer to the picture ob=ectively. 0o not interpret more than what is given.
Bse <omparative degrees to interpret the data correctly.
6nstead of writing the numbers given in the pictures, write words li'e Ksharp
rise, improving, rise, ascend, highest point, decreasing, decline, fall drop etc.
Bsing adverbs is advised here$ sharply, slowly, tremendously, significantly,
moderately etc.
SOLDED EEAM1LE
1. Study the <hart given below, which is the result of the survey conducted in the public
schools and government schools of 0elhi. This depicts the types of activities the
teenagers ;age groups &*$&F years. are involved in, during their leisure time. <omplete
the summary in about %# words.
Tee+a4ers a+d Leis0re Ti#e
Now a da:s ",e "ee+a4ers are #ore "e*,+o9savv: ",a+ ",e *,i&dre+ 0sed "o (e "e+
:ears (a*k. T,e: do +o" %&a: 4a#es &ike L0do@ Carro#@ C,ess a+d o",er i+door
4a#es. More ",a+ F5G o2 (o:s a+d 4ir&s i+ ",e a4e 4ro0% o2 -.9-H :ears %ossess
0%
5%
10%
15%
20%
25%
30%
35%
40%
45%
50%
Watching TV Net Surfng Cell Phone Friends
Pulic
!o"ern#ent
",eir ow+ *e&& %,o+es. I+ a s0rve: *o+d0*"ed re*e+"&: o2 so#e 2000 s"0de+"s o2 "wo
&eadi+4 S*,oo&s9o+e 4over+#e+" a+d ",e o",er a 10(&i* S*,oo&@ ",e 2o&&owi+4 2a*"s
were revea&ed. W,i&e ",e s"0de+"s 2ro# %0(&i* s*,oo&s s%e+d #ore o2 ",eir &eis0re
"i#e i+ +e"9s0r2i+4 a+d "a&ki+4 o+ *e&&9%,o+es@ ",e 4over+#e+" s*,oo& s"0de+"s
s%e+d i" i+ wa"*,i+4 TD a+d "a&ki+4 "o ",eir 2rie+ds.
). Study the graph given below, which is based on a survey done with students of tenth
class in three different types of Schools K government, /rivate and a LI in a suburb of
9umbai. The chart depicts the number of students spea'ing :nglish and 2indi in class Q
of three different schools of 9umbai. -n the basis of the details given in the 3ar$graph
given below, briefly summariEe the data ma'ing comparisons wherever necessary in
about %# words.

2ints( <ompare the percentage of students spea'ing :nglish and 2indi in all the three
sections giving the possible reasons,
6n a study conducted on students of class Q of three various types of schools$
4overnment School, /rivate School and a LI, it was observed
that .........................................................................................................................................
................................................................................................................................................
-/T6-NS T-
<26>0R:N
AGT:R <>ASS Q66
9:06<A>,
:N46N::R6N4 R
-T2:R
/R-G:SS6-NS
!#S
2642:R STB06:S
6N 6N06A
RA3R-A0
)+S
:ARN6N4
>6I:>62--0
&!S
................................................................................................................................................
................................................................................................................................................
.........................................................
*. 4iven below is a /ie$<hart, which shows different types of electronic gadgets, used by
middle$class people in their houses on an average in 0elhi. -n the basis of the data
provided and the unit PScience Pof your 9ain <ourse boo', write in about %# words the
reasons of selecting the electronic gadgets for their houses(
/rice Range
0urability
>ower :lectric consumption
After Sales$services
+. -n the basis of the visual and verbal input provided and ta'ing hints from the
P:ducation P unit of your 9ain <ourse 3oo', write a summary in about %# words about
the possible options a <hild has after passing class Q66.
S-('(2?@)
Sa08(6(1?@)
LG("0@)
/+&rl*--l(20@)
;2ints( Students opt more for ta'ing admission in professional colleges li'e 9edical,
:ngineering, >aw, <A, Agricultural etc. than ta'ing admission in normal colleges to do
courses as 3.A.and 3.Sc..
................................................................................................................................................
................................................................................................................................................
................................................................................................................................................
..........................................
/. Dia&o40e Wri"i+4
The aim of this form of writing is to enable the students to elaborate upon the given
inputs in a grammatically correct and meaningful conversation. Students are supposed to
'eep a chec' on the syntactic errors.
1oi+"s "o re#e#(er w,i&e wri"i+4 a dia&o40e are:
Tense
Sub=ect$verb concord
Type of sentences$ assertive or interrogative
Appropriate inter=ections
<orrect use of imperatives, contractions and punctuations
So&ved E!a#%&e
&. 2ere is a conversation between a doctor and a patient .The patient has a problem of
getting a sleepless nights.
/atient( 2ello 0octor8
0octor( 2ello, so what is your /roblemM
1a"ie+": T0octor, 6 have a problem of not getting proper sleep at nightM
Do*"or: 6Pll give you a medicine for that.
1a"ie+": Than' TTTTTTTTTTTTTTTTTTTT.
Do*"or: Dust ta'e TTTTTTTTTTTTTTTTTTTT
1a"ie+": TTTTTTTTTTTTTTTTTTTTTTTT
Do*"or: TTTTTTTTTTTTTTTTTTTTTTTTT
1a"ie+": TTTTTTTTTTTTTTTTTTTTTTTTTT.
Do*"or: TTTTTTTTTTTTTTTTTTTTTT
>e: Do*a(0&ar:
to get a good nightCs sleep
to ta'e medicine
to ta'e a pill
to stay calm
1a"ie+": 2ello 0octor8
Do*"or: 2ello, so what is your /roblemM
1a"ie+": T0octor, 6 have a problem of not getting proper sleep at nightM
Do*"or: 6Pll give you a medicine for that.
1a"ie+": Than' Oou 0octor. 2ow often should 6 ta'e the medicineM
Do*"or: Dust ta'e one pill about *# minutes before you go to bed.
1a"ie+": Gor how longM
Do*"or: The prescription is for thirty days. <ome bac' if you do not get a proper sleep.
1a"ie+": 6s there anything elseM
Do*"or: 0onCt worry so much about things at wor'. .
1ra*"i*e 80es"io+s
&. <omplete the dialogue between a father and a son.
Gather( what have you decided to do after Q66 classM
Son( 6 have not decided anythingM
Gather( 0on1t you thin' this is the right time to give a thought for that tooM
Son( Oes, perhaps you are right.
GatherTTTTTTTTTTT
SonTTTTTTTTTTTTT..
GatherTTTTTTTTTTTTT..
SonTTTTTTTTTTTTTTT..
). <omplete the dialogue in about %# words(
/riya( ,here had you gone yesterday when 6 called you upM
Ruchi( ,ell, 6 had gone to my aunt1s place.
/riya( ,as there something specialM
RuchiTTTTTTTTTTT
/riya ( TTTTTTTTTTTTTTTTTTTT.
*. The following dialogue between two neighbours is incomplete. <omplete the dialogue
in about %# words.
9itali( 2ello, 61m 9itali, your new neighbour..
Sumita( 2ello, /lease to meet you. 61m Sumita.
9itali( TTTTTTTTTTTTTTT..
Sumita( TTTTTTTTTTTTTTTT..
9itali( TTTTTTTTTTTTTTTTTT.
SumitaTTTTTTTTTTTTTTTTTTT.
LONG COM1OSITION
Lo+4 Co#%osi"io+ (ased o+ vis0a&I ver(a& s"i#0&0s9&e""er $2or#a&I
i+2or#a&) or E&e*"ro+i* #ai& 0A #arks
Der(a& s"i#0&0s *a+ (e i+ ",e 2or# o2 adver"ise#e+"@ +o"i*e@ +ews%a%er@
re%or" a+d vis0a& s"i#0&0s *a+ (e i+ ",e 2or# a %i*"0re@ 4ra%,@ dia4ra#
e"*.
Read the question very carefully
Ta'e hints from your 9<3
,rite the points in *$+ paragraphs
/lan, organiEe and present ideas coherently
Bnder content, credit is given to the students1 creativity in presenting the
ideas.
Divisio+ o2 #arks
Accuracy, fluency $ ) mar's
I+2or#a& Le""ers
USender1s addressV
;0ate.
0ear Uname father momV
UBOD3V
3o0rs a22e*"io+a"e&: $ 2or a&& re&a"io+s)
3o0rs si+*ere&: $ 2or 2rie+ds)
Sender1s Name

:xpression, style $ ) mar's
<ontent $ + mar's
For#a" o2 ",e &e""ers
I+2or#a& &e""er
Language used should be simple and precise and to the point. Thus si#%&i*i":
a+d *&ari": is a must.
To+e of the letter should depend upon the relationship of the writer with the
addressee.
6t should never lose res%e*" which the addressee deserves.
So&ved E!a#%&e
Oou are Aditya. Oour younger brother is staying away in a hostel in an :ngineering
<ollege. Oou are afraid that he may fall into bad company and may resort to habits li'e
For#a& Le""ers
USender1s addressV
;0ate.
;Receiver1s address.
Respected sir ma1am
SB3(
UBOD3V
3o0rs 2ai",20&&:
US64NATBR:V
Name
0esignation
Why drug abuse?- no single reason- addi!s s!ar!s ou! o"
uriosi!y- "or #leasure- !o ge! o$er boredo%- de#ression
sy%#!o%s- loss o" in!eres!s in s#or!s and daily rou!ine &
a##e!i!e loss & unlear $ision & %ood hange !e%#er
!an!ru%s'
0rug$addiction. Bsing the notes below, write a letter warning him about the dangers of
drug addiction in about &)# words.
New >ines, 2aEratgan=
>uc'now
#" August )#&#
0ear Samya'
6 am writing this letter to tell you the bad effects of drug addiction. Oou have entered a
phase in your life, where you will see many new things in your surroundings. Remember,
initially the consumption of drugs starts out of curiosity, but when coaxed by friends,
people start consuming it for pleasure. -ver a period of time, it becomes a habit and then
you cannot leave it very easily. Some people start ta'ing drugs to get out of boredom but
after a period of time it becomes a necessity and they develop the symptoms of
depression. The person becomes a slave to drugs and if he is deprived of drugs, he soon
develops severe withdrawal symptoms. 6t slowly 'ills the person from inside.
Remember, it is a curse and you should never try it, even when you are compelled to do it
or even out of curiosity. 6t is difficult to get out of this habit 2e has to be admitted in
clinic for rehabilitation, which is a long process.
0rugs are a curse to the individual, the family and the society. So you should never even
thin' of trying it. Oou are now in a college away from home and there is nobody who will
ta'e care of you. So, you have to be very cautious in your college life, 6f you want to
succeed in life, a good health is the first step to your success. 9ummy and /apa =oin me
in conveying their love and affection.
Oours affectionately
Aditya
1ra*"i*e 80es"io+s
8-. ,rite a letter to your father as'ing him for more money as your new academic
session has started and you need to buy stationery, uniform etc.
82. ,rite a letter to your friend inquiring about the health of his mother who has =ust
been diagnosed with malaria. Also give suggestions regarding protection from malaria.
8.. ,rite a letter to your sister congratulating her on her outstanding result and as'ing
her about her hostel life.
8/. Oou have recently returned from an educational trip to Daipur. ,rite a letter to your
friend AnshuAnshul sharing your experience. Sign yourself as A=ayA=aya.
For#a& &e""er
For#a" o2 a For#a& &e""er i+*&0des
Se+derCs address
Da"e
Re*eiverCs address
S0(?e*"
Sa&0"a"io+
Bod:I*o+"e+" a+d
Co+*&0sio+
1ra*"i*e 80es"io+s
8 -. Oou lost your wallet on the train. No one has found it yet.
,rite a letter to the station master of the local train station. Oou should write at least &"#
words. 6n your letter
W 0escribe your wallet and what was in it
W Request information about their lost and found service
W Say what other action you would li'e the station master to ta'e
82. ,rite a letter to the editor of a newspaper expressing your concern on the "#S
reservation announced by the government. Leeping in view the unit P:ducation1 and
showing your grave concern over the aid of court abetting it. Not only this, the states can
exceed the limit by producing quantifiable data on bac'wardness.
Some points(
3rilliance is caste blind, but the S<1s verdict will help politics rule 3y all owing the
reservation to exceed "#S the last hope of 6ndia becoming a modern society =ust died
foreverT
They would have to ac'nowledge that social =ustice is best secured when democratic
norms allow the best to rise to the topT
The tendency will be to mar' up the students and in this process, general standards
tend to fallT
-ne can understand and support reservations for Scheduled <astes and scheduled
tribes because these communities were not able to gain any social or economic assets
in ancient times but now it is a thing of the pastT
/reference can be given to quotas but on the same level of meritT
8.. ,rite a letter to the editor of a newspaper expressing your views on the rise of the
number of water$borne diseases li'e malaria, dengue etc. with the onset of the monsoon.
Also give suggestions on how to curb or prevent them. Ta'e ideas from the unit X2ealth
and 9edicineY and use the hints given below(
Tips(
/revent yourself from getting drenched in the rain$use a complete set of rain gear$
a hooded raincoat, an umbrella and a pair of waterproof shoes
6f you are out of home, opt for mineral water over tap water. At home boil water
before drin'ing it, to 'ill the germs or filter itT
6f you get drenched in the rain, =og till you sweat. This will 'eep the body
temperature in chec'
To avoid catching cold, wash your hands frequently with a medicated soap. Bsing
a sanitiEer would be useful too
Supply your body with enough of vitamin A and <. a healthy platter of steamed
vegetables well help prevent the common cold virus
/revent the breeding of mosquitoes
Bse spraysinsecticides, repellants, nets etc.
See a doctor for even a small illness
8/.The senior citiEens are insecure about many things. They are soft targets for robbery,
fraud, property disputes and emotional and physical abuse. ,rite a letter to the editor of a
national daily expressing your concern about the insecurity and fears that the elderly
people are faced with. Suggest ways as to how the government could play an important
role in protecting their interest and also how they should deal with their problem.
E9MAILS
An :$mail, or electronic mail, is a store and forward method of composing, sending,
storing and receiving messages over electronic communicating systems. 6t is the quic'est
way of communicating in writing.
E#ai& wri"i+4 has become a large part of modern communication, particularly in
business. The world has become much smaller now that we have the ability to send and
receive email messages over great distances at an incredible speed.
GAQS a(o0" E(electronic)-Mail
7ow #0*, s,o0&d I wri"e i+ a+ e#ai&B
:mail is meant for =0i*k@ si#%&e *o##0+i*a"io+. 6t should have roughly + or "
paragraphs at the most. 9ost people have a limited attention span with email $ if they are
receiving a lot of mail you want to get the main message explained in the shortest
possible space.
Do s%e&&i+4 a+d 4ra##ar #a""er i+ e#ai&B
/oor spelling and grammar show a lac' of a""e+"io+ "o de"ai& and send the wrong
message about yourself and how you do business. 9ost email programs today have built$
in spell chec'ers so there is no excuse8 Oou wouldnCt send a letter that was poorly
punctuated and uses no capital letters $ why not ma'e sure your email messages loo'
professional tooM
W,a" do **: a+d (**: #ea+B
There is a certain etiquette with email, especially in the use of the *ar(o+ *o%: ;cc (. and
(&i+d *ar(o+ *o%: ;bcc (. fields. 6t is courteous to add addresses to the C**:C field if those
people need to 'now about the sub=ect but are not required to act on the contents.
The C(**:C field is useful where discretion is required. /eople in this field are concealed
from other recipients in the CToC, Ccc(C and Cbcc(C fields. They can themselves see others in
the CToC and Ccc(C fields but not the Cbcc(C fields.
6n this example, an email is sent directly to
Dohn by 3ob. 2e copies Tom by including
TomCs address in the cc( field. 2arry and Sue
are blind copied $ they can see who the
original recipients were but Tom and Dohn
wonCt see their names.
2ow to write an : 9ail
&. Start with salutation.
). ,rite in short paragraphs.
*. Bse capitals appropriately.
+. Sign off the : 9ail
o Bse 5yours sincerely7;when you 'now the name of the addressee.and
Oours faithfully when you have addressed it to 50ear Sir9adam7.
o Bse best regards or 'ind regards
". Bse sensible : 9ail signature( 0o include your name, email address, telephone
number and postal address.
: 9ail messages contain two ma=or sections(
&. 2eader( 6t contains sub=ect, sender, receiver and date.
). 3ody consist of the messageletter
6mportant guidelines to approach towards an : 9ail(
S0(?e*":
6t gives a context to the message.
6t should be short and not a complete sentence.
6t should provide a clue to the content of the message.
Be4i++i+4:
0ear Sir or first name of the person.
Start with greetings.
,hen replying to the message, than' you for your message6 received your
message.
To+e:
/lease let me 'now ;when you expect a reply..
/leaseLindly indicate ;when you need help..
0o not use capital letters.
1ara4ra%,s:
:ach idea must be in a separate paragraph.
Bse complete sentences avoiding S9S language.
DOs and Donts about E Mail
DoS
Use an informative subject line, which says what the email is about.
Write the most important information first.
Use numbers and bullet points to make the message clearer.
Use simple grammar. void things like the passive. !s emails are a fast means of
communication, they tend to be less wordy and comple" than formal letters.#
Write short sentences.
Use paragraphs to keep the email clear and easy to understand
Donts
Write $hello% as your subject line.
Write about irrelevant issues. &he reader will soon hit $delete% if the email doesn%t
get to the point.
'ive personal information that you don%t want anyone else to know. !&he
(mail could end up in the wrong hands#
Use capital letters to write whole words as in emails, this is considered
shouting.
Use different fonts in the email !the recipient%s computer may not be
compatible#
1i*"oria& Re%rese+"a"io+ o2 a+ E Mai&
Use )talics !the reason may be misunderstood, due to cultural
differences#.
Use e"clamation marks.
Use abbreviations like co* and uni, as the recipient may not understand them.
Use acronyms like B&W for the same reason.
Use smileys. &hey may be misunderstood and come across as unprofessional.
Sa#%&e E#ai& Le""er
amitsinha.mca)##%Zgmail.com
<ommonwealth 4ames )#&#
0ear /ranav
6 am really excited to see the -pening <eremony of <ommonwealth 4ames )#&#. 9y
parents could manage some tic'ets of the -pening <eremony for us. 6 will write to you
complete details of the -pening <eremony after watching the programme.
>oo' forward to hearing from you.
Sincerely yours
Amit
a) FORMAL E9Mai&
S0(?e*" Li+e o2 E#ai& Messa4e( <ommunications 0irector /osition $ Oour Name E#ai&
Messa4e:
0ear 2iring 9anager
6 read your =ob posting for a <ommunications 0irector with interest.
6n my position as <ommunications 0irector for QO[ <ompany, 6 wrote articles for the
company website, managed guest author submissions, and wrote and sent a wee'ly email
newsletter to subscribers.
,hile Assistant <ommunications 0irector for Assembly person Santanu 9u'her=ee, 6
researched, drafted and amended legislation, wrote press releases, and was responsible
for office communications and correspondence.
6 also have extensive experience writing on a freelance basis on labour issues, which, 6
believe, would be an ideal match for this position.
Additional writing samples and my resume are attached. 6f 6 can provide you with any
further information on my bac'ground and qualifications, please let me 'now.
6 loo' forward to hearing from you.
Day 9ehra
Address
:mail
2ome /hone
<ell /hone
() INFORMAL E9MAIL
So&ved E!a#%&e
S0(?e*": 10(&i*a"io+ o2 Ar"i*&es
2iTe=as,
Than's for the e$mail. 6t is always nice to hear from people li'e you.
6 have not got any reply, a positive or negative one, from SA4: /ublications yet.
>etCs wait and hope that it will publish a boo'.
2ave you finished your paperwor' for writing academic articlesM
6f you have some free time in near future, 6 would li'e to meet you and
explain to you our next pro=ect.
,e should dine out in 9umbai if we are accepted.
,e need to celebrate ourselves, donCt weM
>etCs have a small end$of$the$year party8
Sincerely,
Ii'ram
1ra*"i*e 80es"io+s:
For#a& E#ai&:
8-. Ta'ing ideas from the following advertisement, write an email in not more than
)## words to your friend to invite him to ta'e part in the 5<hoose Oour <areer <ampaign
)#&#7
6N<>B0:( 0ate, time and venue of the campaign.
2ow the campaign will benefit the students.
82. <rossing the railway lines carelessly not only endangers people1s lives but also
causes delay of trains and loss to railways. So Northern Railways has decided to ma'e an
appeal to various schools in order to educate people about safety on railway trac's.
,rite an e mail to the :ditor of a national daily expressing your concern over the
increasing number of causalities on the railway trac's ta'ing the clue from the following
verbal stimulus
C$%%S& '%() C*)&&) C*+P*,!N 2010
,6>> 3:
6NAB4BRAT:0 3O
SR7I MANGAT RAM SING7A
2-N13>: 96N6ST:R -G :9/>-O9:NT AN0 >A3-BR
-N )"
th
-ctober )#&#.
DENE:
AB06T-R6B9, 0:>26 SA<26IA>OA, N:, 0:>26
TIME: --.007RS
ORGANIJED B3:
&+( ,)-(.&/-&( /0 (M1L/2M(3&
'/4&. /0 3.& ,(L+)
The campaign starts from 25
",
"o 2H
", O*"o(er 20-0
Basi* o(?e*"ive o2 ",e *a#%ai4+:
/rovide educational and vocational guidance to school students.
I+2or#a& E#ai&:
8-. Oou are planning to go on a tre''ing expedition to 9anali. ,rite an email to your
friend inviting him to come with you for a five day trip. 0escribe the wonderful
atmosphere of 9anali and invent the dates, time and other parts of the schedule. Sign
yourself as AmanAn=ali.
82. Oour parents are celebrating their )"
th
wedding anniversary, write an :mail to
your friend inviting himher for the same in about &"# words.
2ints(
0ate, time and venue.
Names of some of the invitees.
8.. Bnder Gulbright Oouth :xchange program, you had gone to /ennsylvania. There
you stayed for a period of one year. After returning from there, you wish to than' your
host parents for providing you a comfortable stay and ma'ing the stay memorable. ,rite
an informal :mail in not more than &"# words.
>-N4 <-9/-S6T6-N
A &o+4 a+swer =0es"io+ $-509-A0 words #i+i#0#) i+ ",e 2or# o2:
Ar"i*&e wri"i+4
S%ee*,
Diar: e+"r:
S"or: wri"i+4
De(a"e
a) Ar"i*&e Wri"i+4
10r%ose o2 wri"i+4 a+ ar"i*&e
An article is piece of writing, comprising about &"# to &%# words, which may generally
focus on themes such as a social concern, narrating an event and describing a place.
For#a" o2 a+ ar"i*&e i+*&0des:
2eading
6ntroduction ;6t may start with a slogan or a question or a startling fact, figure or
statement..
3ody$)
nd
and *
rd
paragraphs may include causes, effects, present state of affairs
etc.
<oncluding paragraph K suggest remediesmeasures to be adopted your
expectations ways of improving the situation etc.
Marks Divisio+:
<ontent( #* mar's
Accuracy( #* mar's
Gluency( #) mar's
Total( #% mar's
Excerpts of an article published by DevelopedNation.or
1ra*"i*e 80es"io+s
8-. Wri"e a+ ar"i*&e o+ ",e 4ive+ "o%i* 0si+4 ",e ,i+"s 4ive+ (e&ow i+ +o" #ore ",a+
-50 words:
Mobile !hones" a #odern utility or a health ha$ard
7INTS
&. Bsage of mobile phones
a. "#S of 6ndia is going to have a mobile phone device by )#&)
b. 4rowing at a fast rate
). Btility of mobile phones
a. Leeps you well$connected
b. 6nexpensive
c. 2andy
d. 2elps in personal and professional tas's
*. 2ealth haEards posed by mobile phones
a. :lectromagnetic radiations
b. 0istraction while driving
c. Average mobile carries &% times more harmful germs than a flush handle
82. Wri"e a+ ar"i*&e o2 -509200 words o+ ",e +eed "o %rovide se*0ri": a+d 0%kee% o2
,is"ori*a& #o+0#e+"s "o %ro#o"e "o0ris#.
%ndias &rchitectural 'eritae" (rency of (p)eep and Safety
7INTS
&. 6mportance of historical monuments in promoting tourism
). Bp'eep of historical monuments
a. 9any historical monuments are not adequately maintained.
b. Several historical sites are also not aggressively promoted among tourists
c. Bnsupervised trespassing at monument causes damage
d. A discipline that fosters cleanliness among general public is need
e. 9any places do not even have basic public amenities.
*. Safety of tourists
a. 6ncidents of crime and fraud targeting the tourists are on the rise
b. Brgent need for the government to ensure proper security
8. ,rite an article on the controversy surrounding experiments at >arge 2adron <ollider
;>2<..
*are 'adron +ollider" & ,i ,an of +ontroversy or & -iant *eap for the Man)ind
26NTS
>arge 2adron <ollider ;>2<. is a giant tunnel$shaped apparatus, in SwitEerland,
that is designed to conduct experiments on sub$atomic particles.
Bsing the >2<, scientists are see'ing answers to fundamental concepts of
science, including the big bang theory and creation of universe.
9any people have raised questions on the security, claiming that certain
experiments may create blac' holes that can destroy our planet.
Scientists re=ect the security concerns, saying that there is no ris' of damage.
The controversy over the security has been the sub=ect of many popular movies
and novels recently, including American film PAngels R 0emons1 starring Tom
2an's.
(. S%ee*,
10r%ose o2 a s%ee*,
A speech is a form expression of views by an individual addressed to the public.
For#a" o2 a s%ee*, i+*&0des:
Salutation or greeting
-pening sentence must clearly mention the topic of the speech
Quotations, sayings, relevant example, along with other figures of speech may be
used.
3e mindful deviating from the stated topic.
Sum up the views in conclusion.
Divisio+ o2 #arks
<ontent( #* mar's
Accuracy( #* mar's
Gluency( #) mar's
Total( #% mar's
So&ved E!a#%&e
To%i*: No",i+4 is I#%ossi(&e
7INTS:
&. 2uman flight was once considered impossible.
). Scientific progress, powered by reason, made human flight possible
a. ,right 3rothers made the first successful flight in an aircraft in &F#&.
b. NASA made the first manned flight to the 9oon in &F!F.
*. The achievements of the sciences have driven the human spirit that see's to tame
the impossible.
No",i+4 is I#%ossi(&e
Ooung men, women, and children of this great country, 6 say to you today that PNothing is
6mpossible1T
2uman flight is nothing but creativity of human mind and it undergoes several struggles
to achieve excellence. 6n &%F", a great well$'nown scientist >ord Lelvin, who was the
/resident of Royal Society of >ondon said, ?any thing heavier than air cannot fly, and
cannot be flown.? ,ithin a decade, ,right 3rothers proved man could fly of course at
heavy ris' and cost.
-n the successful completion of 9oon 9ission in &F!F, Ion 3raun, a very famous roc'et
designer, who built Saturn$I, to launch the capsule with astronauts and made moon wal'
a reality, in &F@" said ?6f 6 am authoriEed, 6 will remove the word impossible?.
6n ancient days, /tolemaic astronomy is a widely used system in calculating the dynamics
of various stars and planets. Assumption by then was that the earth is flat. ,hat a
scientific struggle had to ta'e place to prove that the earth is spherical in shape orbiting
around the sun. The three great astronomers <opernicus, 4alileo and Lepler had to give a
new dimension to the world of astronomy. Today we ta'e it for granted that earth is a
globe, orbiting around the sun, and the sun orbits in the 9il'y ,ay. All the technological
advancements we have today are the outcome of scientific exploration of scientists of
earlier centuries. At no time, man was beaten by problems. 2e strives continuously to
sub=ugate impossibility and then succeeds.
According to the laws of aerodynamics the bumble bee should never be able to fly.
3ecause of the siEe, weight, and shape of its body in relationship to the total wing span,
flying is scientifically impossible. The bumble bee, being ignorant of scientific theory,
goes ahead and flies anyway.
1ra*"i*e 80es"io+s9
8-. Oou have been requested to spea' in the School1s morning assembly on the
importance of politeness, courtesy and compassion in our day$to$day communication.
0raft a speech of &"#$)## words.
26NTS(
&. 9odern life is fast and result$oriented.
). Nowadays, we tend to be to$the$point and curt.
*. 0ecline in values and common courtesies YThan' Oou and /leaseY
+. The use of abusive words is common.
". :ven some of our public leaders are found lac'ing these courtesies.
!. ,e, students, need to maintain decency and decorum in day to day life
82. Oou are invited to address a gathering of your fellow students on the sub=ect of
i#%or"a+*e o2 %,:si*a& ,ea&", i+ o0r &ives. 0raft a speech of &"#$)## words using
cues offered.
7INTS:
&. 2ealthy mind lives in a healthy body.
.his speech is an address to %ndias youth by !resident &!/ &bdul 0ala# abrided1
#odified and edited for education use.
). >ac' of time for physical activity because of the demands of modern lives
a. ,or'ing on computers for hours on$end.
b. ,atching television for unsupervised duration.
c. :xcessive burden of studies.
d. Attraction towards =un' food.
*. 6gnoring the health increase the ris' of many diseases
a. -besity
b. 0epression
+. 0iscipline is the 'ey to resolving these concerns
a. 0oing daily exercises
b. :ngaging in sports activity
c. Bse cycles and wal'
d. Strictly limit fast$foods.
.
C. Diar: E+"r:
10r%ose o2 a diar: e+"r:
A diary entry is an individual1s account of a day of his or her life.
For#a" o2 a+ diar: e+"r::
0ate, day and the time must be recorded on the top$left.
Ierb tenses used most frequently in a diary entry are PSimple /ast1, P/resent
/refect1 and PGuture1.
Oou may begin your entry with a general sentence describing the day or your
momentary feelings.
6n the body, you may discuss an event, your feelings towards that event and if and
how that has affected any of your future plans.
Oou may close your entry with a final remar' on the event discussed or your
future course of action.
Break90% o2 #arks
<ontent( #* mar's
Accuracy( #* mar's
Gluency( #) mar's
Total( #% mar's
So&ved E!a#%&e:
Q&. Today, you visited your ancestral village and watched the farmers harvest a crop of
wheat. 9a'e an entry into your diary describing your experiences at the village.
7INTS(
&. April is usually considered the month of harvest.
). 6t is time of hectic activity for the farmers as they reap their crop.
*. They used many different instruments, hand sic'le, tractor, and motor harvester.
+. Grom afar, the field of crop shone li'e gold and the activity of harvesting seemed
orderly.
". ,omen ma'e a significant contribution in the hard wor', winnowing the produce
and separating grain from chaff.
7arves": Rea%i+4 ",e 1rod0*e o2 7ard Work
0ate(
0ay(
Time(
Today 6 spent the whole day at my ancestral village.
6 was excited to see the harvest. 6 was particularly impressed by the farmers as they
reaped their crop with great care and precision. 6 saw them wor'ing through a field of
golden crop, mostly with their bare hands or with =ust a sic'le. 6 also watched some of the
farmers used tractors and large harvesters.
Among the other activities, the women fol' of the village were collecting the harvest crop
and separating hay from grain. 6 was over=oyed to note that even the hay, which may be
waste for the lay$eye, was neatly stac'ed and preserved for later use. As you may have
imagined, with the grain, 6 found them to be even more careful, as if each grain counted.
,hile going about their tas's, women fol' liven up the day by singing songs of =oy and
than'ing the gods for prosperity. <hildren run through the fields, which 6 first thought
was =ust their playful nature, but as found in everything else, 6 was surprised to note that
there was subtle purpose to it. They made sure that no birds prey on the open mounds of
grain.
Grom afar, the field of crop shone bright and splendid, turning the boo'ish learning that
real wealth lies in hard labour which can even turn fallow land into gold. And to 'now
that this cycle of harvest has continued from almost the time the man'ind became
civiliEed, 6 felt proud of my heritage.
1ra*"i*e 80es"io+s
8-. 6magine you are an astronaut and you have landed on 6ndia1s first manned mission to
the 9oon. Record your experience in your diary in &"#$&%# words.
26NTS
&. The 9oon, which was always on ob=ect of the s'y, is now beneath my feet.
). 0ue to lac' of gravity, a small step also tends to be a big =ump.
*. The land is rugged and grainy. There is nothing moving.
+. -ur planet shines li'e a blue pearl.
82. Oou were witness to a discussion regarding division of your ancestral property, which
turned ugly and bitter. Oou were disappointed to see that a division of the property had
turned close relatives apart. 9a'e an entry into your diary regarding the emotional
turmoil you are going through as a result of the bitter dispute. <omplete within &"#$&%#
words.
7INTS
&. ,ent to attend a discussion regarding division of ancestral property
). 0iscussion turned into a verbal fight between close relatives
*. 0ivision of the family beyond conception
+. >ove and affection disappearing as materialistic concerns dominate
8.. Today you appeared for your final exam for :nglish. Although you prepared
extensively, yet you were not confident, surrounded by fear and doubt. 3ut, much to your
surprise, as you started writing your answers, you grew confident. As your hard wor'
supported you, you were to finish on a very positive note, with feelings of success. 9a'e
an entry into your diary about this experience in &"#$&%# words.
7INTS:
&. Studied till late in the night, but still not confident the next morning.
). Anxiety causing fear and unease before the exam
*. After starting, things fell into place without much stress
+. 3y the end, a feeling of elation and achievement dominated low spirits
d. S"or: Wri"i+4
10r%ose o2 a s"or:K
A story is a narration of a set of fictitious events.
For#a" o2 a s"or: i+*&0des:
Set the context or perspective K P-nce upon a timeT1, Pit happened soT1, P>ong
agoT1
<haracter are introduced and developed through dialogues and narrative.
/lot is the heart of the story, based usually on an incident.
,hen plot thic'ens, it reaches a climaxH this is often the most interesting, yet
unpredictable, turn of the story.
:nd of the story may provide brief detail of the charactersY future.
-ften stories are used to convey a moral message.
Divisio+ o2 #arks 0A #arks
<ontent( #* mar's
Accuracy( #* mar's
Gluency( #) mar's
So&ved E!a#%&es:
Q&. 6nterpret the pictures below and write a short story in &"#$&%# words.
O2 1ro0d Do4s a+d S#ar" 1arro"s
-nce it so happened that a dog and a parrot found themselves on a boat in the ocean. The
dog, called 9ilo, being the favorite pet of the captain, used to wal' about the boat with
great pride. The pride of the dog was legendary among all the pets and animals that
stayed on the boat. 2owever, 9ilo1s pride was not unfounded for the dog had fiercely
fought and brutally defeated many of his rivals.
-n the boat, there lived another famous pet, that of the quartermaster, a parrot called,
/aulie. /aulie had a mythical aura of wisdom and prudence around him. 9ay be it was
his old age or his unintelligible uttering, li'e that of an old sage, that gave him this fame.
Nobody 'new what he had really done to accrue such image, but, everyone was
respectful of the good ol1 /aulie.
-ne day, /aulie awo'e from his long slumber and perched himself on the dec'. The lure
for /aulie, li'e all the other pets, including, the great 9ilo was the beautiful sunlight that
graced the dec' this morning. 9ilo, all =oyful and playful, sought to assert his presence
and once again dominate his small echelon of pets in the boat. 9ilo started daring the
pets into a brawl, but none were aggressive or brave enough, submitting the great 9ilo
one after the other. Ginding total lac' of any worthy or willing any opponent, 9ilo, as in
the past, started ridiculing the other pets and proclaiming his superiority.
Grom aboard an old drainpipe, /aulie was smiling at this display of vainglory. 9ilo
caught /aulie and demand, 5-l1 /aulie, what causes you smir', so sheepishly, this
morningM7 /aulie 'ept calm and remained silent. 9ilo edged further, more
authoritatively, 5/aulie, you have respect, but with no good reason, 6 demand you either
prove yourselves against me or wipe that smir' off your old mangled face.7 /aulie
sleepily uttered, 50irty dog, dirty dog7. 9ilo1s anger surged and seethed beyond bounds,
5/aul, 6 will ma'e you pay for, retract your ugly remar's and submit while there1s still
time.7 -ld /aulie, unperturbed, 50irty dog, dirty dog87 Now there was no stopping, as he
=umped up to the high perch of /aulie, as if plotting a barbarian assault. All the other pets,
astonished at this turn of events, closed their eyes, not willing to witness their old /aulie
slain in fight.
6n a split second, however, the other pets heard a quic' flutter of wings and 9ilo
swooping into the drainpipe. 59iracle87 cried the other pets as they hailed this smart
victory.
As was later found, the disaster was aggravated as 9ilo landed on his master, the
<aptain1s head through that drainpipe and got a good beating in punishment.
1ra*"i*e 80es"io+s:
8-. 6nterpret the pictures below and write a short story in &"#$&%# words.
TTTTTTTTTTTTTTTTTTTTTTTTTTTTTTTTTTTT
TTTTTTTTTTTTTTTTTTTTTTTTTTTTTTTTTTTT
TTTTTTTTTTTTTTTTTTTTTTTTTTTTTTTTTTTT
TTTTTTTTTTTTTTTTTTTTTTTTTTTTTTTTTTTT
TTTTTTTTTTTTTTTTTTTTTTTTTTTTTTTTTTTT
TTTTTTTTTTTTTTTTTTTTTTTTTTTTTTTTTTTT
TTTTTTTTTTTTTTTTTTTT.
82. 6nterpret the pictures below and write a short story in &"#$&%# words.
LLLLLLLLLLLLLLLLLLLLLLLLLLLLLLLLLLLL
LLLLLLLLLLLLLLLLLLLLLLLLLLLLLLLLLLLL
LLLLLLLLLLLLLLLLLLLLLLLLLLLLLLLLLLLL
LLLLLLLLLLLLLLLLLLLLLLLLLLLLLLLLLLLL
LLLLLLLLLLLLLLLLLLLLLLLLLLLLLLLLLLLL
LLLLLLLLLLLLLLLLLLLLLLLLLLLLLLLLLLLL
LLLLLLLLLLLLLLLLLLLLLLLL.
Q*. 6nterpret the pictures below and write a short story in &"#$&%# words.
LLLLLLLLLLLLLLLLLLLLLLLLLLLLLLLLLLLL
LLLLLLLLLLLLLLLLLLLLLLLLLLLLLLLLLLLL
LLLLLLLLLLLLLLLLLLLLLLLLLLLLLLLLLLLL
LLLLLLLLLLLLLLLLLLLLLLLLLLLLLLLLLLLL
LLLLLLLLLLLLLLLLLLLLLLLLLLLLLLLLLLLL
LLLLLLLLLLLLLLLLLLLLLLLLLLLLLLLLLLLL
LLLLLLLLLLLLLLLLLLLLLLLLLLLLLLLLLLLL
LL
e. De(a"e
10r%ose o2 de(a"e
The purpose of a debate is to show s'ill and ability in arguing.
For#a" o2 a de(a"e:
Salutation K PRespected chairperson, honourable =udges, and my dear friendsT1.
6ntroduction K P6 am here to present my views for against the motionT1
3ody of the argument K may include views, facts, rebutting the statements,
questions etc.
<ommon phrases K P6 strongly feel thatT1, P6 would li'e to draw your attention
toT1, P9ay 6 as' all presentT1, P6 wholeheartedly oppose supportT1, P6n my
opinionT1
Divisio+ o2 #arks:
<ontent( #* mar's
Accuracy( #* mar's
Gluency( #) mar's
Total(#% mar's
So&ved E!a#%&e
8-. The motion of the debate is P'o#e2or) should be abolished fro# school
curriculu#1. /lease write a debate in favour of the motion in about &"#$&%# words.
7INTS
&. 2omewor' $a burden on the child
). >eaves no time for physical activities
*. -ften done by parents and not by the students
+. Students often copy the homewor', without understanding
". 6t is a wastage of time and energy
7o#ework s,o0&d (e a(o&is,ed 2ro# S*,oo& C0rri*0&0#
;in favour of the motion.
Respected chairperson, honorable =udges, and my dear friends, 6 stand here today to
spea' for the motion that home wor' should be abolished. At the very outset 61d li'e to
raise this question$6s the home wor' to be done by parents, tutors, or the already
overburdened studentsM
6n my opinion, home wor' is rarely done at home. 6t is often copied in bus, in between
classes, or these days reproduced over telephone. 6 submit, in fact, that the homewor'
frequently becomes such a burden in many families that parents decide to dispense with it
in the speediest and the most convenient manner. As a result, older children, private
tutors, or even parents themselves set to wor' completing it. Gurther, 6 feel very strongly
that homewor' stunts a child1s physical, emotional, and social development. 6t simply
programs the student to become competitive stereotypes. 6 should li'e to draw your
attention to the fact that this monster K2omewor'$ acquires demonic proportions and
finally becomes a holiday horror. 6 fail to understand why my worthy opponents have
chosen to support something that consumes so much of the student1s free time and
energy, both in term$time and holiday time.
9y final argument against homewor' is that it is of limited value. 3y this 6 mean that the
type of homewor' most often set is overwhelmingly dull and mechanical, and seems to
be set purely out of tradition, so as to 'eep students busy out of school.
6 support the motion that homewor' should be abolished.
1ra*"i*e 80es"io+s
8-. The motion of the debate is PModern livin has #ade the people 2ea)1 unhealthy
and disease-prone1. /lease write a debate against the motion in about &"#$&%# words.
7INTS
&. modern living has led to improved longevity
). modern medicine has reduced infant mortality
*. technology has made the life easier and stress$free
+. aware of health and hygiene due to media
". permanent cure found for many dreadful diseases K small pox, malaria, polio
82. The motion of the debate is PEnd 3ustifies the #eans1. /lease write a debate speech
in favour of the motion in about &"#$&%# words.
&. Bltimately, results count for more than efforts.
). -nly shir'ers hide behind the value of Pmeans1, whereas Pwor'ers1 succeed.
*. :nd is the only practical benchmar' of performance, rest is sub=ective.
+. Not everyone has equal P9eans1, but P:nd1 is equal for everyone.
8.. The motion of the debate is P-radin is better than #ar)in for evaluation of
exa#ination perfor#ance1. /lease write a debate in favour of the motion in about &"#$
&%# words.
&. 9a'es the evaluation fairer
). Avoids overwhelming fixation over small decimal number
*. /romotes healthy competition, avoids unhealthy =ealousy
+. Reduce unnecessary mental stress on students
". Some teachers may give ! mar's, while others give + mar's for the same
performance. 4rading will eliminate these differences.
Se*"io+9C $Gra##ar)
A variety of short questions based on 9<Q, involving the use of particular structures
within a context will be tested. Test types used include(
;a. :diting ;+ mar's.
;b. -mission ;+ mar's.
;c. <loEe gap$filling ;one word. ;+ mar's.
;d. Sentence Re$ordering ;+ mar's.
;e. 0ialogue <ompletion ;+ mar's.
;f. Sentence Transformation includes(
Reported Speech ;+ mar's.
Active$/assive Ioice ;+ mar's.
Newspaper 2eadlines ;+ mar's.
;A. EDITING
1oi+"s "o Re#e#(er(
;a. Read the passage carefully
;b. Bnderline the error
;c. >oo' for errors in the use of
/repositions
Tenses
<onnectors
Articles
9odals
4ender
Non$finites
Sub=ect$verb agreement
/ronoun
Number ;singular$plural.
NominaliEation K noun
form
1ra*"i*e E!er*ise
A) EDITING ;\ x % ] + mar's.
T,e 2o&&owi+4 %assa4e ,as +o" (ee+ edi"ed. T,ere is o+e error i+ ea*, o2 ",e &i+es.
T,e i+*orre*" &i+e ,as (ee+ 0+der&i+ed. Re%&a*e ",e 0+der&i+ed word *,oosi+4 ",e
#os" a%%ro%ria"e o%"io+ 2ro# ",ose 4ive+ (e&ow.
So&ved E!er*ise
6 4andhi=i was a prolific writer who ;a. spend several hours ;b. in day writing. 2e ;c.
writes for the =ournals, he edited. 2e wrote letters, speeches ;d. with reports. 2e used the
print medium ;e. for spread his message. 2e personal ;f. letter were even more powerful.
2e used ;g. a powerful medium to reinforce his views ;h. for life.
;a. ;i. spending ;ii. spent ;iii. has spent ;iv. will spent
;b. ;i. an ;ii. the ;iii. a ;iv. one
;c. ;i. wrote ;ii. write ;iii. written ;iv. had written
;d. ;i. or ;ii. along ;iii. and ;iv. many
;e. ;i. of ;ii. on ;iii. in ;iv. to
;f. ;i. lettered ;ii. letters ;iii. will letter ;iv. have letters
;g. ;i. that ;ii. these ;iii. this ;iv. those
;h. ;i. on ;ii. about ;iii. in ;iv. over
A+swer >e:(
;a. ;ii. Spent
;b. ;iii. a
;c. ;i. wrote
;d. ;iii. and
;e. ;iv. to
;f. ;ii. letters
;g. ;iii. this
;h. ;i. on
66 The 54olden Age7 ;a. have opened new values ;b. among intellectual property. Give
years ago, the phase ;c. isn1t even in the vocabulary ;d. with many companies. The
'nowledge of economy has ;e. gave rise to a new ecology of competition, in ;f. whose
intellectual assets are ;g. a principal wellsprings of a shareholder1s wealth ;h. for
competitive advantage.
;a. ;i. would have ;ii. had ;iii. will have ;iv. has
;b. ;i. in ;ii. for ;iii. on ;iv. of
;c. ;i. am not ;ii. wasn1t ;iii. aren1t ;iv. won1t
;d. ;i. for ;ii. on ;iii. of ;iv. in
;e. ;i. give ;ii. given ;iii. gives ;iv. will give
;f. ;i. where ;ii. what ;iii. which ;iv. when
;g. ;i. an ;ii. the ;iii. them ;iv. their
;h. ;i. and ;ii. or ;iii. to ;iv. of
666. ,e don1t eat food only to ;a. tic'led our taste buds, ;b. and also to get nutrients
;c. for run our body, ma'ing it ;d. grew and repair the inevitable wear ;e. but tear. The
food that we eat ;f. contained these nutrients. 3ut these ;g. has to be extracted ;h. by the
digestive system by a complete biochemical process.
a. ;i. tic'les ;ii. tic'ling ;iii. tic'le ;iv. will tic'le
;b. ;i. but ;ii. to ;iii. or ;iv. among
;c. ;i. of ;ii. and ;iii. from ;iv. to
;d. ;i. grown ;ii. will grow ;iii. grow ;iv.have grown
;e. ;i. and ;ii. to ;iii. but ;iv. as
;f. ;i. contains ;ii. will contain;iii. contain ;iv.containing
;g. ;i. had ;ii. will have ;iii. would have ;iv. have
;h. ;i. of ;ii. from ;iii. for ;iv. towards
6I. -ur team played a hoc'ey match ;a. to the D.4. Team. 6t ;b. is played in our
ground. The match started when the refree ;c. blow the whistle. Grom the very beginning
it was ;d. most interesting than any other ;e. games 6 have ever witnessed. 6n the first half
the D.4. Team ;f. scores a goal. 6n ;g. a second half, our team scored two goals and won
;h. a match.
a. ;i. for ;ii. of ;iii. with ;iv. in
;b. ;i. was ;ii. were ;iii. will be ;iv. can be
;c. ;i. blown ;ii. blew ;iii. will blow ;iv.has blown
;d. ;i. the most ;ii. the more ;iii. much ;iv.more
;e. ;i. game ;ii.gamed ;iii. gaming ;iv. has game
;f. ;i. score ;ii. will score ;iii. scored ;iv.have score
;g. ;i. an ;ii. the ;iii. other ;iv. another
;h. ;i. an ;ii. second ;iii. during ;iv. the
-96SS6-N
1oi+"s "o Re#e#(er(
;a. Read the passage carefully and identify the tense in which the passage
has been written.
;b. Bse a pencil to identify the missing word.
;c. >oo' for the following missing words(
0eterminers
Nouns
Ierbs
Adverbs, Ad=ectives
/ronouns
<onnectors
/repositions
Sub=ect$Ierb agreement
Articles
9odals
6n the passage given below one word has been omitted in each line. The omitted word has
been indicated by a . Replace with the most appropriate option from those given below.
;+9ar's.
;a. -ne thing we all must doto cooperate
;b. ,ith police and pay heed to their advice.
;c. They warn us nottouch unidentified,
;d. unclaimed suspicious ob=ects li'e transistors,
;e. brief cases etc. ,e need watch out for
;f. abandoned cars, scooters and report
;g. detailssuch ob=ects to the nearest police
;h. control room dialing &##. -ne should
not touch the ob=ects till the bomb disposal squad arrives
;a. i. was ii. is iii. has iv. have
;b. i. a ii. an iii. the iv. some
;c. i. to ii. in iii. from iv. by
;d. i. but ii. li'e iii. and iv. non of the above
;e. i. for ii. to iii. from iv. at
;f. i. a ii. an iii. the iv. all
;g. i. at ii. of iii. from iv. some
;h. i. by ii. in iii. at iv. from
Fi&& i+ ",e (&a+ks
6 Gill in the blan's choosing the most appropriate option from those given in the brac'ets
;+ 9ar's.
;6. Solved :xercise(
9any hunters ;a. ^^^^^^^^^^^^^^ ;am, in, of , over. Africa say that ;b.^^^^^^^^
;the, some, one, many. leopard is the most dangerous of all the cats, ;c. ^^^^^^^^^^
;most, much, more, many. dangerous than even the lion. -ne reason is ;d. ^^^^^^^^^^
;the, these, those, that. the leopard doesn1t ;e. ^^^^^^^^^^ ;seem, li'e, li'ely, seemed. to
be particularly afraid of men. A leopard ;f. ^^^^^^^^^ ;li'e, li'es, li'ed, li'ely. to eat
baboons but if a man comes by ;g. ^^^^^^^^^ ;if, as, when, where. the leopard is hungry,
then the leopard is as li'ely to attac' him ;h. ^^^^^^^ ;as, when, whose, which. he would
a mon'ey.
A+swer >e:( ;a. in, ;b. the, ;c. more, ;d. that ;e. seem ;f. li'es ;g. when ;h. as
;66. Sachin Tendul'ar, one of 6ndia1s ;a. ^^^^^^^ ;good, better, best, the best. 'nown
cric'eter, ;b. ^^^^^ ;am, are, is, was. leaving for the B.S.A. ;c. ^^^^^^^^^ ;on, in at, for.
Saturday night for an operation of ;d. ^^^^^ ;their, theirs, his, him. left hand ring finger.
Tendul'ar has to undergo ;e. ^^^^^ ;an, the, a, or. operation at a hospital in 3altimore ;f.
^^^^^ ;for, and, or, if. would be away for six wee's.
;666. The use of water as ;a. ^^^^^^^^;a, the, an, or. agent in the treatment of disease
is 'nown ;b. ^^^^^^^^ ;as, to, according, by. hydrotherapy. The beneficial effect of
water ;c. ^^^^^ ;could, would, can, will. be used in the treatment of many diseases.
,ater is ;d. ^^^^^^^ ; one, many, other, some. of the heavenYs choicest blessings. ,ater
acts on the body in various ways according ;e. ^^^^^^ ;on, in, to from. its temperature. 6t
helps to remove toxins ;f. ^^^^^ ;for, from, away, in. the body. 6t also helps to bring ;g.
^^^^^ ;up, over, above, down. the temperature in case of fever. The :gyptians used
heating$ cooling bath ;h. ^^^^^^^^ ;of, for, to, over. various treatments.
;6I. The successors of GeroE Shah Tuglaq were wea' and incapable ;a. ^^^^^^^^ ;for,
over, from, of. resurrecting the 0elhi Sultanat. The wealth of 6ndia was ;b. ^^^^^^^ ;in,
over, for, above. a state of confusion and anarchy prevailed here. ;c. ^^^^^^ ;these, this,
that, those. encouraged Timur ;d. ^^^^^^ ;for, to, towards, in. invade 6ndia. After
<apturing several places;e. ^^^^^^^^ ;across, over, on, above. his way and massacring
thousands of ;f. ^^^^^^^^^^^ ;mob, people, mass, civiliEation . he captured >oni on the
ban's of river Oamuna. Sultan 9ahmud ;g. ^^^^^^ ;from, of, in, over. 0elhi tried to
oppose Timur ;h. ^^^^^ ;beside, with, besides, along. a huge army of &#,### cavalry,
+#,### infantry and &)# elephants.
;I. 6n the state of Lerala, it ;a. ^^^^^^^^ ;is, am, was, were. a practice ;b. ^^^^^^^
;of, for, to, in. use Dac' fruit leaves for ma'ing ;c. ^^^^^^^^ ;a, an, it, the. improvised
spoon. The =ac'fruit leaf was folded ;d. ^^^^^^^ ;to, and, instead, or. sealed at one end
;e. ^^^^^^^ ;with, without, across, over. a small stic' ta'en from the coconut leaf. This
was used ;f. ^^^^^^^ ;for, as if, because. a spoon for drin'ing the gruel. ;g. ^^^^^^^
;for, of, in, about. 6ndia, especially in the South, the banana leaves are also ;h. ^^^^^^^^
;common, commonly, most, mostly. used instead of plates on auspicious occasions.
;I6. -nce there was a family ;a. ^^^^^^^ ;that, who, this, whose. lost all
its money. They had ;b. ^^^^^^^^^ ;been, be, to, for. sell their big house and all their
fields. 3ut the parents ;c. ^^^^^^^ ;do, does, did, done. not forget ;d. ^^^^^^^ ;this, that,
these, those. they had once been rich, and they did not let ;e. ^^^^^^^ ;her, his, their,
theirs. daughter forget either. -ut ;f. ^^^^^^^ ;from, of, in, for. all their vast wealth, they
managed to 'eep a slender silver spoon. 5,e ;g. ^^^^^^^ ;can, could, will, would.
rebuild our family fortune from it,7 her father used ;h. ^^^^^ ;to, from, in, as . say.
$C) CLOJE9GA1 FILLING $o+e word) $M ! A N / #arks)
;a. Read the paragraph before attempting to fill in the blan's.
;b. Gorm a general idea about the paragraph.
;c. 4enerally the blan's are filled up with an article, preposition, modals, verb,
noun, relative pronoun, ad=ectives, con=unction, auxiliary, pronouns,
determinations etc.
;d. An idea about the tense of the paragraph also helps in filling the blan's.
;e. After filling the blan's, chec' the sub=ect$verb concord.
;6. -ne hot day in 0ecember 6 ;a. ^^^^^^ been standing perfectly still for a few
minutes ;b. ^^^^^^^ dry weeds, ;c. ^^^^^^^^ a slight rustling sound ;d. ^^^^^^ from
near my feet. 4lancing down 6 saw the head ;e. ^^^^^^^the nec' of a large serpent ;f.
^^^^ slowly passed by me. 6n a moment, the flat head was lost ;g. ^^^^^^^ sight among
the weeds ;h. ^^^^^^^ the long body continued to be seen moving slowly by.
a. ;i.had ;ii. has ;iii. have ;iv. will have
;b. ;i. beside ;ii. among ;iii. between ;iv. middle
;c. ;i. where ;ii. which ;iii. what ;iv. when
;d. ;i. come ;ii. came ;iii. will come ;iv. have come
;e. ;i. and ;ii.or ;iii. as ;iv. along with
;f. ;i. this ;ii. that ;iii. those ;iv. these
;g. ;i. of ;ii. for ;iii. from ;iv. but
;h. ;i. as ;ii. because ;iii. so ;iv. but
;66. -n )# 9ay, &%&% the 3ritish explorer Sir Stamfor Raffles, reported the finding of
a giant flower ;a. ^^^^^ Sumatra. 6t was named Rafflesia ; b. ^^^^^^ him. 9easuring
almost a meter across and ;c. ^^^^^^^^^ up to && 'gs, the Rafflesia is ;d. ^^^^^^^
world1s largest flowers. 6t is ;e. ^^^^^^^^^ called 5Stin'ing >ily7 ;f. ^^^^^^ of the
unpleasant smell it emits. ;g. ^^^^^^^^ people, who have been fortunate to see it, agree
that they are ;h. ^^^^^^^ by its beauty.
a. ;i.of ;ii. in ;iii. over ;iv. inside
;b. ;i. for ;ii. after ;iii. by ;iv. above
;c. ;i. weigh ;ii. weighing ;iii. weighed ;iv. tot
;d. ;i. the ;ii. a ;iii. one ;iv. an
;e. ;i. almost ;ii.also ;iii. so ;iv. sometime
;f. ;i. for ;ii. as ;iii. because ;iv. due
;g. ;i. many ;ii. this ;iii. that ;iv. those
;h. ;i. overwhelming ;ii. over$whelmed ;iii. whelmed ;iv. overwhelm
666. The second ;a. ^^^^^^ mountain in the world after 9t. :verest is L$), also ;b.
^^^^^ as 9t. 4odwin Austen. 6t is %!&! mts ;c. ^^^^^^^^ and has been named ;d. ^^^^^
2enry 4odwyn Austen an :nglishman ;e. ^^^^^^^ surveyed it in the late &F"#s. L$) is
generally considered ;f. ^^^^^^^ difficult to climb than the :verest which was first
scaled in &F"*. The first to ;g. ^^^^^^ the top of L$) was an 6talian expedition ;h. ^^^^^
by Ardito 0eslo in &F"+.
a. ;i.high ;ii. highest ;iii. higher ;iv. the highest
;b. ;i. well 'nown;ii. 'now ;iii. 'new ;iv. 'nown
;c. ;i. high ;ii. higher ;iii. the highest ;iv. highest
;d. ;i. over ;ii. after ;iii. by ;iv. above
;e. ;i. whom ;ii.which ;iii. whose ;iv. who
;f. ;i. more ;ii. much ;iii. most ;iv. the most
;g. ;i. reached ;ii. reach ;iii. reaching ;iv. well reach
;h. ;i. lead ;ii. leading ;iii. led ;iv. heading
6I. The ancient Ii'ings ;a. ^^^^^^^^ many ravens with them when they ;b. ^^^^^^^ on
a =ourney over the high seas. After a few days of leaving the shores, they ;c. ^^^^^^^^
release a bird once every few days. 6n the beginning of the voyage the ravens ;d. ^^^^^^^ fly
bac' to the shore from where they ;e. ^^^^^^^^ started. 3ut after some time, the ravens
would fly ;f. ^^^^^^^^^the direction of a new land because these birds ;g. ^^^^^^ sense
direction and lands. The Ii'ing would then ;h. ^^^^^^ their ships in the direction the birds
had ta'en.
a. ;i.carry ;ii.carried ;iii. carrying ;iv. bring
;b. ;i. went ;ii. go ;iii. gone ;iv. start
;c. ;i. will ;ii. do ;iii. did ;iv. would
;d. ;i. will ;ii. have ;iii. can ;iv. could
;e. ;i. has ;ii.have ;iii. had ;iv. did
;f. ;i. in ;ii. to ;iii. for ;iv. of
;g. ;i. would ;ii. could ;iii. will ;iv. can
;h. ;i. steered ;ii. steer ;iii. steering ;iv. ta'e
I. A man saw a thief brea'ing into a house. 6nstead ;a. ^^^^^^ raising a cry, he decided
to catch ;b. ^^^^^^^ thief red$handed. 2e entered the house ;c. ^^^^^^ the window and
moved stealthily ;d. ^^^^^^^^ the door. The room was pitch dar' and he could not hear
anything. ;e. ^^^^^^^ he approached the door, it then suddenly ;f. ^^^^^^ open from the
other side. The other man as'ed, 5,hat ;g. ^^^^^ you doing in ;h. ^^^^^^ houseM7
a. ;i.for ;ii.of ;iii. in ;iv. on
;b. ;i. a ;ii. an ;iii. the ;iv. are
;c. ;i. from ;ii. for ;iii. in ;iv. into
;d. ;i. toward ;ii. towards ;iii. to ;iv. in
;e. ;i. so ;ii.since ;iii. because ;iv. as
;f. ;i. flew ;ii.flow ;iii. flown ;iv. flowing
;g. ;i. is ;ii. am ;iii. are ;iv. were
;h. ;i. our ;ii. my ;iii. his ;iv. mine
I6. A personality is a person in totality. ;a. ^^^^^ individuals who ma'e ;b. ^^^^ mar' in
the world and leave footprints on the sands of ;c. ^^^^^ are 5personalities7. ;d. ^^^^^^
change the world with their thoughts and actions ;e. ^^^^^^^ become memorable. -ther
individuals follow ;f. ^^^^^^^ and try to develop themselves into such personalities. 6ndia is
a land of eminent personalities whose contributions is ;g. ^^^^^^^^^ fields are matchless. 6t
is important that we learn ;h. ^^^^^^^^ from their lives and ma'es our lives worth.
a. ;i.the ;ii.a ;iii. an ;iv. are
;b. ;i. total ;ii. a ;iii. the ;iv. an
;c. ;i. times ;ii. timed ;iii. timeless ;iv. time
;d. ;i. them ;ii. their ;iii. they ;iv. there
;e. ;i. which ;ii.when ;iii. who ;iv. whose
;f. ;i. their ;ii. them ;iii. there ;iv. themselves
;g. ;i. a ;ii. an ;iii. the ;iv. various
;h. ;i. lessons ;ii. lesson ;iii. thing ;iv. things

D) SENTENCE RE9ORDERING
1OINTS TO REMEMBER
Always loo' for the sub=ect first and then the predicate part which contains the verb.
Also loo' for the ob=ect, the helping verbs, ad=ectives etc. and try to place them
meaningfully.
No"e : $a) Dumbled words in a re$ordering exercise to test syntax will involve sentence in
a context.
$() :ach sentence is split into sense groups ;not necessarily into single words. and
then =umbled up.
Rearra+4e ",e 2o&&owi+4 words a+d %,rases "o 2or# #ea+i+420& se+"e+*e. T,e 2irs" o+e
,as (ee+ do+e as a+ e!a#%&e:
SOLDED EEERCISE:
$ I ) $a) in every I has become of the day the order pollution wal' of life
;b. /ollution has become the order of the day in every wal' of life
J 2as bcome the order of the day in every wal' of life
;d. -der of the day in every wal' of life has become
Answer ( ;b.
E) DIALOGE COM1LETION
$ - ! / N / Marks )
1oi+" "o Re#e#(er:
;&. >oo'ing at the ending of the preceding sentence
;). Read the preceding dialogue and the one that follows it.
;*. 6dentify the type of sentences usedH Assertive, 6nterrogative, :xclamatory or
6mperative( before choosing the options.
;+. The entire dialogue should be meaningful and relevant and should use the
appropriate tenseverb forms.
1ra*"i*e E!er*ises
$I) So&ved E!er*ise:
Read the following conversation between a brother and a sister. <omplete the conversation
by choosing the correct option from the ones given belowH
Sister ( ,hen do you to leave for 9umbaiM
3rother ( 6 am leaving the day after tomorrow
Sister ( ,here have you decided to stayM
3rother ( $a)OOOOOOOOOOOOOOOOOOOOOOOOOOOOOOOOOO
Sister ( ,hy are you not staying with Aunt and BncleM
3rother ( $()OOOOOOOOOOOOOOOOOOO.. 6 won1t be able to en=oy my holidays.
Sister ( 0on1t tell me that you1ll stay in a hotel8
3rother ( 0efinitely not $*)OOOOOOOOOOOOOOOOOOOOOOOOO
Sister ( 2ow are you both spending your vacationM
3rother ( $d)OOOOOOOOOOOOOOOOOOOOOOOOOOOOOOOOOOOO
; a. ;i. 6 won1t decided as yet ;ii. 6 won1t decide as yet
;ii. 6 hadn1t decide as yet ;iv. 6 haven1t decided as yet
;b. ;i. They were too strict ;ii. They are too strict
;iii. They will be too strict ;iv. They are too funny
;c. ;i. 6 plan to stay with aunty ;ii. 6 plan to stay in a hotel
R uncle
;iii. 6 plan to stay with Rahul ;iv. 6 won1t stay with aunt R Bncle
;d. ;i. ,e1ll go nowhere ;ii. ,e1ll go sight seeing
;iii. ,e1ll stay nearby ;iv. ,e1ll go somewhere
A+swer >e:(

;a. iv 6 haven1t decided as yet
;b. ii They are too strict
;c. iii$ 6 plan to stay with Rahul
;d. ii K ,e1ll go sight seeing
$III) 0octor( 2ow long have you been sic'M
/atient ( 6 have been having this headache $a)OOOOOOOOOOOOOOO one
month.
0octor ( 2ave you, ta'en any medicineM
/atient( No 6 haven1t. 3ut 6 $()OOOOOOOOOOOOOOOOO by an eye K
specialist.
0octor ( ,hat did he sayM
/atient( 2e said that my eyes are fine
0octor( $*)OOOOOOOOOOOOOOOOOOOOOO sleeping late at night M
/atient ( No, 6 sleet regularly around &# p.m.
0octor ( 2ave you $d)OOOOOOOOOOOOOOOOOOOOOOOOOO in the post or
latelyM
/atient ( No but 6 had in used my head due to a fall when 6 was ten years
old
;a. ;i. for the past ;ii. since the past
;ii. during the past ;iv. for a past
;b. ;i. got my eye test ;ii. will get my eyes
;iii. got my eyes tested ;iv. would get my eye
;c. ;i. has you been ;ii. had you been
;iii. had you be ;iv. have you been
;d. ;i. met with are accident ;ii. meets with an accidents
;iii. meet with an accident ;iv. met with the accidents

$ID) 6nterviewer ( ,hy do you ant to be a teacherM
Smita 6 li'e the profession and wish to share my 'nowledge with
children.
6nterviewer ( $a)OOOOOOOOOOOOO in serving in a co K educational
schoolM
Smita ( No sir, 6 don1t have any problem.
6nterviewer ( ,hat $()OOOOOOOOOOOOOOOOOOOOO can teachM
Smita ( 6 can teach :nglish and Social Studies.
6nterviewer ( ,e ;*)OOOOOOOOOOOOOOOOOOOOOOOOO..
Smita ( 6t would be even better to teach them Sir.
6nterviewer ( All right then. Oou $d)OOOOOOOOOOOOOOOOO our school
from 9onday.
Smita ( Than' you, Sir 4ood day to you.
;a. ;i. 0o you have a problem ;ii. did you have a problem
;ii. 0on1t you have a problem ;iv. 0idn1t you have a problem
;b. ;i. are the sub=ects that you ;ii. were the sub=ects you
;ii. will be the sub=ects that you ;iv. was the sub=ects you
;c. ;i. give you =unior classes ;ii. may give you =unior classes
;iii. would give you =unior classes ;iv. are give you =unior classes
;d. ;i. will =oined ;ii. may =oin
;iii. shall =oined ;iv. can =oin
$D) Ii'ram ( 0id you watch the show 56ndian 6dol7 last nightM
Ra=ani ( No, 6 had gone shopping with my mother
,as $a)OOOOOOOOOOOOOOOOOOOOOOOOOO M
Ii'ram ( Oes, it was quite interesting. ,e had two guest singers
on
-n the show
Ra=ani ( Really, who $()OOOOOOOOOOOOOOOOOOOOOOOOO M
Ii'ram( They were Lailash Lher and Bdit Narayan.
Ra=ani ( 2ow sad 6 missed itM 6 wish $*)OOOOOOOOOOOOOOOOOO.
Ii'ram ( 0o not worry. 61ve taped it for you. Oou ;d)OOOOOOOOOO
anytime you want.
Ra=ani ( Than's a lot. 6 sure will as soon as 6 finish my wor'.
;a. ;i. its very interesting ;ii. it very interested
;iii. it very interesting ;iv. its very interested
;b. ;i. are they ;ii. was they
;iii. are them ;iv. were they
;c. ;i. 6 have saw it ;ii. 6 had seen it
;iii. 6 had saw it ;iv. 6 have seen it
;d. ;i. could watch ;ii. will watch it
;iii. can watch it ;iv. would watch it
F) SENTENCE TRANSFORMATION INCLDES
i) Re%or"ed S%ee*,
$- ! / N / #arks)
1oi+"s "o Re#e#(er :
;a. The given dialoguedirect speech is to be changed into indirect speech.
;b. Leep in mind the reporting verbH the change of pronounsH the removal of all inverted
commas.
;c. Bse the tense of the reporting verb.
;d. 6dentify the type of sentence being changed$Assertive, 6nterrogative, :xclamatory
and 6mperative.
;e. Gor a close ended question ;answer is either Oes of No. use 5if7 or 5whether7.
;f. Bse a ;.. full stop after completing every sentence.
;g. Read the passage again to chec' the tense and the conversion of sentence.
1RACTICE EEERCISES
Read the following CONDERSATIONS between two spea'ers. The complete the given
exercises in reported speech by choosing the correct options given below the exercises.
SOLDED EEERCISE
;6. Dudge ( ,hat are you charged withM
/risoner ( Sir , for doing my 0iwali shopping early.
Dudge ( 3ut that1s not an offense8 And how early were you
exactly doing itM
/risoner ( ,ell Sir8 To avoid the rush, 6 thought it would be better
if 6 completed it before the store opened.
The =udge being in a good wood before the 0iwali festivals as'ed the prisoner
$aOOOOOOOOOOOOOOOOO. The prisoner, sensing his good mood, politely replied
$()OOOOOOOOOOOOOOOOO. The surprised =udge declared that it wasn1t an offence.
2e further inquired $*)OOOOOOOOOOOOOOOOO. 2umbly the prisoner replied that to avoid the rush,
he had thought it would be better ;d)OOOOOOOOOOOOOOOOOOO.
;a. ;6. what he is charged with ;66. what he is charge with
;666. what he was charged with ;6I. what he was charge with
;b. ;6. doing his 0iwali Shopping early.
;66. that he was charged with doing his 0iwali shopping early
;666. that he is charged with doing his 0iwali Shopping early.
;6I. That he was doing his 0iwali shopping early.
;c . ;6. how early had he been exactly doing it.
;66. how early have he been exactly doing it.
;666. how early has he been exactly doing it.
;6I. how early was he doing it
;d. ;6. if he has completed it before ht store had opened.
;66. if he had completed it before the sore had opened.
;666. he had completed it before the store had opened.
;6I. if he had completed if before the store has opened.

A+swer >e::
;a. ;666. ,hat he was charged with.
;b. ;6I. that he was doing his 0iwali shopping early.
;c. ;6. how early had he been exactly doing it.
;d. ;66. if he had completed it before the store had opened.
$III) Gather (4ood 9orning, Sir,. 6 have come to get my child admitted in
your school.
/rincipal ( 6n which class do you wish to get your child admittedM
Gather ( 6n class 66, SirM
/rincipal ( Sorry, we do not have any vacancy in that class..
Gather ( Sir, my son is very intelligent. Oou will be proud to have him
in your school.
/rincipal ( ,hat has he learnt till nowM
Gather ( To wal' and to tal', Sir.
The father of a child met the /rincipal of a school and told him that $a)OOOOOOOOOOOO. The
principal enquired about the class for admission and the father told him that it was class 66.
The principal then informed him that $()OOOOOOOOOOOOOOOOOOOOOO in that class. The father
told him that his son was very intelligent and them ;the principal school.
$*)OOOOOOOOOOOOOO. .The principal, further, as'ed him $d)OOOOOOOOOOOOOOOOOOOOO. The
father proudly replied that he had learnt to wal' and to tal'.
;a. ;6. he has come to get his child admit in their school.
;66. he have came to get his child admitted in the school.
;666. he had came to get his child admitted in his school.
;6I. he had come to get his child admitted in his school.
;b. ;6. they do not have any vacancy in that class.
;66. they do not had any vacancy in that class.
;666. they did not have any vacancy in that class.
;6I. they did not had any vacancy in that class.
;<. ;6. would be proud to has him in the school.
;66. would be prould to have him in his school.
;666. will be proud to had him in his school.
;6I. will be proud to have him in his school.
;d. ;6. what he had learnt till now.
;66. what had he learnt till then.
;666. what he has learnt till now.
;6I. what has he learnt till then.

;I. 9other ( Sonal, what are you drawingM
Sonal ( 6 am nit drawing. 6 am writing to my friend.
9other ( 3ut you can1t write8
Sonal ( 6t doesn1t matter. 9y friend can1t read either.
The mother as'ed sonal $a)OOOOOO. Sonal replied that she wasn1t drawing but $()OOOO.
The mother remar'ed with surprises that $*)OOOOOO. Sonal <almly replied that $d)OOOOOO
read either.
;a. ;6. what she is doing.
;66. what is she doing.
;666. what was she doing.
;6I. what she was doing.
;b. ;6. was writing to my friend.
;66. is writing to my friend.
;666. is writing to my friend.
;6I. am writing to my friend.
;c. ;6. she can not wrote.
;66. she could not wrote.
;666. she can not write.
;6I. she could not write .
;d. ;6. it didn1t mater as my friend couldn1t.
;66. it didn1t matter as her friend couldn1t.
;666. it doesn1t matter as her friend couldn1t.
;6I. it doesn1t matter as my friend couldn1t.

;6. /atient ( <an 6 have an appointment with the doctor this
eveningM
Receptionist (61m sorry. 6 can1t fit you in for at least two wee's.
/atient (3ut 6 could be dead by then
Receptionist (No problem. 6f your wife informs me 6 will cancel the
appointment.
A patient went to his doctor1s clinic and requested the receptionist for an
appointment with the doctor that evening. The receptionist apologiEed saying that
$a)OOOOOOOOOOOOOOOOOOOOOOO in for at least two wee's. The disappointed
patient said that $()OOOOOOOOOOOOOOOOOOOOOOOOOOOOOOOOO by then. The
receptionist calmly replied that $*)OOOOOOOOOOOOOOOOOOOOOOO and reassured
him that $d)OOOOOOOOOOOOOOOOOOOOOOO cancel the appointment.
;a. ;i. she can1t fit you
;ii. she couldn1t fit him
;iii. she can1t fit him
;iv. she couldn1t fir you
;b. ;i. he could have been dead
;ii. he can he been dead
;iii. he may he dead
;iv. he could be dead
;c. ;i. it can1t be a problem
;ii. it won1t be a problem
;iii. it isn1t be a problem
;iv. it wasn1t be a problem
;d. ;i. if his wife informs her she will
;ii. if your wife informed her she would
;iii. if his wife informed her she would
;iv. if your wife informed her she will.
ii) 1assive For#s
1oi+"s "o Re#e#(er(
;a. Read the given instructions carefully and rewrite the process in passive voice.
;b. 3e careful about the tensetime used
;c. 6f the process given in the first examplefirst instruction is in present passive, the
rest should also be written in present passive. The same should be followed in
case of past passive also.
;d. 0o not mix the tenses while reporting the process in passive form.
;e. 6nstructions are generally written in Active voice.
1RACTICE EEERCISES:
Read the given instructions in the practice exercises. <hange these instructions into
passive by filling in the blan's. <hoose the correct options from the ones given below to
fill in the blan's(
SOLDED EEERCISE
$I) 7ow "o Make Ora+4e S=0as,
Ta'e & doE. Gully ripe oranges
Remove the rinds
:xtract the =uice
Strain through a thic' cloth
9ix \ 'g sugar, \ tea$spoon citric acid
Add a pinch of potassium meta K bisulphate, few drops of colour and essence
Stir till thoroughly dissolved
-ne doEen fully ripe oranges are ta'en. Their rinds $a)OOOOOOOOOOOOOOOOO and the =uice
$()OOOOOOOOOOOOOOOOO. Then the =uice $*)OOOOOOOOOOOOOOOOOOOOOOOOOOOO through a
thic' cloth into a stainless steel vessel. \ 'g sugar, \ tea K spoon citric acid, a pinch od
potassium meta K bisulphate, a few drops of color and essence $d)OOOOOOOOOOOOOO in
the =uice. The mixture is stirred tell it us thoroughly dissolved.
;a. ;i. were remove ;ii. are removed
;iii. was removed ;iv. is remove
;b. ;i. was extracted ;ii. are extracted
;ii. is extracted ;iv. was extract
;c. ;i. was strain ;ii. is strained
;ii. were strained ;iv. are strain
;d. ;i. are add ;ii. is added
;ii. is add ;iv. are added
A+swer >e::
;a. ;ii. are removed
;b. ;iii. is extracted
;c. ;ii. is strained
;d. ;ii. is added
NSOLDED EEERCISES
$II) 7ow "o #ake Ca+d: Co+e Or+a#e+"
0raw a half circle on red construction paper ma'ing it && inches in diameter
<ut out and roll the circle into a cone and a tape
<ut a piece of coloured ribbon for a hanger
,ith a pencil tip, punch a hole in opposite sides of the cone
/ull the ribbon through the holes
<andy cone ornament is ready. 0ecorate the cone with design cut from gift
wrapping paper.
A half circle is drawn on a red construction paper ma'ing it && inches in diameter.
The circle $a)OOOOOOOOOOOOOOOOOO out into a cone and a tape. A piece of colored
ribbon $()OOOOOOOOOOOOOOOOOOOOO for a hanger. Next, with a pencil tip a hole
$*)OOOOOOOOOOOOOOOOOOOOO in opposite sides of the cone. Then the ribbon is pulled
through the holes. <andy cone ornament is ready. The cone $d)OOOOOOOOOOOOOOOO
with design cut from gift wrapping paper.
;a. ;i. is cut and roll ;ii. are cut and rolled
;iii. is cut and rolled ;iv. are cut and roll
;b. ;i. was cut ;ii. are cut
;iii. were cut ;iv. is cut
;c. ;i. is punched ;ii. is punch
;iii. are punched ;iv. are punch
;d. ;i. are decorate ;ii. is decorated
;ii. is decorate ;iv. are decorated
$III) 7ow "o #ake 1o%*or+
/lace a pan on a gas stove
9elt a small amount of butter
Add salt, pinch to turmeric
Add corn to above ingredients
<over the pan with a lid
Toss the corn tell 5popping1 sound reduces
Serve in large bowl while still warm
At first a pan is placed on a gas stove. A small amount of butter $a)OOOOOOOOOOOOOO
in it. Then salt and a pinch of butter $()OOOOOOOOOOOOOOOOOOOOO in it. 6mmediately
raw corn $*)OOOOOOOOOOOOOOOOOOOOOOOOO to the above ingredients. The pan is
covered with a lid. The corn in the pan $d)OOOOOOOOOOOOOOOOOOOOOOOOOO till the
popping sound reduces. The popcorn is served in a large bowl while still warm.
;a. ;i. are melt ;ii. is melt
;iii. are melted ;iv. is melted
;b. ;i. are added ;ii. were added
;iii. is added ;iv. are added
;c. ;i. are added ;ii. is added
;iii. is add ;iv. was added
;d. ;i. are tossed ;ii. is toss
;iii. is tossed ;iv. are toss

iii) NEWS1A1ER 7EADLINES
1OINTS TO REMEMBER
2eadlines must he written in the /assive simple present form.
Should be brief and self explanatory.
Relevant new information can be added but do not add irrelevant information.
,hile expanding the headlines, they should be properly connected with the whole
sentences.
Look a" ",e +ews%a%er i"e#s (e&ow. T,e+ 0se ",e i+2or#a"io+ i+ ",e ,ead&i+es "o
*o#%&e"e ",e re%or". Co#%&e"e ",e ,ead&i+es (: *,oosi+4 ",e *orre*" a+swer 2ro# ",e
o%"io+s (e&ow:
SOLDED EEERCISE
$A) De+40e *&ai#s seaso+Cs 2irs" vi*"i#
A &# year old boy ^^^^^^^^^^^^ late on ,ednesday night at the 2oly Gamily
2ospital
$a) 3ecome 0engue1s first victim
$() ,ill become 0engue1s first victim
$*) 3ecame 0engue1s first victim
$d) 3ecomes 0engue1s first victim
$B) Gov". "o 2ra#e %o&i*: 2or ",e Disa(&ed soo+
The centre ^^^^^^^^^^^^^^^^^^^ for persons with disability.
$a) ,ill decide to frame a policy
$() 2ad decided to frame a policy
$*) 2ave decided to frame a policy
$d) 2as decided to frame a policy
$C) Te*,+i*a& Fa0&" ",is Me"ro servi*es
9etro services on the busy. Tis$2aEari$ Sahadra section ^^^^^^^^^^^^^^^ on
,ednesday morning due to an electric failure causing inconvenience to daily
commuters
$a) Are hit
$() 6s hit
$*) ,ere hit
$d) ,as hit
$D) Wa"er 1ark *o#i+4 0% +ear 1ra4a"i Maida+
The 00A on Saturday announced that ^^^^^^^^^^^^^^^^^ near the /ragati 9aidan
$a) A water par' is being built
$() A water par' are being built
$*) A water par' was being built
$d) A water par' were being built
A+swer >e:
$a) (iii) become 0engue1s first victim
$() ;iv.has decided to frame a policy
$*) $iii) were hit
$d) ;i. a water par' is being built
+so&ved E!er*ises

$ID)
$A) Lo*a& Ar"is" E!,i(i"s i+ a040ra"ed
An exhibit of the local artists ^^^^^^^^^^^^^^^^ by the >t. 4overner today in the 0elhi
2aat.
$a) ,ere inaugurated
$() ,ill be inaugurated
$*) 6s inaugurated
$d) ,as inaugurated
$B) 2/ Mi+ers die i+ #i+e9(&as"
According to Reuters )+ miners ^^^^^^^^^^^^^^^ at a blast in a coal mine in eastern
B'raine
$a) 2ave died
$() 2as died
$*) 2ad died
$d) ,ill have died
$C ) Ladak, Fes"iva& e+ds
The six$day >ada'h festival ^^^^^^^^^^^^^^^^ with the rendition of the traditional +a'
dance by artists here
$a) 2as ended
$() 2ad ended
$*) 2ave ended
$d) 6s ended
$D) Rai+ a+d %ower %&a: ,avo* i+ *i":
,ee'end revelers and shoppers ^^^^^^^^^^^^^^^^ unaware with a sudden shower on
Sunday evening. The rain lashed most parts of the city, leading to massive traffic =ams.
$a) ,as caught
$() ,ere caught
$*) ,ill caught
$d) 2ad been caught
$D) $A) -/00 De,i*&es *,a&&a+ed 2or "ra22i* vio&a"io+@ drivi+4 &i*e+ses *a+*e&&ed
According to 0</ ;traffic. nearly &+## vehicles ^^^^^^^^^^^^^^^^ for violating traffi
rules during the first four days of the enforcement drive.
$a) 2ave been challaned
$() ,ere challaned
$*) ,as challaned
$d) Are challaned
$B) B&a*k 7o&e s%o""ed +ear Ear",
According to some Australian scientists in <anberra, a blac' hole ^^^^^^^^^^^^^^^^ in
space close to the earth
$a) 2as been spotted
$() 2ave been spotted
$*) 6s been spotted
$d) ,ere been spotted
$C) M:sore Joo we&*o#es +ew 40es"
A female giraffe calf ^^^^^^^^^^^^^^^^^^^^ in 9ysore [oo on Saturday to five$year
old Lhushi and her male companion Lrishnara=a.
$a) 6s born
$() ,as born
$*) 2ad born
$d) ,ill be born
$D) B0s *r0s,es a 4ir& "o dea",
A three$ year$ old girl ^^^^^^^^^^^^to death when a bus ran over her in patpargan=.
$a) ,as crushed
$() 6s crushed
$*) 2as been crushed
$d) 2ad been crushed
$DI) $A) 5 &ak, ?o(s "o (e *rea"ed (: 20-0LLL.. 1&a++i+4 Co##issio+
9r. A.R. Sahni, member of the /lanning <ommission said yesterday that
^^^^^^^^^^^^^^^^ by the year )#&#
$a) " la'h =obs will be create
$() " la'h =obs will be creates
$*) " la'hs =obs will be create
$d) " la'h =obs will be created
$B. >,e& Ra"+a 2or Sa+ia re*o##e+ded
Sania Nehwal1s growing stature in sports arena had led her name ^^^^^^^^^^^^^^^^^
for the Lhel Ratna Awarded
$i) 3eing recommended $ii) been recommending
$iii) 3een recommended $iv) have been
recommended
$C ) >ids 2a&& i&& a2"er ea"i+4 #id9da: #ea&
#F 'ids of a 9<0 school in South$0elhi ^^^^^^^^^^^^^^^^^^^ after having mid$day meal
on Griday
$i) fell ill $ii) fallen ill
;iii) fall ill $iv) have fell ill
$D) More swi+e F&0 *ases o+ ",e rise
Nearly 66 new swine flu cases ^^^^^^^^^^^^^^ from the capital on Griday, bringing the
total number of positive cases to @+ in the last two months
$a) ,as reported
$() ,ere reported
$*) 2as been reported
(d)$ad een re-orted
SECTION D $LITERATRE)
FICTION
T2: TR63BT:
0AS2 3:N2BR
INTRODCTION
The Tribute is a simple and emotional story which revolves around a young man
called 3abuli. 6t highlights the wide range of approach, attitude and behavior of human
beings towards each other. The positive qualities of a person such as generosity, nobility and
tenderness can elevate a man to great heights where as meanness, narrow$mindedness and
selfishness ma'es him stoop very low. 3abuli, the main character, is the narrator himself7.
GIST OF T7E LESSION
1ARA -9.
3abuli reached his office and unexpectedly received a letter from his elder brother. 2e felt
uncomfortable and guilty as he had not written to him for a long time. 2e recollects that
during his student days he was very regular in visiting his home. 2owever after marriage he
became so, busy and pre occupied with city life that he almost forgot his mother and brother
at village. Gor two years he had not visited his village. 9other often complained to him about
it in letters and even reminded him of pre$marriage time. 2e realiEed that he had been =ust
avoiding and trying to show that he was very busy with his =ob and family. :ven mother has
also understood his position and busy schedule. She stopped complaining.
1ARA /9F
3abuli1s elder brother normally doesn1t write to him. 2e has selfless love towards him. 2e
has always been very much concerned about his well being right from his childhood. 2e
remembers his student1s days as he would ma'e it a point to get his favourite dish ;fish.
prepared exclusively for him. 2e did not change with time, not even with his family, and
household responsibility. 2is elder brother had as'ed him to come home as a bitter quarrel
had cropped up between elder brother1s wife and middle brother1s wife. The outcome was
division of property among three brothers. 2is presence was unavoidable. The second brother
was very adamant about the divisionH he wanted it at any cost.
1ARA F9H
3abuli felt helpless and orphaned as he realiEed his carelessness in neglecting his family at
village. 6n the evening he informed his wife about the partition. She was totally unaffected as
she had no emotions and love for her husband1s family. All of sudden, she became concerned
and behaved as if she were all prepared for it. She came closer to him and planned for the
future. She reminded him to give her the entire amount so that she could spend it =udiciously.
She proposed to buy a fridge, scooter and deposit the money in ban'. She heartlessly
declared that there was no use 'eeping the land in the village for others.
1ARA -09-.
3abuli felt helpless and listened to his wife1s plans li'e an innocent lamb ;here it means that
he 'nows she is wrong but he doesn1t have courage to stop her.. 2e becomes nostalgic and
remembers the wonderful days of his bachelor days when the word P2ome filled his heart
with loving emotions1. The affectionate word Pbrother1 used to generate deep love in him.
3ut today he has changed so much with the passage of time, he felt wea' and helpless. 2e
could not understand how a stranger ;his wife. wal'ed into his life and made him forget his
deep love for his family. The shoc' of the letter faded in a few days and 3abuli came to his
normal self.
1ARA -/9-F
-n Saturday when 3abuli boarded the bus, he experienced the same familiar feeling, as he
used to feel while going home before marriage. 6n his hurry to get on and ta'e his favorite
seat, he dropped the /rasad of >ord >ingra= and felt very guilty for being so careless in the
past. 2e felt as if everybody was loo'ing at him sarcastically and questioning him about his
lost love and emotions for the family at village. 2e reached in the evening. 2e was received
by his elder brother as he had already informed him. 2e immediately too' his brief case
stating Pit must be heavy1. 3abuli was so nervous and perplexed that he even forgot to touch
his feet. ,hile wal'ing behind elder brother he remembered his childhood days.
1ARA -A922
,hen he used to return from tuition late in the evening, his elder brother used to escort him
bac' home unfailingly lest he should be frightened. -ften he carried 3aluli1s bag of boo's,
notes and lantern. Sometimes he carried him on his shoulders while going to the fields for a
wal'. 3abuli remembered his past and recalled his childhood days. 2e felt as if time had
coagulated for him. 2e had changed with the time but the same time could not bring any
change in elder brother1s life. 2e was so persistent in discharging his duties. 2e offered to
carry 3abuliYs suitcase.
1ARA 2.92<
-n reaching home, 3abuli felt the tension and uneasiness in the atmosphere. 2e was not
welcomed in the usual manner. 6nstead, it was all quiet and calm. -nly his mother came and
stood near him quietly. The middle brother and his wife were also not seen anywhere. The
atmosphere of the house was unusualH there was death li'e silence all around. 6f seemed as if
the house were preparing for final collapse. 3abuli1 tried to be normal with everybody but it
was very unpleasant experience for him. The middle brother and his wife displayed no
emotions, as they were all prepared for partition. 3abuli was very restless that night. Seven or
eight people gathered the next mid day to supervise the division. Three brothers were present
but mother was seen no where around. 3abuli felt as if his parents had nourished a living
body since the day of their marriage and the three brothers were now waiting to claim their
slice. Then they would part and go on their own different ways.
1ARA 2F9.0
0uring division all the household articles were heaped in the family courtyard. 6t contained
of almost everything. A long list of all the items was made ;including the little figures of the
family idols..The elder brother rose and placed his wrist watch on the heap with other things.
2e became emotional and left the place. 6t symboliEes his brea'ing all tries with the past.
3abuli recalled that the watch had been given to him when he was in eleventh class. The
elder brother had mortgaged that same watch to send 3abuli to 0elhi for an interview.
3abuli was silent during the partition. 2is second brother often whispered things into
his wife1s ear whereas the elder brother was calm and composed li'e perfect gentleman
loo'ing all the proceedings dispassionately as he had done in the past. :ven while discussing
anything with second brother, he had that same attitude$ no sign of disgust and regret.
1ARA .-9..
2e remembered that how elder brother was always selfless. The year their father died
they were living under great financial strain and did not have sufficient money to buy
blan'ets to save from the biting cold. -ne night the elder brother left for fields without a
blan'et on his shoulders covering 3abuli with his own blan'et so that he could sleep
comfortably. 3abuli felt guilty for his selfish attitude as now he had a comfortable income,
but he never cased to buy any warm cloth for his elder brother. 2e was still satisfied and
happy with that old torn blan'et that he had covered him once. That same blan'et was before
his eyes. The process of division was finally over. The elder brother had smilingly given the
middle brother whatever he demanded. The middle brother offered to buy 3ubuli1s share for
eighteen thousand rupees.
1ARA ./9.5
6n the evening the elder brother too' 3abuli to the paddy fields which belonged to him. 2e
quietly followed him from boundary to boundary feeling the hard wor' and sincere efforts
sowed into the field. 6t was spar'ling li'e pearls on the bosom of paddy fields. 6n the
morning, before the left for 3hubaneswar, he handed over the paper on which the detail of
his share was written to his sister$in$law. -n the 3lan' side of the paper, he wrote a short
note stating that he ;elder brother. has been his land from where he could harvest everything
;all the blessings and happiness. in life. 2e needs only his love and blessings. 2e requested
him to accept it as a to'en of deep respect and tribute to him.
S64N6G6<AN<: -G T2: T6T>:
PTribute1 means something that is offered in order to show one1s admiration and respect for
someone. 6n story 3abuli, the youngest brother pays tribute to his eldest
brother. Thus PT2: TR63BT:1 is a very apt title. 6n due course 3abuli forgot
his village and family, but the letter written by his elder brother touches him
deeply and he is sha'en bac' to realisation. As a mar' of respect, he hands
over his share of property to the elder brother for all that he had done for the
family and particularly for 3abuli himself.
>6T:RARO 0:I6<:
6n the story we find that two literary devices have been used.
S696>:( ;it means one person or thing has been compared with another by means of li'e or
as..
:xample(
a. 6 listened to all this li'e an innocent lamb loo'ing into the dar'ness.
9etaphor( 6t means the comparison between two people or things is implied.
:xample(
a. A lot of cobwebs have settled around me.
b. ,e moved from boundary to boundary. :very where 6 could feel the imprints of his
feat, his palm and his fingers. -n the bosom of the paddy field spar'led the pearls of
my elder brother1s sweat.
EETRACTS FOR COM1RE7ENSION
MLTI1LE C7OICE 8ESTIONS
&. Read the following extracts and choose the correct option. :ach question carries
three mar's and each sub part carries one mar'.
A. 6 shran' within for not writing letter home all these days.
i. 2ere the spea'er is
a. 3abuli1s mother
b. 3abuli
c. 3abuli1s second brother
d. 3abuli1s elder brother
ii. The phrase P6 shran' within 1means
a. 2e felt ashamed of his conduct.
b. 2e probed into his inner felling.
c. 2e found no scope of giving expression to his feelings.
d. 2e concealed all his thoughts within himself.
iii. 2e shran' within for not writing letters home because
a. 2e felt guilty of his laEiness.
b. 2e was afraid of his family members.
c. 2e felt guilty and ashamed of neglecting his family.
d. $e .elt scared o. his #other and elder rother/
3. ,e were waiting for the final separation, as if ready to slice out the flesh of the domestic
body which our parents had nourished since the day of their marriage and then we would run
away in three different directions clutching a piece each.
i. 2e was eagerly waiting for the final separation
a. The elder brother
b. The mother
c. The second brother
d. 3abuli
ii. The household and the ancestral property has been narrated as Pdomestic body 1
a. 3ecause the division was li'e slaughtering.
b. 3ecause the parents produced, reared and nurtured the family.
c. 3ecause the division seemed same as cutting a living being into pieces.
d. 3ecause the household had the characteristics of a living being.
iii. Y,e run away in three different direction clutching a piece eachY means
a. ,e are prompted by greed.
b. >est our shares be snatched by each other.
c. To avoid 'noc'ing against each other.
d. 3ecause of great hurry.
<. 2e was showing me the fields, as a father would introduce a stranger to family members.
i. 2ere Phe1 and Pme1 refer to K
a. 3abuli to the elder brother
b. The second brother to 3abuli.
c. The elder brother to 3abuli.
d. The elder brother to the second brother.
ii. :lder brother was showing 3abuli the fields
a. So that he may not demand his share of field.
b. To satisfy 3abuli that he got his due share
c. To ma'e 3abuli familiar with his share of property.
d. To stri'e a deal with him to ta'e away his share of property.
iii.12e showed1 me the fields because of K
a. 2is habits of po'ing his nose into other affairs.
b. 2is habit of having no scope for any confusion anywhere.
c. 2is familiarity with the fields
d. 2is Pfatherly attitude towards1 3abuli and sense of responsibility.
0. A cold sweat drenched me. 6 felt helpless, orphaned.
i. 3abuli had such feeling of despair.
a. ,hen his father died.
b. ,hen he read the news of partition.
c. ,hen his wife didn1t show any concern.
d. ,hen his brother had as'ed him to come home.
ii. 2e felt Phelpless1
a. 3ecause of his inability to stop the impending partition of the family
b. 3ecause he had neglected his mother.
c. 3ecause of his wife1s cold behaviour.
d. 3ecause he had neglected his share of property.
iii. 2is reaction to the news of partition was$
a. 0isgust
b. Anger
c. 2elplessness
d. /ity
:. :verywhere cloud feel the imprints of his feet, his palm his fingers.
i. P:verywhere1 here refers to
a. the ancestral home.
b. the way from the bus stop to the village.
c. the paddy fields.
d. the household articles.
ii. The expression Pimprint of his feet1 means
a. 3abuli1s brother used to go for a wal' there.
b. 3abuli1s brother used to wor' hard there.
c. 3abuli1s brother1s hard wor' and efforts shown there.
d. The footprint of elder brother.
6ii. 3abuli1s spo'e these words when K
a. 2is brother wal'ing in front of him.
b. 2is brother had covered him with his blan'et.
c. 2is brother was showing him the land.
d. 2is brother was standing near the pile of household items.
2. S,or" A+swer 80es"io+s i+ a(o0" .09/0 words $2 #arks ea*,)
Q&. ,hy did 3abuli have the feeling of guilt and shame when he loo'ed at the letter from
villageM
Q). ,hat were the contents of the elder brother1s letterM
Q*. ,hy did 3abuli describe the city life Pmoribund1M 2ow did it draw him away from home
and village laterM
Q+. ,hy didn1t 3abuli1s elder brother usually write to him, in spite of being so loving and
caringM
Q". 2ow was 3abuli received and treated at home when he used to go there during his pre$
marriage daysM
Q!. :xplain Pa lot of cobwebs1 have settled around me.
Q@. 2ow was 3abuli received by his elder brother when he visited the village for the division
of the ancestral propertyM
Q%.16 =ust can1t understand how a stranger could all of a sudden become so intimate, only
sharing a little warmth by giving a silent promise of 'eeping close 1. ,ho is 3abuli referring
to here and whyM
QF. ,hat short of feelings did 3abuli experience when he boarded the bus to go to the
village after receiving elder brother1s letterM
Q&#. P6n the entire house, there was an air of unusualness Krather the stillness of the
graveyard 1. ,hat made 3abuli feel this when he reached homeM
Lo+4 A+swer 80es"io+s i+ a(o0" -00 ' -50 Words $ < #arks)
Q&. P6t was li'e the butcher1s 'nife going to the stone to sharpen itself 1. The story PTribute1
contains a number of references to the wor' of butcher. ,hat is the significance of these
thoughtsM
Q). ,hy doesn1t 3abuli see his elder brother before living for the city after the divisionM
Q*. 0uring the partition, the elder brother remained 1calm and composed1 showing no sign of
disgust and regret. <omment on this.
Q+. After writing a note to his elder brother 3abuli writes a letter to his wife explaining about
his decision and reasons behind giving his share to the elder brother M
05/ &lder rother did not change 1ith ti#e and gro1ing res-onsiilities ut
2auli changed/ &3-lain/
CTIE ' 1IE B3 NIC7OLAS FIS>
I+"rod0*"io+
C0"ie 1ie written by Ni*,o&as Fisk is a science fiction. <utie /ie is the name given to a
creature captured from another planet, named Quta$pi by human beings. 2e is a strange
creature, with bright rainbow coloured features, and soon becomes the centre of attraction of
the whole world. 3ut soon the craEe disappears, and as the story unfolds, we come to 'now
how he manages to get bac' to his planet.. This story, li'e other science fiction stories and
movies, reveals human interest in other planets and aliens. The emphasis is on the basic human
connection between the infant ,inters and <utie /ie, the :xtra Terrestrial creature. The
bonding between <h ,inters and <utie /ie depicts that irrespective of where we come from,
the need to reach out to others is felt by all.
<utie /ie is treated as a media sensation. <utie /ie became an instant celebrity $ he was a
commercial success sold as <utie /ie dolls, colouring boo's, tee$shirts, fan clubs, cereal
cartoons, cartoon serials etc /eople ta'e interest in <utie /ie as long as he was cute but lose
interest once he starts shedding his feathers and loo'ing ugly. The best example of
consumerism is when people completely discard this highly popular commodity in no time at
all and go after a pop star when <utie /ie loses his appeal.
Gis" o2 ",e s"or:
1ara $-9A)
The scientists on earth built a space ship$Questar, which accidentally captured a
creature living on the planet Quta$pie.
,hen this extra$terrestrial creature was brought to earth and shown on T.IH
<h$tsal created so much interest that at once he captured the attention of the world.
/eople found him cute because of his shining feathers with shimmering patterns
displaying rainbow colors.2e became an instant celebrity.
2e had a curved, cosy, and rounded body, dar' liquid eyes. 2e had bushy little hands,
complete with thumbs that he used with astonishing speed and s'ill
<utie /ie became an instant celebrity $ he was a commercial success sold as <utie /ie
dolls, colouring boo's, tee$shirts, fan clubs, cereal cartoons, cartoon serials etc.
:veryone wanted to touch him, see him, hold him
/ara ;F$&%.
2e was given a proper name $ <h$tsal $ and put in a glass prison because he could
live only in an environment similar to that on Quta$pi
. The glass prison was a 'ind of scientific tomb which had a temperature of &%#oG, F%
per cent humidity and an atmosphere of hydrogen, oxygen and careful proportions of
a doEen exotic gases.
Scientists tried to create an atmosphere similar to that of Quta /ie.
.Though <h$tsal was captured from Quta$pie, the planet was not really his home. 2e
merely chanced to be on Quta$pi.
2is elders had sent him there for his initiationH he went out as a boy and was to return
as a ?man?, after having faced difficulties and dangers in strange places. Questar
chanced to travel to Quta$pi and in scooping soil, roc', shale etc from it as samples,
scooped <h$tsal too.

<h$tsal remembered the scientists or inmates of Questar$silver creatures with bubble


faces. 3ut he had become unconscious, as he was in prison of the scientist =ailers,
white$clad men with tubes and goggle faces.
2e 'new that these =ailers didn1t mean to torture him. 2e couldn1t tal' to them
because he had no vocal chordsH he could only loo' past their goggle glasses into
their eyes, begging them to understand, but they could not.
<h$tsal groomed his feathers because they were filters, temperature $ controllers,
respirators etc
. 2is whis'ers were receiverCs antennae, language carriers, voice, life$line. These
lin'ed him to his people and he had to 'eep them in wor'ing order by grooming.
3ut unfortunately in the glass prison, these did not wor' as the atmosphere was not
conducive to his existence. 2e started losing feathers and survival became difficult.
So he had to face a lot of miseries $ he was deaf, dumb, alone, cut off from his land,
his people
1ara $209.-)
2owever, <h$tsal fought his despair with all the courage he could muster up. 2is five
stomachs and numerous organic filters helped him to stomach the food and
atmosphere supplied to him. 2e used to sleep and dream of his home.
3ut on the nineteenth day of his captivity on earth, <h$tsal awo'e to a horrifying
calamity.
2is =ewelled feathers from his bac' and sides and down from his body had fallen out
in the night. This calamity was as bad as it would be if humans suddenly found one
day that they had hair $ coarse hair all over their bodies. This loss of hair made him
feel na'ed and hideous.
2is feathers were to him what clothes were to human. To lose his feathers was to lose
himself. To become na'ed was to become a monster.
2e felt hideous and close to death. 2e crouched in a corner of his glass prison. 2e
realiEed how powerless he was now. The scientists wor'ing on him also pitied him
but they too were powerless.
1ara $.295-)
-ne day, two scientists came to <h$tsal to try a change of diet. They examined him
through the glass of their gas$mas' goggles and hastened to get out because strange
gases began to seep in through their mas's.
-ne of them pressed the button that closed the glass wall. The food bowl =ammed it
which caused a gap. <h$tsal then escaped into the human world through this gap.
<h$tsal sensed that he could breathe in the air and the temperature was pleasant. 2e
would have sent a message of than's to his god, but his whis'ers, his voice, were
gone
Soon he began to explore the world around him. 2e found food in 9rs. <hats worthCs
house. 2e tasted cacti and put some into his bag
Now he wanted a friend desperately as he was very lonely. 2e saw a cat with fur. 3ut
the cat attac'ed him with its claws. 2e questioned the creature with his mental power
and found it useless because the cat had only simple, crude thoughts K basic comfort,
hunting, mating, food etc.
2e also came across a dog, but it too was useless for him.
Then <h$tsal felt rain and saw that his muEEle had some tiny, wire li'e pro=ections.
2e was happy because his feathers were growing
2e was still loo'ing for a friend when he found a small baby <hristopher 2arry
,inters, who was of the same siEe as he was and completely na'ed. 2e learnt that
,inters was an ungrown specimen of the senior species of earth
,hen winterCs mother pic'ed up the baby, waves of love and maternal emotions
flowed between ,inters and his mother which <h$tsal recogniEed by his own
experience.
1ara $5295H)
-ne night, <h$tsal made his way into <hristopherCs nursery and lay down by ,inters.
2e as'ed him to send mental vibes coaxing ,inters to communicate. ,inters smiled
in his sleep. 6n human world ,inters had yet to learn to spea' but as a specimen of
human race he could communicate.
Grom ,inters, <h$tsal learned what it was li'e to plunge through a great wave, to
hunt down animals in dar' forests, to shoot an arrow 'nowing where it would hit
. 2e also learnt of the glories of battle, the terrors of defeat, the chill wic'edness of
sna'es, the smell of wood smo'e etc.
6n turn, <h$tsal told ,inters of the building of crystal cities, of creatures in caves, of
the pioneer ships that opened up the galaxy, of the Ienus invaders and how they were
defeated, of the five ways of 'nowing 4od, of the taste of a certain food that grew
only when the planetCs three moons were full.
1ara $<09<H)
3oth <h$tsal and ,inters underwent a great change after their meetings and tal's
. Now ,inters often laughed and gurgled, bubbled and pulled his motherCs hair
. <h$tsalCs feathers and whis'ers grew. 2e searched for food and exercised vigorously.
when ,inters was four months old, <h$tsalCs feathers were so splendid that they
gathered in the dar'ness
. -ne particular night, he heard his motherCs voice faintly, bro'enly. <h$tsal was
overcome with =oy and forgot all caution. 2e became a firewor', a bombshell,
spinning and Eooming and bouncing off tree trun's
,hen two young poachers saw him, he beamed a single pulse of ecstasy so powerful
that they fell bac'wards. They did not report of a spaceship landing for the fear of not
being believed.
The news headline screamed of spaceship landing. 6t was a big story for the moment
but li'e all news it soon fiEEled out.
. -ne fine day, <h$tsal was goneH the scientists stared at burnt grass in a roped$off
area where his spaceship landed to ta'e him bac' home. No one 'new what became
of <utie /ie.
-nly ,inters could vouch for <utie /ie1s presence.
-nly$one person on earth 'new, however, that was ,inters, but he was too young to
tal' about <h$tsal. 2e tried to tell his mother, but the mother only half$believed him
and scolded him on listening to him, coaxing him to drin' up his mil'.
<haracter Analysis
CTIE 1IE
<utie /ie was an alien accidentally captured by Questar from a planet called Quta$
pi.
2e had feathers displaying colours li'e the rainbow. The feathers were pearly down
on the belly, radiant and shimmering.
2e had dar', liquid eyes, a gentle mouth and bushy little hands.<h$tsal was an
adolescent but was small in siEe.
2e had no vocal chords, 2e could not spea'.
The whis'ers were his receivers, antennae, language carriers, voice and lifeline. 2e
had five stomachs and numerous organic filters.
2e was sensitive, was very upset on losing his feathers and was ashamed of his
condition.
2e did not give up hope, managed food, forced his whis'ers and feathers to grow.
2e exercised furiously, groomed his plumage continuously.
2e was affectionate, he became friendly to <.2. ,intersH was over=oyed on hearing
his mother1s voice. 2e finally reached his planet safely.

C7RISTO17ER 7ARR3 WINTERS. A new born baby, develops an affinity with the
alien, cannot spea' anything, and en=oys the *o#%a+: o2 ",e a&ie+.
M0&"i%&e C,oi*e 80es"io+s. Ea*, =0es"io+ *arries . #arks a+d ea*, s0(%ar" is 2or -
#ark.
Read ",e e!"ra*"s a+d a+swer ",e 2o&&owi+4 =0es"io+s (: *,oosi+4 ",e #os" a%%ro%ria"e
o%"io+.
-. 7e s,0dderi+4&: re#e#(ered ",e si&ver *rea"0res wi", (0((&e 2a*es. 7e 4&i#%sed
",e# (rie2&:@ a+d &os" *o+s*io0s+ess. Now ,e was a %riso+er o2 w,i"e *&ad #e+ wi", "0(e
a+d 4o44&e 2a*es.
a. <utie /ie was on the planet of Quta K pi because
;i.. that was his planet
;ii.. he was there for his initiation
;iii.. he was on a vacation
;iv. he was a scientist
b. The scientists 'ept <utie /ie
;i.. in water
;ii.. in a glass prison
;iii.in a space ship
;iv.in a garden
c. The Csilver creatures with bubble facesC were
;i.. men in space suits
;ii.. inhabitants of Quta /i
;iii.. brothers of <utie /ie
;iv.. imaginary creatures
2. 7is 2ea",ers were s"i&& 04&:@ (0" ",is did +o" worr: ,i#. T,e i#%or"a+" ",i+4 was ",a"
,is w,iskers 4rew. 7e "ook "o *ro0*,i+4 ri4id&:@ 2or ,o0rs o+ e+d@ ,ead 0%@ s"ari+4 i+"o
",e +i4," sk:@ w,iskers vi(ra"i+4@ rea*,i+4 o0" "o ,o#e: a+d "r:i+4 +o" "o des%air w,e+
+o a+swer *a#e.
a. The whis'ers were important to him because
;i. they were his voice
;ii. they were his lifeline
;iii. it was only through them that he could communicate
;iv. they made him loo' beautiful

b. 0uring the night, he would crouch rigidly, for hours on end
;i. staring into the night s'y
;ii. trying to reach out to home
;iii.signaling to people in bubble suits
;iv.vibrating his whis'ers to communicate with someone.
c. 2e did not crouch during the day because
;i..he 'ept sleeping during the day.
;ii.he 'ept himself hidden from the human eyes.
;iii.he 'ept himself hidden from the police
;iv..the whis'ers started growing
.. A (i4 *rea"0re@ +o" +aked@ *a#e o0" "o a""e+d ",e (a(:. T,is@ as C,9"sa& =0i*k&:
de*&ared was ,is #o",er. C,9"sa& *o0&d %i*k 0% ",e s"ro+4@ ",i*k waves o2 &ovi+4
e#o"io+s ",a" 2&owed (e"wee+ ",e #o",er a+d ",e (a(:.
a. A big creature that came to attend the baby was
;i..the baby1s mother
;ii..the baby1s father
;iii..<h$tsal1s mother
;iv.<h$tsal1s father
b. <htsal deduced that
;i.the big creature did not love the baby very dearly
;ii. the child cried on seeing the big creature
;iii.strong emotions flowed between the baby and the big creature.
;iv..the big creature was not na'ed li'e him.
c. <hristopher later tells his mother about <h$tsal .Then
;i. she is excited
;ii.she does not hear him
;iii.she does not believe him
;iv. she scolds him
/ T,e +ews%a%er :e&&ed i". TD 2o&&owed i" 0%. I" see#ed a (i4 s"or: (i44er ",a+ ",e 4ir&
si+4er@ a rea& se+sa"io+
a . The sensational news in the newspaper was
;i.Spaceship lands in 2ert1s village
;ii.Alien has arrived in village
;iii.<utie /ie reached his home planet
;iv.A popular girl singer has arrived
b. Sensation means
;i. excited interest
;ii. a device that responds to a certain stimulus
;iii. having or showing good sense
;iv. aware
c. 9ar' the statementstatements which isare true
i. There was no lin' between the alien creature and the spaceship
ii. Scientists stared at burnt grass in a roped off area Kfrom a patch of burnt field with a
furrow leading to a dimple li'e a P81
iii. <hristopher1s mother thin's that <hristopher is =ust dreaming about the alien.
iv. <utie /ie went bac' in Quta /ie
5. 7e 2o0+d a *rea"0re as&ee%. I" was a ,a+dso#e ",i+4 wi", 20r ",a" re#i+ded C,9"sa&
o2 ,is ow+ 2ea",ers. T,e *rea"0re awoke a+d o%e+ed 4o&de+ e:es. I" ar*,ed i"s (a*k9saw
C, "sa& a+d a""a*ked 2as" a+d vi*io0s&:.
a. <h$tsal quietened the furry creature that attac'ed him
;i. by singing to it
;ii.by tal'ing to it
;iii. by using mental power
;iv. by hitting it
b. The creature that <h$tsal saw was a
;i. baby
;ii. dog
;iii. cat
;iv. fox
c. <h Ktsal deserted the furry creature because
;i. it growled and wagged its tail
;ii. it only thought of its basic comforts
;iii. it went on purring
;iv. he was afraid of it.
S,or" A+swer =0es"io+s i+ a(o0" .09/0 words $ 2 #arks)
&. 2ow did the people react to see <utie$/ie shown on T.I setsM
). 0escribe <utie $ /ieCs appearance.
*. 2ow did <utie /ie affect the commercial side of public lifeM
+. ,hat 'ind of a special place was created by the scientists to 'eep <h$tsalM
". 2ow did <h$tsal get capturedM ,here was he captured fromM
!. ,hy couldn1t <h$tsal establish any 'ind of contact with his captorsM
@. ,hy did <h$tsal try so hard to grow his feathers and whis'ersM
%. 2ow did <h$tsal sustain himself though he underwent great physical miseries on earthM
F. ,hat appalling reality did <h$tsal wa'e up to on the nineteenth day, :arth timeM
&#. ,hat important feature did <h$tsal have which his mother also possessedM
&&. 2ow did <h$tsal find the infant <hristopher 9ary ,intersM
&). ,hat did <utie /ie learn from his interaction with ,intersM
&*. ,hat information did <h$tsal give ,inters, lying down besides himM
&+. ,hat was the effect of <h$tsalCs friendly tal's on ,intersM
&". ,hat happened to <h$tsal after he heard his mother one dayM
&!. 0o you thin' <utie /ie has human characteristicsM ,hyM
Lo+4 A+swer 80es"io+s i+ a(o0" -009-50 words $ < #arks)
&. Oou are <utie /ie1s mother. After getting news from <utie /ie that he was safe on :arth,
you decide to write a letter to your friend mentioning his adventures on :arth.
!al8e *-&()
<utie K /ie captured by a scientist imprisoned
given proper climatic conditions, lost his whis'ers and feathers, no signal
manages to escape, feels lonely, meets <hristopher, feels homesic'
experiences =oy and sorrows as humans do, ta'en away by a space ship from home
). Oou are <h$stal. Oou were captured and brought to the earth by the scientists. Oou were
luc'y enough to establish contact with your mother and return to your native planet.
,rite a letter to your friend describing your experience.
*. <hristopher grows up into a teenager and recollects his experiences with <utie /ie.
0escribe these experiences in the form of a diary entry.
+. Oou are <.2. ,inters, who has grown up and became a space scientist. 6n one of Space
Research 9ission you happened to reach <utie /ie1s planet and renewed your friendship.
,rite a diary extract describing your reunion R your experiences on the alien planet.
". As <hristopher1s mother, you are surprised at his strange behaviour. Oou =ust can1t
understand his stories and are confused and worried. ,rite a diary entry expressing your
concern over your son1s behaviour.
!al8e *-&()
Surprised at my son1s strange behaviour and afraid$$ -r is he suffering from some
mental or psychiatric ailment $ No, it cannot happen$. 3ut then why is he behaving so
strangelyM
2e is too young to tal' about aliens but still he spea's endlessly about an alien that
befriended him
/erhaps it is because of television and its science$fiction rap is so full of news these
days about an alien spaceship which they say landed in the 2erts Iillage.
Some months ago, they did show an alien K <h$tsal.
6s there any connection between him and my sonM$. <hristopher does not 'now how
to hold a spoon properly and there he is tal'ing about aliensM
,ill my son come to any harmM -h 4od please help us8
________________
T,e Le""er
B: D,0#ake"0
I+"rod0*"io+
The letter is a poignant story about parental love and longing to meet one1s children.
<oachman Ali suffers the pain of separation from his only child and dies waiting for a letter
from her. 2er separation from him transforms him completely and ma'es him a
compassionate and sensitive man. The theme explores emotions of love, relationships, and
the grief that parting brings. There is also an emphasis on the need to empathiEe with others
instead of showing indifference and insensitive behaviour. 0huma'etu has portrayed the
reality of life by his lively imagination invested it with emotion, and touched it with a
romantic idealism. 2is story casts an irresistible spell by the freshness of its theme, style and
technique, a rich variety of incident, plot and situation, and its diverse world of distinctly
individual characters, brilliant and idealistic.
Gis" o2 ",e &esso+
1ara -9-2 ' A&iCs +ever e+di+4 wai" 2or +ews 2ro# Miria#
The story begins with a description of the long and arduous =ourney that Ali ma'es
every day to the post office in the hope of receiving a letter from his daughter 9iriam
who has not been in touch with him since her marriage " years ago. 2e starts early
when the whole town is asleep. Though he is poor and old, it is his faith and love for
his daughter ma'es him bear the bitter cold as he plods supporting himself on a staff.
The post office becomes his place of pilgrimage .2e dedicatedly goes there for five
long years, sits there through the day ,is moc'ed and =eered at by the post office
employees as he sits at a specific place each day
2e is treated li'e a mad man by everyone and is the ob=ect of ridicule for one and all.
They thin' that he comes in vain to receive a letter that would never come. They
would call out his name falsely to indicate that he has received a letter and en=oy the
disappointment on his face.

1ara -.9-5 ' A&i 'a *,a+4ed #a+
Ali was a s'illed and clever hunter once .2e was so addicted to hunting that he could
not spend a single day without hunting. Ksomething he was very good at. As he grew
older he began to change .2is only daughter 9iriam married and left him to stay with
her soldier husband from a regiment in /un=ab .2e transforms completely and feels
lonely in her absence. 2unting no longer interests him. 2e understands the meaning
of love and separation when he misses his daughter and in the simple hope of
receiving a letter from her some day he goes to the post office religiously.
Although he has never received a letter, he 'eeps at it.
1ara -<9.0 99Miser: a" ",e %os" o22i*e
The post office becomes a place of pilgrimage for him because of the devotion and
regularity with which he comes to visit it. Receiving a letter from his daughter
becomes the sanctimonious purpose of his life.
Nobody at the post office seems to understand <oachman Ali. They are indifferent
and use him only a sub=ect of their ridicule and derision. They =ust want to en=oy the
sight of him =umping to the sound of his name. They =ust have fun and laughter at his
expense, never for once trying to understand his pain
3ut Ali, does not pay heed to the cruel treatment that he receives and with ceaseless
faith and endurance he comes daily to the post office even if to go empty handed.
The post office employees simply write him off as a mad man before the post master.
1ara .-95- ' A&i e#(ra*es dea", 're#ai+s ,o%e20&
Towards the end of his life Ali suffers from ill health and stops coming for a while
/eople at the post office`have no sympathy , understanding or concern to try and
guess the reason but are curious to 'now why he hasn1t come
At last he returns on recovering a little but signs of ill health ,old age and approaching
end can be seen on his face
2e can no longer remain patient and pleads with the ill$tempered postmaster as'ing
him if there was a letter for him
The postmaster who is in a hurry gets irritated and calls him a pest. 2e is very rude to
Ali and thoughtlessly and angrily scolds him.
Ali is sad and helpless. 2is patience is exhausted but his faith remains intact.
3efore departing that day, Ali gives five gold guineas to >a'shmi 0as Kthe office
cler' and extracts a promise from him to deliver his daughter1s letter at his grave
Ali is never seen again as he dies before receiving any letter
1ara 529F2P1oe"i* 60s"i*e $a literary device that shows an ironic twist of fate
intimately related to a characterCs own conduct. The postmaster who was rude to Ali
suffers =ust li'e Ali did.
Time ta'es a turn. The postmaster is restless and anxious because he has not received
any news from his daughter who is in another town and is unwell
2e anxiously loo's through the mail only to find 9iriam1s letter addressed to Ali.2e
immediately recalls the past and realiEes the pain and anguish Ali must have gone
through. A single night spent in anxiety ma'es him understand Ali1s heart and soul
2e is filled with a deep sense of remorse and repentance for having been rude to Ali
0ecides to hand over the letter himself to Ali
2e hears a soft 'noc' on the door at "`thin's it is Ali who has come to receive the
letter. 2e opens the door immediately and sees old Ali bent with age standing outside.
Actually it is a hallucination that the postmaster gets. 2e is fearful and astonished to
see the unearthly loo' on Ali1s face. Ali disappears as he came leaving the postmaster
in a state of utter shoc'.
>a'smi 0as, the cler' is shoc'ed to hear the postmaster call out the name of
<oachman Ali who has now been dead for three months. The letter is found near the
door
>a'smi 0as tells him about his last meeting with Ali to convince him
That evening both of them go to place the letter on Ali1s grave
The postmaster understands the essential human worth of letters and doesn1t =ust treat
them as envelopes and postcards anymore
/art of his penance is to 'eep waiting for the letter from of his daughter.
_______________________
I. M0&"i%&e C,oi*e 80es"io+s9 :ach question carries * mar's and each subpart is for &
mar'.
Read ",e 2o&&owi+4 e!"ra*"s a+d a+swer ",e =0es"io+s (: *,oosi+4 ",e #os" a%%ro%ria"e
o%"io+:
-. Be,o&di+4 ",e woode+ ar*, o2 ",is (0i&di+4@ ",e o&d #a+ was 2i&&ed wi", ?o: ",a"
",e %i&4ri# 2ee&s w,e+ ,e 2irs" sees ",e 4oa& o2 ,is ?o0r+e:.
a. This old man was filled with =oy on reaching the building because(
i. 2is daughter has sent him a letter
ii. 2e had reached here with great difficulty
iii. 2e loved coming here
iv. 2e was expecting some good news
b. The old man1s happiness is compared to that of a %i&4ri# because
i. 2e had completed his pilgrimage too
ii. 2e was happy li'e other pilgrims
iii. 2e was soon going on a pilgrimage
iv. 2e was filled with hope and faith
c. The word ?o0r+e: here refers to(
i. The final =ourney to heaven
ii. The tiresome =ourney through the town
iii. The trip from home to town
iv. The =ourney to the pilgrimage
2. B0" w,e+ ",e eve+i+4 o2 ,is &i2e was drawi+4 i+@ ,e &e2" ,is o&d wa:s a+d
s0dde+&: "ook a +ew "0r+.
a. The phrase eve+i+4 o2 &i2e refers to
i. 3ad times
ii. Time of death
iii. -ld age
iv. Sad ,unhappy times
b. P2e left his old ways1 means(
i. 2e stopped visiting his daughter
ii. 2e stopped 'illing birds
iii. 2e gave up violence
iv. 2e stopped tal'ing to his friends
c. The transformation in his life was a result of
i. 9iriam1s elopement
ii. Separation from his only daughter
iii. 2is old age
iv. 2is visit to the post office
d. 2is life too' a new turn means(
i. 2e pic'ed up a new =ob
ii. 2e realiEed the meaning of love and separation
iii. 2e accepted 9iriam1s alliance
iv. 2e gave up hunting
.. 7is e:es were 2i&&ed wi", "ears o2 ,e&%&ess+ess@ 2or ,is %a"ie+*e was e!,a0s"ed@
eve+ ",o04, ,e s"i&& ,ad 2ai",
a. The tears of helplessness were a result of
i. 2is illness and exhaustion
ii. 9iriam1s long absence
iii. 2is disappointment
iv. The indifferent and rude attitude of the postmaster
b. 2is patience was exhausted because
i. he was an impatient man
ii. 2e was growing old
iii. 2e was tired of wal'ing to the post office daily
iv. 2is wait for his daughter1s letter had extended to five years
c. Ali1s faith is rewarded when
i. The postmaster learns a lesson
ii. 9iriam sends a letter for her father
iii. 2e finally meets death
iv. 2e gets the letter on his next visit
/. For +ow ",e %os"#as"er 0+ders"ood A&iCs ,ear" a+d ,is ver: so0&. A2"er s%e+di+4
(0" a si+4&e +i4," i+ s0s%e+se@ a+!io0s&: wai"i+4 2or +ews o2 ,is da04,"er@ ,is
,ear" was (ri##i+4 wi", s:#%a",: 2or ",e %oor o&d #a+L
a. The postmaster was anxious because
i. 2is daughter had married secretly
ii. 2is daughter had gone away
iii. 2e had seen Ali1s ghost
iv. 2e hadn1t heard from his daughter
b. The postmaster felt sympathetic towards Ali because
i. Ali cried tears of helplessness
ii. 9iriam1s long$awaited letter came after his death
iii. 2is own sorrow transformed him
iv. The cler' told him that Ali was dead since long
c. The postmaster1s experience teaches us that
i. -ne must not be rude and indifferent to others
ii. :veryone li'es to receive letters from their children
iii. :ach letter has an essential human worth
iv. ,e must try to identify with and understand others1 sorrows
5. A&i was +ever see+ a4ai+ a+d +o o+e "ro0(&ed "o i+=0ire a2"er ,i#
a. Ali was never seen again because
i. 2e stopped coming to the post office
ii. 2e was already dead
iii. 2e went away to 9iriam
iv. 2e changed bac' to his old ways
b. No one bothered to inquire about him because
i. They were relieved that he had stopped coming
ii. They were unsympathetic and indifferent
iii. They despised him
iv. 2is absence was hardly noticed
c. The last persons who saw Ali alive was
i. The postmaster
ii. The cler'
iii. The postmen
iv. 2is daughter 9iriam
II. S,or" A+swer =0es"io+s i+ a(o0" .09/0 words$ 2 #arks ea*,)
&. 2ighlight Ali1s character traits as a Shi'ari.
). ,hat was the reason behind Ali1s transformationM
*. ,hy was Ali considered to be a madman by the post office officialsM
+. ,hat was the attitude of the postmaster towards AliM
". Ali had Pexhausted his patience but not his faith1. :xplain.
!. ,hat do we understand about Ali1s character from his regular visits to the post office
in spite of all the humiliation that he facedM
@. 2ow Ali1s faith is finally vindicatedM
%. PThe haughty temper of the official had quite left him1. ,hat change came about in
the official and howM
F. P2e dropped it as though it had given him an electric shoc'1. ,hat is being referred
toM ,hy was it such a shoc'M
&#. ,hat promise did Ali extract from the cler' and how did he ensure that the promise is
fulfilledM
&&. ,hat significant lesson did the postmaster1s experience teach himM
&). 2ow did doubt and remorse trouble the otherwise cold and merciless postmasterM
&*. The officials at the post office had much fun at the expense of Ali. <ite two examples
to prove this.
&+. ,hat was the /ostmaster1s state of mind after he had given Ali1s letter to >a'shmi
0as for delivering it to him
III Lo+4 A+swer 80es"io+s i+ a(o0" -009-50 words: < Marks
&. PThe newly awa'ened father1s heart in him was reproaching him for having failed to
understand Ali1s anxiety1. As the postmaster write a diar: e+"r: outlining your
feelings about your former behaviour with Ali.
). 6magine you are Ali .Oou are completely exhausted by your futile visits to the /ost
-ffice. ,rite a letter to your friend Ashraf telling him about the disappointment and
humiliation that you undergo everyday at the /ost -ffice and your decision to not go
there anymore.
*. Dustify the title$ The >etter
+. 6magine that Ali writes a letter to his daughter 9iriam after he hands over the five
guineas to the cler' at the post office. ,rite his letter.
". 6magine you are >a'smi 0as. Oou have finally fulfilled the promise that you made to
Ali. ,rite a letter to your friend expressing how satisfied and relieved you are
!. 6magine Ali writes his diary daily. 2e feels disgusted with life in going to the post
office and waiting daily for 9iriam1s letter which never comes. This feeling of utter
despair has been triggered by the /ostmaster1s insult. Ali writes his diary page that
evening. ,rite that page.
@. After " years, 9iriam writes a letter to her father, expressing her personal problems
for which she could not write. ,rite the letter on behalf of 9iriam.
%. >a'shmi 0as writes a letter to 9iriam, mentioning the sudden death of her father Ali.
,rite the letter, giving details of Ali1s sufferings and hardships.
________________
The Bltimate Safari
3y Nadine 4ordimer
I+"rod0*"io+
Nadi+e Gordi#er was born in Springs, South Africa on November )#, &F)* to 6sidore
4ordimer, an immigrant Dewish watchma'er, and Nan 9yers, who had immigrated to South
Africa from 4reat 3ritain as a young child. 6n &FF&, 4ordimer was awarded the Nobel /riEe
for >iterature. Also in &FF&, 4ordimer published 5ump and /ther 6tories, the collection
which includes ?The Bltimate Safari.7 >ong regarded as one of South AfricaCs leading
political activists and intellectuals she attributes the inspiration for the story to a visit she
made to a camp for 9oEambique refugees. All of her boo's, including her collection of short
stories`in some way address apartheid or its effects.
?The Bltimate Safari,? follows the story of an unnamed narrator and her family as they leave
their 9oEambique village for a refugee camp across the border in South Africa.
Gis" o2 ",e S"or:
1ara -9/
&he Ultimate 6afari is the story of a family from 9oEambique that sets off on a long
=ourney by foot in search of refuge from war.
The narrator is a young girl who, li'e the other characters in the story, remains
nameless. The girl has an older brother and a baby brother.
The story begins on the night that their mother ventures out of the house in search of
coo'ing oil. She never returns home perhaps the bandits have 'illed her.
The childrenCs father disappeared some time ago. 2e is a solider fighting the war
against ?bandits? who have destroyed the country.
The young girl explains that the first two times the bandits came to their village, they
stole everything. The third time they came, there was nothing left to steal so they set
the houses on fire. .
,ithout their mother, the children do not 'now where to go. The narratorCs eldest
brother 'eeps a piece of bro'en wood in his hand all night for protection
The children wait for their mother all of the next day. They donCt 'now what day it is
since the village church and school are no longer open.
,hen the sun goes down, the childrenCs grandparents arrive. Someone in the village
had told them that the children were left alone. Their grandmother is big and strong,
while their grandfather is small and quite old. The grandmother ta'es the children
bac' to her house.
1ara 59A
The childrenCs grandmother decides that the family should leave the village. They
want to be away from the bandits and the place where their mother never returned.
The narrator is happy to thin' of such a place as ?away.? The grandmother exchanges
her church clothes for dried ?mealies? to ta'e with them.
The grandmother thought they would get water from a river but they are forced to
turn bac'. They stop at a nearby village. 2ere the grandmother sells her shoes in
exchange for a plastic container to carry water.
At the village, the family meets other people who are also leaving. The family follows
this group, because they seem to have a better idea of where they are going.
They travel through Lruger /ar', an enormous reserve of animals.
As the group =ourneys toward the par', women who carry children on their bac's
grow tired. The par' is surrounded by an electric fence.
1ara H9-2
The narratorCs brother catches a tortoise for the family to eat. The man who leads the
groups explains that they canCt ma'e a fire because the smo'e would signal the police
that they are there.
6f the group is found they will be sent bac' to where they came from. They must
move li'e animals among the animals and 'eep away from roads and white peopleCs
camps.
A loud noise frightens the children. The narrator thin's itCs the police, but it is =ust a
herd of elephants. The leader tells them to stand still and stay quiet until the elephants
pass. The elephants are too big to be frightened of anything
0uring the day, when the sun is hot, the group sees sleeping lions. The lion blend
into the color of the grass $ the leader spots them and leads the group the long way
around them.
They are all getting tired. The narratorCs older brother is difficult to wa'e once he falls
asleep. Glies crawl on the grandmothers face and she doesnCt even swat them off. The
granddaughter is frightened.
They wal' day and night. Sometimes, the group stops to sleep at night. They sleep
close together.
-ne night they hear lions approaching, and group gets closer together. The ones who
are sleeping on the edges want to be closer to the middle. The narrator prays for the
lions to leave.
The leader sha'es a tree branch and shouts out to scare the lions away. They lions
leave, but the group can hear them growling from far away.
The grandfather is tired. The leader and oldest grandson have to carry him from stone
to stone when the group crosses rivers.
The grandmother is still strong, but her feet are bleeding. She canCt carry her bas'et
on her head anymore so she has to leave their belongings behind. They can carry
nothing except the youngest brother.
The group eats some strange fruit that ma'es them sic'. The grandfather wanders off
to be by himself when he is sic'.
1ara -.9-H
The rest stay near each other in the long elephant grass. The leader tells them they
have to move on, but the family as's the others to wait for the grandfather to catch
up. The grandfather doesnCt come bac'.
They loo' for the grandfather the next day by splitting up in groups. No one can find
him. The narrator and the grandmother spot a vulture soaring overhead. The leader
tells them they must move on. 2e says that if the children donCt eat soon, they will
die.
The grandmother doesnCt respond to the leader. 2e tells her he will bring her and the
children water before the others leave.
The narrator is worried that the police will find them. She cries, but the grandmother
doesnCt notice.
The grandmother gets up and tells the children they must follow the others.
1ara 2092.
The group comes to a big tent that the narrator describes as being much bigger than a
church or school. This is a refugee camp.
The tent is li'e a village. /eople made bloc'ed off living areas inside the tent with
cardboard to mar' their space. Some used ground roc's to draw designs on sac's.
Small children play in the mud but not the narratorCs youngest brother. The
grandmother ta'es the young boy to the clinic on the day the doctor comes to the
camp. The doctor thin's it is the result of not eating proper meals during war times
and while passing through Lruger /ar'.
2e has not developed properly. The youngest brother lies around all day. 2e seems
li'e he wants to say something but canCt. Sometime he smiles, but he doesnCt laugh.
The narrator and the older brother were li'e the youngest when they arrived too. The
people from the nearby village too' them to the clinic. 2ere they had to sign a paper that
stated they had come ?away? through Lruger /ar'. The children were given something to
drin' and shots to 'eep them from getting sic'.
1ara 2/
The grandmother is still strong. She signs the children in and gets them a place in the
tent near one of the sides
3ecause the grandmother is still strong, she wor's where people build houses.
She buys tea, sugar, mil', and soap with the money she earns. The store gives her a
calendar that she hangs in the tent.
The narrator does well in school. The narrator and her older brother have to do their
homewor' in the afternoon before dar' because candles are too expensive. There is
no room in the tent $ $ people have to lie down close together, as they had in Lruger
/ar'.
The grandmother still hasnCt been able to buy herself any shoes but she bought her
two eldest grandchildren shoes for school.
The children clean them every morning and the grandmother inspects their cleaning
=ob. No other children in the tent have real shoes.
1ara 25
The narrator says some days in the tent it feels li'e home before the war, li'e they had
never gone away. ,hite people come to ta'e photos of people in the tents. They say
they are ma'ing a film but the narrator doesnCt 'now what this is.
A white woman as's the grandmother questions that are translated by someone into
their own language. She as's how long they have been there.
The grandmother says they have been there for two years and one month. The woman
as's what her hopes are for the future. ,hen the grandmother says she has no future
hopes, the woman as's what her hopes are for the children.
The grandmother hopes they will get an education, get good =obs and have money.
The woman as's if the grandmother wants to return to 9oEambique. She says that
when the war is over, they will not be allowed to stay here.
The grandmother says she will not go bac'H there is no homeH there is nothing.
As the narrator listens to her grandmotherCs statements, she wonders why she wonCt go
bac'.
The narrator wants to go home. She would ta'e the long voyage bac' through Lruger
/ar' after the war is over and the bandits are gone. She wants to see her mother again
and assumes that she will be waiting there.
She e3-ects that her grand.ather 1ill e there too4 that so#eho1 he #ade
his 1a5 ac6 slo1l5/ She sa5s4 7The58ll e ho#e and , 1ill re#e#er
the#/7
,he'es dealt within the story
A%ar",eid
3etween &F+% and &FF), the Republic of South Africa had an institutionaliEed system
of racial segregation 'nown as ?apartheid?`the Afri'aner word meaning
?separateness.?
The system stripped all South African blac's, coloreds, and 6ndians of their
citiEenship rights, apartheid was instrumental in helping whites to maintain power in
the predominantly blac' country.
Nearly all of 4ordimerCs wor' addresses, in some way, the effects apartheid have had
on whites and blac's ali'e.
The mention of ,hite people in the story is only at two three places but they way it
has been done conveys how differently are they treated by the natives.
Fa#i&:
The story effectively explores the role of family in difficult times.
The story begins with the fact that the narrator has lost her parents to the war.
The way 4randmother ta'es over the responsibility of the three young children is an
indication of the family ties, bonding and sense of responsibility.
The way grandmother 'eeps the family together, the sacrifice that she ma'es for the
future of these children is reflection of the strong family bonding.
7o#e&ess+ess
-ne of the ma=or effects of the South African policy of apartheid was the
displacement of millions of blac's in the region.
At the end of the story, a white reporter as's the grandmother if she ever wants to
return home. ,hile the young girl dreams of a day she will be reunited with her
mother and grandfather in their home village, the grandmother responds directly by
saying, ?There is nothing. No home.7
The finality of her tone reflects how she has severed all contacts with her previous
life.
0isplacement from one1s home ma'es people feel rootless and the author captures
that in =ust one sentence.
Ro&e o2 Wo#e+
6n a society ruled by war, the women of the villages were forced to ta'e over all
parenting responsibilities, becoming both the homema'er and wage earner.
6n ?The Bltimate Safari,? the burden of this dual responsibility falls onto the
shoulders of the grandmother, who must not only lead her grandchildren to safety, but
who must also ta'e over the care of her own husband whose dementia has rendered
him useless.
To a lesser degree, the narrator must also ta'e over parenting responsibilities by
carrying and caring for her infant brother who begins to grow wea' from malnutrition
during their tre'.
C,ara*"ers
T,e Da04,"erI",e Narra"or
A young girl of nine or ten who reveals very little about herself.
6t is through her eyes that the story of her and her familyCs arduous tre' away from
their village to the refugee camp is told.
She understands very little about the war, or the reasons behind it, except to comment
about the fear the bandits have instilled into her people and to describe the effects
their raids have had on her life.
An astute observer, she conveys much of the tone of the story through her
descriptions of the tre'( her grandfather roc'ing to and fro ma'ing little noisesH flies
buEEing on her grandmotherCs faceH her older brother becoming silent li'e their
grandfather.
Although we ultimately learn very little about the narrator herself, it is through her
descriptions that the story unfolds.
T,e Gra+d2a",er
The grandfather does little more than roc' side to side and ma'es little noises in this
story.
2e is clearly suffering from some form of dementia or the effects of a mental
brea'down, and in the course of the tre' through Lruger /ar', he wanders off through
the high grasses, becomes lost, and must be left behind by the family.
T,e Gra+d#o",er
As the matriarch of her extended family that includes her husband and her
grandchildren`the narrator, and the narratorCs younger and older brothers`the
grandmother is the strongest adult character in the story.
6t is through her vision and leadership that the family is able to escape the danger
wrought by the rebels and travel through Lruger /ar' to a refugee camp across the
border.
-nce her family settles into the refugee camp, she finds wor' hauling bric's, and she
oversees her grandchildrenCs education.
T,e Li""&e Bro",er
>ess than a year old when the family is forced to leave their village, the little brother
is three when the story ends.
6n that time he suffers greatly from malnutrition, and as he grows older, his older
sister notices that he barely spea's, a result, she believes, of having too little food
during their =ourney.
T,e Mo",er
Similar to the father, we 'now nothing about the mother except that she left one day
for the store and never returned, forcing the narratorCs grandparents to ta'e over
responsibilities for the children during the war
T,e Fa",er
Although he never appears in the story, the fatherCs absence, and presumed death in
the war, is significant as it helps to set the tone of the story, and without him, the
narratorCs family must survive on their own.
I Si4+i2i*a+*e o2 ",e "i"&e o2 ",e s"or:
The word CultimateC may be understood as the most defining moment in the narratorCs
life. 6n one sense she ma'es the ultimate =ourney of her life that leaves lasting
impression on her life. 6t ma'es definite changes her life. 6t ultimately matures her.
-n the other hand, the word CsafariC holds ironical meaning in the lesson. 6nstead of
en=oying a safari, ;watching and observing the animals in their natural habitat. she
suffers a safari $forced on her by circumstances. The safari is through Lruger /ar' but
she is not so luc'y as to go through it as an expedition. 6nstead her life becomes an
expedition itself with dangers looming large at every corner, with diseases
surrounding the area and she is forced to ma'e a =ourney against her wishes. 6n fact
the trip is her and her family1s last desperate attempt to save their lives.
The title though rings a sad tone but sums up the story so appropriately in it. 6t is
indeed the ultimate =ourney that the circumstances force on the narrator and her
family. They come out of it bruised, battered, emotionally spent but determined more
than ever to ma'e the best of the opportunity that life has served them.
(ul!i#le Choie )ues!ions (eah *ues!ion arries + %ar,s and eah sub
#ar! is "or -1 %ar,)
Based on your unders!anding o" !he #oe%. hoose !he orre! o#!ions
"or !he "ollo/ing0
A 45e stayed there all day. 5e 2aited for her. % dont )no2 2hat day it 2as6
there 2as no school1 no church any #ore in our villae1 so you dont )no2
2hether it 2as Sunday or Monday
i. There was no school or church in the village as
a. 6t was a bac'ward village
b. The bandits had burnt everything
c. They had been closed by the order of the bandits
d. They had been destroyed in the Glood
ii. The spea'er waited for
a. 2er grandmother
b. 2er mother
c. The bandits
d. 2er younger sister
iii. The running of school and church is an indication of
a. 6nterest of students in academics
b. The 4overnment is ta'ing care of things
c. /eace times
d. /eople are religious
B 42e 2ere lad to thin) there #ust be such a place6 a2ay
i. P,e1 refers to
a. 4randparents
b. <hildren
c. 3andits
d. Iillagers
ii. Such a place refers to
a. Safe place
b. :ntertaining place
c. Gamily place
d. >oving place
iii. The decision to go was ta'en by
a. 4randfather
b. Narrator
c. 4randmother
d. bandits
< 4'e said 2e #ust #ove li)e ani#als a#on ani#als1 a2ay fro# the roads1
a2ay fro# the 2hite people
i. This piece of advice was given by
a. The elder brother
b. The man leading them
c. The grandfather
d. The police
ii. They were as'ed to move li'e animals because
a. They were illegally crossing the forest
b. They were being trained to live in the forest
c. This was the only way they could save themselves
d. They were being hunted by the white people
iii. The statement suggests that the people should adopt the quality of
moving
e. Stealthily
f. <amouflaging
g. Aggressively
h. 3raEenly
D 4% sa2 flies cra2lin on our rand#others face and she didnt brush the#
off6 % 2as frihtened.1
i. The grandmother didn1t brush the flies away as
a. She had become senile
b. She was too exhausted
c. She li'ed the feel of flies on her face
d. She was sleeping
ii. The P61 is frightened because
a. /eople around her were losing their loved ones
b. She feared for the well being of her grandmother
c. They were going through a dangerous part of the =ungle
d. ,hat would she tell her mother
iii. The spea'er and her family is at
a. A refugee camp
b. 2er village
c. Lruger /ar'
d. <orbett /ar'
: Our rand#other loo)ed a2ay fro# her and spo)e-.here is nothin. No
ho#e.
i. The grandmother is spea'ing to
a. 4randdaughter
b. ,hite woman
c. 2er neighbor
d. Refugee officials
ii. The grandmother doesn1t want to go home as
a. She has severed all connection with the past
b. She li'es living in the refugee camp
c. 4oing bac' will revive old memories
d. She has chal'ed out a new life for them
iii. -n hearing this the granddaughter is
a. <onfused
b. Angry
c. 6rritated
d. Resolute
II S,or" A+swer 80es"io+s i+ a(o0" .09/0 words $2 #arks)
&. ,hy do you thin' the 4randmother didn1t want to go bac' to the villageM
). ,hat efforts did the 4randmother ma'e to give a home away from home to
her grandchildrenM
*. ,hy did the 4randmother decide to move with the rest of the group instead of
waiting for her husbandM
+. 4randmother exchanged her shoes for the water container for the =ourney.
,hat does this action of her reflect about herM
". 2ow did the children protect themselves through the night of third attac'M
!. :xplain the statement made by the narrator, P6 say grandmother before
grandfather because it1s li'e that1
@. ,hy did the grandmother decide to leave the villageM
%. ,hat did the guide mean when he said, Pmust move li'e animals among the
animals1
F. 0escribe the narrator1s first impression of the Refugee camp.
&#. 2ow did the unseen presence of the lions affect the leader of the groupM
III Lo+4 A+swer 80es"io+s i+ a(o0" -009-50 words $< #arks)
&. The narrator compels the grandmother to write her memoirs. As grandmother
write the ordeal of bringing up three 'ids in the war ridden country.
). The white woman, who ma'es a documentary on the inmates of the refugee
camp, is impressed by the determination of the grandmother. She writes a
letter to the Red <ross Society as'ing them to increase her stipend. ,rite the
letter.
*. The narrator is unhappy with the 4randmother1s decision to never go bac'.
She writes in her diary her feelings and desire to go bac' to her native place to
find her mother.
+. The ultimate safari is a story of courage and determination. :lucidate
1OEMS
Ni4," o2 ",e S*or%io+
B: Nissi# EQekie&
I+"rod0*"io+
6t is a short narrative poem in which Nissim :Ee'iel recalls the night his mother was stung by
a scorpion in a poor village in 6ndia. Anxious neighbours pour into her hut to offer advice and
help. All sorts of cures are tried by the neighbours, her husband and the local holy man, but
time proves to be the best healer $ CAfter twenty hours it lost its sting. After her ordeal, the
mother is simply than'ful that the scorpion stung her and not the children.
At another level, it depicts the 6ndian ethos and cultural richness through a simple incident
and epitomiEes the typical 6ndian motherhood which depicts sacrifice and affection. 6n this
poem Nissim :Ee'iel poignantly describes a motherCs selfless love for his children as she, in
spite of having been bitten by a scorpion herself and narrowly escaping death, is grateful to
god, that the scorpion had bitten her and spared her children
Gis" o2 ",e 1oe#
>ine &$))
The poet remembers the night when his mother was bitten by a scorpion. 2e says that
the continuous rain for ten hours had driven the scorpion to ta'e shelter in the house,
where it crawled beneath a sac' of rice.
,hen his mother entered the dar' room, the scorpion bit the mother on her toe. She
shrie's and writhes in pain
The peasants came into their hut ?li'e swarms of flies? to sympathiEe with the family
and participate in the rituals
The neighbours buEEed the name of god hoping to defuse the effect of the poison
.through the power of their chants they hoped to stop the scorpion from moving for
they believed that if the mother moved the poison would spread.
The neighboursC candles and lanterns throw Cgiant scorpion shadowsC on the walls
;line &*.. The scorpion has already fled, so are these images could be of the people
themselvesM ;A scorpion has eight legs, so the shadow of a small group of people
standing together could loo' li'e a scorpion..
They searched for the scorpion in the light of candles and lanterns.
The superstitious villagers lin' the scorpion to Cthe :vil -neC ;line &#.. They hope
that this suffering may be a cleansing from some sin in the past They claim that the
poison will help in many ways, for example by burning away the sins of the womanCs
former life $ Cher previous birthC ;line &F. and ease her life after this one $ Cher next
birthC ;line ))..
>ine )*$**
They also prayed that the sum of evil may be balanced in this unreal world against
the sum of good that she had done. They prayed that the poison would purify her of
her desires and ambitions. 6n other words the mother1s suffering was for her good.
All the people sat around, the mother in the centre, while she continued to groan and
twist with pain, on the mat.
There was peace of understanding on each face as they felt that she had approached
her end. 6t is ironic that they are at peace while she is in discomfort. The
understanding clearly is about the divine purpose behind the suffering and so they
don1t feel the need to do anything other than praying to alleviate her pain. 9ore
neighbours continued to enter the house with more candles and lanterns. The insects
were also increasing and the rain too continued.
>ine *"$+%
The narrator1s father is a disbeliever and a rationalist $ a doubter of superstition and a
believer in scientific reason, in other words, he does not believe in superstitions and is
not religious. Oet when his wife is suffering, he tries Cevery curse and blessingC ;line
*@. as well as every possible antidote of which he can thin' to help her. 2e is far from
being rigid about his beliefs.
2e puts a mixture of powders and herbs on her toe, a little paraffin on the bite and lit
it with a match $ hoping to burn the poison away.
The narrator saw his motherCs toe on fire and must have felt afraid. A holy man, the
priest, performed some ritesC to probably tame the poison.
-nly after twenty hours did the poison subside the mother was relieved of the pain,
and than'ed god that the scorpion bit her and spared her children
A &ook a" ",e 1oe#:
The poem is written in free verse. There is no apparent rhyme scheme followed.
The language used is quite prosaic.
The poem starts off in first person and then switches to third person.
The poet only narrates what happened in an ob=ective manner.
The poem is narrated from the point of view of the child
The poem also depicts the behavior of people in small places where everyone gets
involved in everyone1s life
The poet is sympathetic towards the Scorpion in the first few lines, rain had driven
the scorpion to shelter under the sac' of rice, it had to ris' its life in the rain again.
The superstitious believes of the villagers are highlighted. ; with every move of
scorpion the poison will spread, sins of previous birth are burned, sufferings now will
decrease the miseries of next life, poison will purify her of desire and ambition.
The father who is a rationalist resorts to everything suggested. This depicts his
concern for his wife.
The poet describes the night and happenings very vividly, the poem is full of images.
The poem almost reads li'e a story in prose. The first part ;line &$+". is long and full
of activity $ the scorpionCs bite and the reaction of the villagers.
The second part, the motherCs reaction, is =ust three lines long
The final three lines are poignant and effective( where everyone else has been
concerned for the mother, who has been in too much pain to tal', she thin's of her
children. 3y using direct speech for the mother, :Ee'iel effectively shows his
motherCs selflessness. 2e chooses her simple words to end the poem to highlight his
love and admiration for her
The mother doesnCt show any bitterness over her ordeal( she is =ust grateful that it was
she who was hurt rather than her children
The title is deceptive. The scorpion does not ta'e .The poem is not really about the
scorpion or its sting, but contrasts the reactions of family, neighbours and his father,
with the motherCs stateliness and courage.
Li"erar: Devi*es 0sed i+ ",e %oe#
I#a4er: can be defined as the representation of an experience through language.
Though visual imagery is most often used in poetry, an image may also represent a sound, a
smell, a touch or a feeling or sensation. 6n this poem, the poet has made use of various types
of imagery(
Iisual imagery
scorpion crawling beneath a sac' of rice
peasants came li'e swarms of flies
Smell imagery
smell of candles
smell of burning oil in the lanterns
Tactile experience
scorpion biting the mother
father pouring paraffin on the toe.
6nternal sensation
fear
pain
Sound imagery
buEEed the name of god a hundred times
they clic'ed their tongues
Tra+s2erred E%i",e"9 diabolic tail
Si#i&e9 peasants li'e swarm of flies
A&&i"era"io+$ poison purify ;> )@.
poured a little paraffin ;>*F.
flame feeding ;>+&.
herb and hybrid
Re%e"i"io+9 more candles, more lanterns$$$$$$ ;reflects poets frustration.
They said
I M0&"i%&e C,oi*e 80es"io+s $Ea*, =0es"io+ *arries . #arks a+d ea*, s0( %ar" is 2or 0-
#ark)
2ased on 5our understanding o. the -oe#4 choose the correct o-tions .or the
.ollo1ing9
-. To *raw& (e+ea", a sa*k o2 ri*e
1ar"i+4 wi", ,is %oiso+9 2&as,
O2 dia(o&i* "a&e i+ ",e dark roo#9
7e risked ",e rai+ a4ai+.
a. The scorpion had to part with its poison
i. in self defense
ii. in anger and frustration
iii. in fear of being hunted
iv. for sheer pleasure
b. The figure of speech used in the above lines is
i. simile
ii. metaphor
iii. transferred epithet
iv. personification
c. The tone of the poet in the above lines is
i. frightened
ii. sympathetic
iii. angry
iv. respectful
2. Wi", *a+d&es a+d wi", &a+"er+s
T,rowi+4 4ia+" s*or%io+ s,adows
O+ ",e s0+ (aked wa&&s
T,e: sear*,ed 2or ,i#: ,e was +o" 2o0+d.
a.The search was conducted by
i. the family
ii. the villagers
iii. the flies
iv. the peasants
b. The scorpion is being searched as
i. it has stung the father
ii. it has spoiled the rice
iii. it has stung the mother
iv. its movements would cause more pain
c. The second line indicates that
&. the observer is frightened by the activities
). the shadow of the scorpion was falling on the walls
*. there was a giant scorpion hiding somewhere
+. the observer1s fear of scorpions ma'es him visualiEe shadows
.. T,e: said@ a+d ",e: sa" aro0+d
O+ ",e 2&oor wi", #: #o",er i+ ",e *e+"re@
",e %ea*e o2 0+ders"a+di+4 o+ ea*, 2a*e.
a. PThey1 have been expressing their
i. 0esires
ii. Iiews
iii. 3eliefs
iv. ,ishes
b. PThey1 sat around the mother as
i. She is dying
ii. She is very angry
iii. She is suffering
iv. She is narrating
c. PThey1 are at peace and have accepted that
i. The poison will ta'e its own course of time
ii. The mother will die soon
iii. They have done everything that is to be done
iv. The mother had to suffer to pay for her sins
/. I wa"*,ed ",e 2&a#e 2eedi+4 o+ #: #o",er.
I wa"*,ed ",e ,o&: #a+ %er2or# ,is ri"es
"o "a#e ",e %oiso+ wi", a+ i+*a+"a"io+.
A2"er "we+": ,o0rs
I" &os" i"s s"i+4
a. The sting has been flamed to
i. Lill the effect of the poison
ii. 9a'e her go through more pain
iii. Teach her a lesson
iv. Geed her properly
b. taming the poison refers to
i. reducing the effect of the poison
ii. tying the scorpion with a string
iii. taming the scorpion
iv. 'illing the scorpion
c. the last line means that
i. the scorpion lost its stings in the rain
ii. the pain of the sting was assuaged
iii. the father burned the stings
iv. the scorpion too' twenty hours to lose its stings
II S,or" A+swer 80es"io+s i+ a(o0" .09/0 words $ 2 #arks ea*,)

3ased on your reading of the poem answer the following questions briefly(
&. ,hat is the poet1s attitude towards the scorpionM
). ,ho is the P:vil -ne1 in the poemM
*. 2ow is the scorpion both diabolic and vulnerableM
+. ,hat did the villagers want to achieve by repeating 4od1s nameM ,hyM
". ,hy were the villagers unable to find the scorpionM
!. 0o you thin' the Iillagers1 beliefs were genuineM substantiate
@. 0o you thin' the observer in the poem is comfortable with the interest of the
villagersM
%. ,hy do you thin' the father went against his principlesM
F. ,hat does the father1s action reflect about himM
&#. 2ow did the mother react to the stingM
&&. 2ow is the line Pafter twenty hours it lost its sting1 an anticlimax to all the rituals,
prayers, powders and mixturesM
&). ,hat do you thin' was the impact of that night on the poetM
III Lo+4 a+swer =0es"io+s i+ a(o0" -009-50 words: < #arks
&. The poem PNight of the Scorpion1 gives an insight into the beliefs of the people.
<omment
). Though the poet only describes the happenings of the night yet his reactions are
visible in the poem. :lucidate
*. ,hy were the father1s reactions contrary to his beliefsM
+. 3ased on your understanding of the narrator in the poem, write a page in his
diary about the night1s happening.
". As the father of the narrator write a letter to your friend about hoe the incident
affected you.
!. <omment on the appropriateness of the title.
ODE TO T7E WEST WIND B3 1.B. S7ELLE3
I+"rod0*"io+
5 -de to the ,est ,ind7 is a symbolic poem written by the great Romantic poet /.3.
Shelley. Shelley sees the west wind as a symbol of destruction and preservation. The
,est ,ind indicates the end of autumn and the coming of winter. Shelley observes the
changing of the weather and its effects on the environment. 2is aim is to awa'en
man'ind to the =oy of the coming future which would remove human anguish and
sorrow.
6n ?-de to the ,est ,ind,? Na"0re is grander and more powerful than Ma+ can hope to be.
The natural world is especially powerful because it contains elements li'e the ,est ,ind and
the Spring ,ind, which can travel invisibly across the globe, affecting every cloud, leaf, and
wave as they go. 9an may be able to increase his status by allowing Nature to channel itself
through him.
The ,est ,ind in Shelley1s ode is depicted as an autumnal wind, preparing the world for
winter. As a result, the poem is filled with images of death and decay, reminders of both
+a"0ra& a+d ,0#a+ #or"a&i":. The spea'er hopes that the death of one world will be
inevitably followed by a new rebirth and a new spring.
As the spea'er of ?-de to the ,est ,ind? feels himself waning and decaying, he begs the
wind to use him as an instrument, inhabit him, distribute his ideas, or prophesy through his
mouth. 2e hopes "o "ra+s2or# himself by uniting his own spirit with the larger ?Spirit? of
the ,est ,ind and of Nature itself.
Gis" o2 ",e %oe#
1ar" I $Li+es-9-/)
Shelley begins his poem by addressing the ,ild ,est wind of autumn. The dead
leaves are scattered and are driven li'e ghosts who run away from the enchanter
The leaves are yellow, blac', pale and$ hectic red. Through the imagery of pestilence
Kstric'en multitudes, Shelley ma'es us aware that he is addressing$more than a pile of
leaves. 2e is addressing leaves falling in great numbers ;multitudes.. The leaves are
diseased and withered. They have lived their lives on the trees and are falling.
The ,est ,ind scatters the seeds far and near and covers them with dust. The wind
is li'e a chariot that carries leaves and seeds to the cold earth. The seeds remain li'e
dead bodies ;corpse. in their grave till the advent of spring. Spring announces its
coming by blowing the trumpet. ,hen the east wind blows, the winged seeds
sprout out with sweet buds. They fill the atmosphere with beautiful hues and smell.
The ,est ,ind destroys and uproots plants and at the same time carries the seeds that
sprout in spring. The ,est ,ind is thus the destroyer and preserver. The poet calls out
to the ,est ,ind to hear his message.
1ar" II $Li+es -592A)
Shelley describes the activities of the ,est ,ind in the air. The ,est ,ind carries on
its surface loose clouds which seem to have fallen from the boughs of heaven and
ocean $$$ branches of a tree whose roots are in the ocean and branches are in the s'y
The loose clouds, for example, are probably cirrus clouds, harbingers ;?angels?. of
rain.
As the leaves have been shed from boughs, these clouds have been sha'en from the
heavier cloud masses, or ?boughs of 2eaven and -cean?
6n >atin, ?cirrus? means ?curl? or ?loc' of hair?H it is thus appropriate that these
clouds resemble a 9aenadCs ?bright hair? and are referred to as the ?loc's of the
approaching storm?
9aenads who are worshippers of the Roman god of wine and vegetation, 3acchus ;in
4ree' mythology, 0ionysus. were wild, dancing women with streaming hair.
2ere, the spea'er compares the appearance of the cirrus clouds strea'ed across the
horiEon with the maenadsC blown tresses.
This image seems especially appropriate in that 3acchus0ionysus is associated with
the natural world and the wind and clouds are primary elements of nature
The ,est ,ind is the dirge ;funeral song. of the dying year. As the year draws to a
close, nature prepares for its funeral. The coming night is described as a sepulchre, a
burial tomb that will be mar'ed by lightning and hail from a storm. This last day will
end in dar'ness under storm clouds. Shelley requests the ,est ,ind to hear him.
1ar" III $Li+es 2H9/2)
Shelley says that the ,est ,ind awa'ens the blue 9editerranean from his summer
dreams. The 9editerranean sea is pictured as$ being smooth and tranquil, sleeping
quietly and peacefully alongside the old 6talian town of 3aiae .
The 9editerranean sea is dreaming of old palaces and towers which once stood on
shores but is sha'en up from his sleep by the ,est ,ind which brings in an autumn
storm.
The shores of the 9editerranean are now overgrown with moss ;small green or
yellow plants grown on wet surfaces. and flowers. Their sight or smell overpowers
the senses of the poet.
,hen the ,est ,ind blows over the Atlantic -cean the waves rise and separate
themselves ;cleave. and prepare a sort of passage ;chasms. for the ,est ,ind.
0eep down in the sea1s bottom, the vegetation ;sapless foliage of the ocean. is
drained of colour when it hears the sound of the ,est ,ind =ust as a person turns pale
with fear and it despoils itself.
Shelley begs the ,est ,ind for a third time that it should hear him and appeals to it
to listen to his message.
1ar" ID $Li+es/.95<)
Shelley now establishes a direct lin' with the west wind. As a boy, Shelley was swift,
proud and uncontrollable li'e the ,est ,ind but now the misfortunes of life have
crushed him.
Now Shelley finds himself chained and bound but there was a time when he was li'e
the ,est ,ind$$$uncontrollable. Shelley wishes he could be free of the life1s burdens.
2e says that if he were a dead leaf the wind could bear him. 6f he were a cloud,
he would fly with the wind. 6f he were a wave, it could push him
.6f he were as a boy the companion of the wind1s wandering over heaven, then he
would never have needed to pray to the wind and$ invo'e it because then they both
would run speedily together. 2e pleads with the wind to lift him as a wave, a leaf a
cloud.
1ar" D $Li+es 5F9 F0)
Shelley as's the wind to ma'e him his lyre =ust as the forest is its lyre ; a 'ind of
musical instrument with strings fixed in a B$shaped frame.. Shelley is losing his
leaves li'e the forest but he has the strength to give a message. The tumult and
disturbance of the powerful harmonies of the ,est ,ind will produce music from
within him. This music will sound sweet though it will have a sad and autumnal tone.
2e requests the ,est ,ind to become one with him. 2e as's the wind to drive his
dead thoughts ;Shelley1s thought will bring about regeneration among man'ind li'e
the scattered seeds which sprout up during the spring season. across the universe, li'e
withered leaves, to quic'en a new birth.
2e requests the west wind to be his own spirit. As a breeEe might ignite a glowing
coal, the spea'er as's for the ,est ,ind to breathe new life into him and his poetic
art. 2e as's the wind, by the chanting of this verse, to scatter$ his words among
man'ind
The ,est ,ind is the trumpet of the prophecy of the golden period of man'ind. Dust
as winter ;distress. is followed by spring ;=oy. good times cannot be far behind
sorrow. ,here there is sorrow, there is =oy or happinessH where there is dar'ness,
there is light. Dust li'e cycle of seasons, we have the cycle of life. ,hen one reaches
low ebb in life, one becomes positive again. Shelley has good hopes for man'ind.
Shelley1s poem thus ends on a note of expectancy.
M0&"i%&e C,oi*e 80es"io+s. Ea*, =0es"io+ *arries . #arks a+d ea*, s0(%ar" is 2or -
#ark.
Read ",e e!"ra*"s a+d a+swer ",e 2o&&owi+4 =0es"io+s (: *,oosi+4 ",e #os" a%%ro%ria"e
o%"io+.
-. 3e&&ow@ a+d (&a*k@ a+d %a&e@ a+d ,e*"i* red@
1es"i&e+*e9s"ri*ke+ #0&"i"0des ",o0
W,o *,ario"es" "o ",eir dark wi+"r: (ed
a. 9ultitudes here refers to
;i..heaps of dead and diseased leaves
;ii..crowd
;iii..varieties of leaves and flowers
;iv. heaps of coloured leaves
b.The expression chariotest to their dar' wintry bed means the
;i..the wind burying seeds in the ground is li'e a charioteer driving corpses to their graves.
;ii. the wind is li'e a charioteer driving dead leaves to their graves.
;iii..the wind blows mildly li'e a charioteer driving corpses to their graves
;iv..the wind destroys everything and drives corpses to their graves.
c. The spea'er appeals to the ,est ,ind
;i..a second time.
;ii..a third time
;iii..for the first time
;iv. for the last time
2. T,o0 Dir4e
O2 ",e d:i+4 :ear@ "o w,i*, ",is *&osi+4 +i4,"
Wi&& (e ",e do#e o2 a vas" se%0&*,re@
Da0&"ed wi", a&& ",: *o+4re4a"ed #i4,"
O2 va%o0rs@ 2ro# w,ose so&id a"#os%,ere
B&a*k rai+ a+d 2ire a+d ,ai& wi&& (0rs": O ,earR
a. A 0irge means a
;i..melancholic song
;ii..funeral song
;iii..carol
;iv..happy song
b. The closing night is
;i.. li'e a dar'$domed tomb constructed of thunderclouds, lightning, and rain.
;ii..li'e a dirge preparing nature for the funeral.
;iii.. li'e a dar'$domed tomb constructed of dead leaves and winged seeds.
;iv..li'e a sepulchre. of rain and leaves.
c. The spea'er through the given lines describes the
;i. beauty of the ,est ,ind
;ii. the might of the ,est ,ind
;iii.wea'ness of the poet
;iv. cause of rain
.. A+d saw i+ s&ee% o&d %a&a*es a+d "owers
80iveri+4 wi",i+ ",e wave;s i+"e+ser da:@
A&& over4row+ wi", aQ0re #oss@ a+d 2&owers
So swee"@ ",e se+se 2ai+"s %i*"0ri+4 ",e#R
a. The 9editerranean sees
;i. in his autumn dreams the old palaces and towers
;ii. in his summertime drowsiness old palaces and towers
;iii. in summer dreams old palaces and castles
;iv .in summer dreams new palaces and old towers.
b. The places that are overgrown with moss and flowers are the
i.shores of the 9editerranean sea
ii.shores of the Atlantic ocean
iii.depths of the 9editerranean sea
iv. depths of the Atlantic ocean
c. 9editerranean is wa'en up from his dreams by the
;i. :ast ,ind
;ii. storm
;iii. ,est ,ind
iv. old palaces and towers.
5.Be ",ro04, #: &i%s "o% 0+awake+Cd ear",
T,e "r0#%e" o2 a %ro%,e*: R O Wi+d@
I2 Wi+"er *o#es@ *a+ S%ri+4 (e 2ar (e,i+d B
a. Spring stands for
;i.birth and new life
;ii.death and decay
;iii.total extinction
;iv..destruction
b. The poetic device used in T,e "r0#%e" o2 a %ro%,e*: is
;i..simile
;ii..alliteration
;iii..personification
;iv..metaphor
c. The emotion of the spea'er expressed in the given lines is
;i. pessimism
;ii. optimism
;iii. realism
;iv. humanism
S,or" A+swer 80es"io+s i+ a(o0" .09/0 words $2 #arks ea*,)
&. 2ow is the ,est ,ind both a destroyer and preserverM
). ,hat will happen when all the clouds are gathered by the windM
*. 0escribe the storm and its elements that the ,est ,ind will bring.
+. 0escribe the effect of the ,est ,ind on the blue 9editerranean.
". ,hat was the effect of the ,est ,ind on the Atlantic -ceanM
!. 2ow was the poet li'e the ,est ,ind in his boyhoodM ,hat change has come about in
him with the passage of timeM
@. ,hy does the spea'er describe his own thoughts as ?dead?M
Ialue points(
death can be productive, because it creates an opportunity for new life and rebirth
%. 2ow does the ,est ,ind become the trumpet of a prophecyM
Lo+4 A+swer 80es"io+s i+ a(o0" -009-50 words: < Marks
&. ,hat activities of the ,est ,ind have been described in the poemM
). 2ow does the poem reveal Shelley1s love for libertyM
Ialue points
Shelley appreciates the activities of the ,est ,ind on the land, in the air, and
oceans
identifies his own personality with the ,est ,ind
was swift, proud and uncontrollable li'e the ,est ,ind in his childhood
requests the wind to liberate him from his sufferings, to lift him as a wave, a leaf
or a cloud
implores the ,est ,ind to scatter his thoughts among man'ind
sees the ,est ,ind as the trumpet of a prophecy$$$$ prophecy that man would be
liberated from his sufferings.
*. ,hat appeal does the poet ma'e to the ,est ,indM
+. 2ow is the natural world being transformed in ?-de to the ,est ,ind?M 2ow is the
spea'er being transformedM
Ialue points(
,ith reference to the ,ild ,est ,ind and its action -n :arth( a destroyer of
dead leaves and preserver of winged seeds
6n,ater( wa'es 9editerranean, cleaves chasms on Atlantic, the sea plants destroy
themselves.
6nS'y( commotion, loose clouds, effects blac' rain, fire, lightning.
Gire( scattering poet1s spar's so thoughts in the noo' and corner of the world.
Air( the wind itself stri'es every where to destroy and preserve
The spea'er in ?-de to the ,est ,ind? experiences one 'ind of transformation$ his
own gradual decline$ he desires another 'ind of transformation, a fusion with the
powers of nature.
". 6s transformation of the poet a positive experience in this poemM 6s it necessaryM 6f so,
whyM
1oe"i* devi*es i+ SOde "o ",e Wes" Wi+dS
*ist of Si#iles
& .$$$$$$$$$$$$$$$$$$$$$$$$$$$$$$$$$$$$$$$$$$$$$$$$the leaves dead
Are driven, li'e ghosts from an enchanter fleeing
$E!%&. 60s" &ike ",e 4,os"s w,o 2&ee 2ro# a+ e+*,a+"er@ ",e dead &eaves are drive+ (:
",e Wes" Wi+d .T,e &eaves are *o#%ared wi", 4,os"s).
). The winged seeds, where they lie cold and low
:ach li'e a corpse within its grave
$ E!%&. T,e seeds re#ai+ dor#a+" 0+der ",e ear", &ike dead (odies i+ ",eir 4rave "i&&
",e *o#i+4 o2 s%ri+4)
*. 0riving sweet buds li'e floc's to feed in air
$ E!%&. Swee" (0ds o%e+ o0" i+ ",e s%ri+4 a+d ",eir %o&&e+ 4rai+s &ook &ike 2&o*ks o2
s,ee% i+ ",e air)

+. >oose clouds li'e earth1s decaying leaves are shed
Shoo' from the tangled boughs of 2eaven and ocean
$ E!%&. C&o0ds see# "o 2a&& dow+ 2ro# ",e sk: &ike ",e de*a:ed &eaves w,i*, are
s,ed o+ ear",
". >i'e the bright hair uplifted from the head
of some fierce 9aenad
$E!%&. T,e a%%eara+*e o2 ",e *&o0ds s%readi+4 i+ *0r&s a+d swir&s is *o#%ared "o
",e (ri4," 0%&i2"ed ,air o2 Mae+ad@ a 2e#a&e 2o&&ower o2 Ba**,0s@ ",e Greek God
o2 wi+e a+d 2er"i&i":)
!. -ne too li'e thee$ tameless swift and proud
$ E!%&. T,e %oe" ide+"i2ies ,i#se&2 wi", ",e Wes" Wi+d.I+ ,is (o:,ood da:s
S,e&&e: was 0+*o+"ro&&a(&e@ swi2" a+d %ro0d &ike ",e Wes" Wi+d
@. 0rive my dead thoughts over the universe
>i'e withered leaves to quic'en a new birth
$ E!%&.T,e wi",ered &eaves are drive+ awa: (: ",e Wes" Wi+d.Si#i&ar&:
S,e&&e:Cs ",o04,"s are "o (e drive+ a*ross ",e 0+iverse.
%. And by the incantation of this verse
Scatter, as from an unextinguished hearth
Ashes and spar's, my words among man'ind
$ E!%&. 60s" &ike ",e as,es a+d s%arks are s*a""ered 2ro# a ,ear",@
S,e&&e:Cs words s,o0&d (e s*a""ered a#o+4 #a+ki+d)
Lis" o2 Me"a%,ors
-. Dark wi+"r: (ed
$ E!%&. T,e ear", is *o#%ared "o a (ed w,ere ",e &eaves wi&& s&ee% ",ro04,
wi+"er.)
2. Wi+4ed seeds
$ E!%&. Seeds w,i*, are dis%ersed (: ",e wi+d ,ave %a%er: %ro?e*"io+s w,i*,
&ook &ike wi+4s)
.. 7er *&ario+
$ E!%&. T,e w,is"&i+4 o2 ",e wi+d is *o#%ared "o ",e so0+d o2 ",e "r0#%e")

/. Ta+4&ed (o04,s o2 7eave+ a+d O*ea+
$E!%&. C&o0ds are &ike ",e 2o&ia4e o2 a "ree wi", rai+ a+d &i4,"+i+4 &ooki+4 &ike
",e "r0+k a+d (ra+*,es o2 a "ree. T,is "ree see#s "o ,ave roo"s i+ ",e o*ea+ a+d
(ra+*,es i+ ",e sk:.)
5. T,or+s o2 &i2e
$E!%&. Co#%ariso+ o2 ",e di22i*0&"ies o2 &i2e wi", ",e ",or+s o2 a %&a+".)
<. Tr0#%e" o2 a %ro%,e*:
$ E!%&. Wes" Wi+d see#s "o (e a++o0+*i+4 ",a" 4ood ",i+4s wi&& soo+ ,a%%e+
?0s" as a "r0#%e" a++o0+*es a ,a%%: eve+".)
1erso+i2i*a"io+
-. Wes" Wi+d $des"ro:er a+d %reserver K s:#(o& o2 (o", dea", a+d re9(ir",)
2. T,o0 (rea", o2 A0"0#+Cs (ei+4
.. 7ear@ O, ,earR
/. T,e (&0e Medi"erra+ea+ w,ere ,e &a:
L0&&Cd (: ",e *oi& o2 ,is *r:s"a&&i+e s"rea#s
5. T,: voi*e
;E!%.-95 .I+ a&& ",e a(ove %,rases or e!%ressio+s@ ",e %oe" addresses ",e Wes"
Wi+dIMedi"erra+ea+ as ",o04, i" is a &ivi+4 e+"i":.)
Me"a%,orI1erso+i2i*a"io+
-. 1es"i&e+*e9s"ri*ke+ #0&"i"0des
; :xpl. 9etaphor$ <omparison of the diseased leaves with the people struc' by
disease.
/ersonification K Shelley is addressing more than a pile of leaves =ust as he addresses
a person. 59ultitudes7 is associated with people
2. Lo*ks o2 ",e a%%roa*,i+4 s"or#
; :xpl. 9etaphor$ <omparison of the stormy clouds with the uplifted hair of 9aenad.
/ersonification$ 5>oc's7 ;hair. used with reference to the loc's of a person.
.. A+4e&s o2 rai+ a+d &i4,"+i+4
; :xpl. 9etaphor$ <omparison of the stormy clouds with the angels ;messengers of
4od..
/ersonification$ Angels ;messengers of 4od. are li'e living beings
T,e Fro4 a+d ",e Ni4,"i+4a&e
B: Dikra# Se",
I+"rod0*"io+
Dikra# Se",C is a well$'nown poet in 6ndia. 2e was born in &F") in <alcutta ;Lol'ata.. The
poem T,e Fro4 a+d ",e Ni4,"i+4a&e is from his boo' of poems called Beas"&: "a&es 2ro#
7ere a+d T,ere ;&FF&.. As the title ma'es it explicit that the poems are based on narratives
from various parts of the world, 4reece, <hina, 6ndia and B'rain not to mention the fantasy
world of 4up. The poems highlight the tension between good and evil. These tales in verse
are simple to read and are li'e the fables, teaching important lessons of life. The poem under
study portrays the cunning frog and the Naave Nightingale who pays the price of her
innocence by losing her purity, her originality and ultimately her life.
Gis" o2 ",e 1oe#
> &$*+
-nce upon a time a frog living in 3ingle 3og ;marshy area. croa'ed away in his
unpleasant voice all night.
The other creatures hated his voice and did everything in their power to ma'e him
quiet$threw stic's, stones, prayed, insulted, complained , threw bric's but all in vain.
Nothing could stop the frog from minstrel ling ;singing. all night in his harsh voice.
-ne night his renditions were interrupted when a Nightingale perched itself on the
Sumac Tree and burst into the most melodious song ever heard under the Sumac Tree.
The other creatures of the 3ingle 3og unused to such sweet voice heard her
mesmeriEed by the beauty of the song.
<reatures from all over the forest were drawn towards the bird1s song and as'ed for a
repeat performance. Toads, Tiddlers, teals 0uc's and 2erons cheered her song.
Not used to praise the Nightingale 'ept giving performance the whole night.
Next night when the nightingale got ready for the performance ; She had all the
mannerisms of an artist$clearing of throat, ad=usting the mi'e, her appearance..
>ine *"$!#
The Grog made his presence felt, introduced himself as the owner of the tree, 'nown
also for his deep voice and as a columnist for the 3og Trumpet;9agaEine.
2ighly impressed by the Grog1s credentials, the Nightingale was too eager to hear
about his opinion.
The Grog without mincing any words criticiEed her art as a professional.
Nightingale being too naave believed and defended herself by claiming to be original.
The Grog brushed her claimed aside and told her in no uncertain terms that without
his guidance she would be nobody.
>ine !&$%+
Bnable to hide her excitement at her good luc', the nightingale accepted the honour
of being trained by a musician of repute8
The frog made it very clear that he would be charging a modest fee but assuring her
that she would be able to pay bac'.
So the Nightingale began her career as a performer$infused with confidence, filled
with a Eest for performance and inspired by the fawning, sang and became a sensation
overnight.
The magical quality of her song attracted creatures of the 3ingle 3og from far and
wide
The opportunist frog cashed on the crowd and started charging admission fee.
The arrogant and heartless frog never let an opportunity to ma'e the Nightingale
realiEe who was in control.
,ithout a care for the bad weather, he would order her to practice, push her to her
limit
6n his role of a patron he became a hard tas' master, pushing the Nightingale to the
verge of collapse.
>ine %"$&&#
0ay after 0ay this went on, the Nightingale now sleep deprived and exhausted went
on performing.
The Sumac tree of 3ingle 3og was humbled ;honoured.by the presence of the elite of
the 3ingle 3og ;/oet is being sarcastic about people who come to see musical
performance only for the sa'e of showing off =ewels, without little bit of interest in
the art.
:mploying the pun on words the poet lists few royal titles$0u'e, <ount, <ardinal,
9allord, :arl who visited the Sumac tree to hear the Nightingale perform.
-n one hand the Grog was happy as his business was so profitable on the other hand
he was =ealous of the success of the Nightingale.
2is greed made him unreasonable, everyday he would point out her flaws, criticiEed
her singing, advised her on the techniqueH remind her of her goal of being successful
and how much she owed him.
>ine &&&$&+#
This constant badgering by the Grog robbed the Nightingale of her creativity. She
started sounding bored, uninspired and no amount of mannerism could revive the
audience.
The cash collection started falling and the disinterested applause of few who still
were coming gave her no happiness.
6t made her sorrowful and depressed.
The frog was livid as his business was getting affected. 2e lost his temper and
became cruel. As'ed her to follow trends and give the people what they want.
The Nightingale tried very hard, collected her wits about her, performed but alas she
burst a vein and collapsed.
The calculative Grog quic'ly washed his hands off the Nightingale. 2e pointed out
how the Nightingale was a stupid creature, he tried to teach her but she was very
nervous, easy on the ear and gullible.
2e became a little braEen as he pointed out the 3ird should have 'nown that one1s
song should be one1s own ;meaning $Oou can1t let other people dictate your life. She
should have 'nown when to say no..
The manipulative Grog not only cleared himself of all responsibility of the 3ird1s
death but also made it appear that the bird was responsible for her death.
2e pro=ects himself as a confident, practical and wise creature who sings with a blan 8
2ence the Grog resumed his position in the 3og and continued to blare out in his
harsh and unpleasant voice undisputed and unrivalled.
T,e #ora& o2 ",e %oe# is that one has to pay for one1s gullibility. 3eing inspired and
influenced by someone un'nown and stranger is indeed a foolish wor'. The
nightingale had to suffer for her mis=udgment. 6f our self image is based on what
others ma'e us believe we are, we will always be misguided. 6t is important to 'now
your own worth..
The NightingaleCs gullible servile nature results in her tragic death. The pompous,
scheming frog plans her destruction after winning her trust. >ac' of confidence and
good =udgement of character can bring misery from which there is no escape.
Fro4:
2e has been portrayed as arrogant, pompous, haughty, patroniEing,
opportunist, condescending and boastful.
2e is used to his position of the only singer of the 3ingle 3og
The entry of the melodious nightingale is an eye opener
3eing practical and worldly wise he uses his position to influence her
2e impresses upon her his superiority by discussing her art condescendingly
2e manipulates the situation to his advantage, starts ma'ing profit by charging
admission fee.
2is haughtiness is evident when he doesn1t tolerate any slips on the
nightingale1s part
2e gradually wor's on the nightingale1s wea'ness$ of being rich and
famous;downfall of many.
A smooth tal'er brushes away the blame of nightingale1s death by
philosophiEing it that Pyour song must be your own1

Ni4,"i+4a&e:
She is portrayed as the innocent, naave, gullible, polite, unsure, timid, shy and
nervous type
The taste of appreciation enhances her self esteem ma'ing her gullible
The desire to impress ma'es her blind to the faults of the frog
2er purity, originality and piousness gets shadowed by the greed for status
and commercial success
Trapped in the vicious circle of success pushes herself to a point of exhaustion
Addicted to her status, unable to get out of the clutches of the frog she tries$
Alas the misguided efforts result in her death
I M0&"i%&e C,oi*e 80es"io+s $ea*, =0es"io+ *arr: . #arks a+d ea*, s0( %ar" is 2or 0-
#ark)
Based o+ :o0r 0+ders"a+di+4 o2 ",e %oe#@ *,oose ",e *orre*" o%"io+s 2or ",e 2o&&owi+4:
-. So ",e Ni4,"i+4a&e o+*e #ore@
80i"e 0+0sed "o s0*, a%%&a0se@
Sa+4 "i&& daw+ wi",o0" a %a0se.
a. The nightingale was not used to so much appreciation as
i. She had had no audience before
ii. She was a nobody so far
iii. This was her first performance
iv. She was unaware of her musical talent
b. The creatures of the 3ingle 3og requested her for another song as
i. They were tired of listening to the unpleasant notes of the frog
ii. They were all free and wanted to be entertained
iii. 2er musical notes were a welcome change
iv. The frog was on a holiday
c. The nightingale1s song made the frog thin' about
i. ,ays of insulting the nightingale
ii. 2ow ungrateful were the creatures of 3ingle 3og
iii. 2ow to ta'e advantage of the situation
iv. ,ays of teaching a lesson to the creatures of 3ingle 3og
2. T,a" a *ri"i* o2 s0*, +o"e
7ad dis*0ssed ,er ar" a+d ",roa"
TI do+C" ",i+k ",e so+4 is divi+e@
B0"9 o,@ we&& a" &eas" i"Cs #i+e7
a. The Pcritic1s claim for his reputation was that he
i. ,as 'nown for his singing
ii. ,as owner of the tree
iii. ,rites for the bog trumpet
iv. Reflects his arrogance
b. The nightingale is apologetic about her song as
i. The frog did not li'e it
ii. 6t was copied from some where
iii. The frog had pointed out the technical flaws
iv. The nightingale lac'ed confidence
c. 2er claim that Pat least it1s mine7 implies that
i. The song was original
ii. She was proud of it
iii. She wanted to show off
iv. She was confident
.. TB0" I *a+C" sii+4 i+ ",is wea",erU
T*o#e #: dear9weC&& si+4 "o4e",er.99999
9999So ",e 2ro4 a+d +i4,"i+4a&e
6o0r+e:ed 0% a+d dow+ ",e s*a&e
a. The nightingale resists the frog1s suggestion as
i. 6t was very hot
ii. 6t was raining
iii. 6t was very humid
iv. 6t was very cold
b. The way frog brushes away her excuses reflects his
i. Arrogance
ii. 2ard tas' master
iii. Servile
iv. <lever
c. Dourneyed up and down the scale1 means
i. Travel in 3ngle 3og
ii. Travelling up and down the tree
iii. Singing on different notes
iv. /opularity ratings
/. Da: (: da: ",e +i4,"i+4a&e
Grew #ore sorrow20& a+d %a&e
Ni4," o+ Ni4," ,er "ired so+49999
Ti&& ",e (irds a+d ",e (eas"s 4rew "ired
a. The nightingale was growng sad and pale as
i. The frog was always criticiEing her
ii. 2e was ma'ing her practice for long hours
iii. She was afraid of displeasing her
iv. She felt trapped
b. The birds and the beasts grew tired of
i. ,aiting for the nightingale
ii. /aying the frog
iii. Bn inspired songs of the nightingale
iv. Nightingale1s repetitive performance
c. The effect of nightingale1s sub standard performance was
i. The creatures started going to the frog1s performance
ii. The creatures did not appreciate her performance
iii. The gross collection at the tic'et office fell down
iv. The nightingale stopped singing
i. .
II S,or" a+swer =0es"io+s i+ a(o0" .09/0 words $2 #arks ea*,)
3ased on your reading of the poem answer the following questions brieflyH
&. 6n spite of the frog1s crass cacophony how could he emerge as the winnerM
). ,hat chance incident brought some respite to the creatures of 3ingle 3ogM
*. 2ow did the frog en cash on nightingale1s popularityM
+. ,hat characteristics helped the frog to overpower the NightingaleM
". ,hat piece of advice did the frog give to nightingale to improvise her singingM
!. ,hy do you thin' the nightingale lost her appeal for the massesM
@. 2ow did the celebrity status affect the nightingaleM
%. ,hat do you thin' the nightingale was afraid ofM
F. 6n what sense was the nightingale a stupid creatureM
&#. 2ow did the frog become the unrivalled 'ing of the 3ogM
III Lo+4 a+swer =0es"io+s i+ a(o0" -009-50 words $ < #arks)
&. 0o you thin' nightingale was responsible for her own deathM Substantiate your answer
with examples from the poem
). Ta'ing the nightingale as a protbgbe was a ploy employed by the frog to save his
position. <omment
*. The poem is an example of how professional rivalry can play with the lives of people.
:laborate
+. 3ring out the symbolism as used by the poet in the poem 5The frog and the
Nightingale7
". The nightingale had to pay heavily for her innocence and gullibility. :lucidate

MIRROR
B3 ' SL3DIA 1LAT7
6NTR-0B<T6-N( The poem 59irror1 is an autobiographical account of mirror,
highlighting its qualities. 6t reflects how a woman is disappointed and frustrated to see her
gradually fading beauty and youth. 6t symboliEes the eternal truth of old age and loss of
beauty in the course of life. The mirror is real and shining. 6t is unbiased. 6t shows what
comes to it. 6t is a monologue of the protagonist 9irror who is free of any pre conceptions
and pre=udices. 6t is not distorted by emotions of love or hatred as human beings do.
46ST -G T2: /-:9(
>ine &$+
The mirror in the poem symboliEes the eternal truth of life. 6t is real and shining. 6t is
silver in color and appears perfectly smooth.
6t does not =udge other with pre$conceived ideas li'e the human beings.
6t does not have any pre$decided ideas and doesn1t get affected by love or disli'e.
6t has no feelings, emotions and preferences. 6t reflects exact what comes to it. 6t
shows the person1s accurate image from any 'ind of distortions.
Since mirror is exact, truthful and ob=ective, it may appear as unfeeling and cruel
though it is not so.
>ine "$F
The mirror says that it is above any 'ind of personal feeling. 6t is li'e 4od, who sees
the truth and views everything from all corners of the world. 6t is something mystic
and divine above the ordinary and the worldly. So, li'e 4od, the mirror loo's at
everything with equal eye. 9ost of the time it 'eeps thin'ing about the opposite wall
which is pin' and has spec'les on it. 6t has grown passionate about it and now it has
begun to thin' of the wall as a part of its heart, here it means that it has become an
integral part of its being, consciousness and existence. The wall flic'ers i.e. its view
sha'es and dims with changes in the shades of light. The mirror1s view of the wall is
interrupted by the people1s faces and the dar'ness when they stand between the
mirror and the wall.
>ine &#$&*
The mirror says that sometimes it is in the form of a la'e as when the woman loo's
into the mirror, the memories of life flash across her mind as both are very deep ;la'e
and mirror. revealing her true identity as a person. She attempts to see her flaws
behind the dar'ness. The illusion created by the reflection of candles and the moon
ma'es her appear beautiful and different from what she really is ;old and ugly.. 3ut
the mirror reflects her image truthfully without any bias.
>ines &+$&%
The true ;Bgly and old. image as reflected by the mirror ma'es the woman
Pfrustrated and depressed. She cries and throws her hands in agitation and despair. She is
ageing but it is difficult for her to accept the natural process of ageing. She comes and
stands in front of the mirror every morningH it is her face that replaces the dar'ness. She
misses her youth and beauty. She is no longer charming as she was in her youth. She feels
insecure and insignificant. She feels, she has become very terribly ugly. The loss of her
youth and beauty ma'es her behave li'e a fish out of water or a terrible fish trapped in the
pond of time.
S64N6G6<AN<: -G T2: T6T>:
The poem deals with the varied qualities of mirror K exactness, truthfulness and its
relationship with the human beings ;here the old woman. for years together. The whole poem
revolves around the mirror. 2ence, the title is quite apt and appropriate.
The poem conveys strong message to the humans$ rather than living in the world of
illusion, we need to change our thin'ing and attitude. ,e need to learn to face and accept the
reality of life without indulging in too much of self$pity.
R2O9: S<2:9:
The poem doesn1t have any a rhyming scheme. 6t is written as a free verse.
/-:T6< 0:I6<:
&. /ersonification( The poetess has made an inanimate ob=ect K mirror, as a living ob=ect.
:x. 6t is truthful as an honest person, :x( She turns to those liars, the candles, and the moon.
). 9etaphor
9irror is the la'e. 6t has the common features of a la'e.
:xH Now 6 am a la'e.
:x( ,hatever you see, 6 swallow immediately.
M0&"i%&e C,oi*e 80es"io+s $Ea*, =0es"io+ *arries . #arks a+d ea*, s0( %ar" is 2or 0-
#ark)
;6. 6 am silver and exact, 6 have no preconceptions. ,hatever 6 see 6 swallow
immediately =ust as it is unmisted by love or hatred. 6 am not cruel, only truthful ^^^^
;a. 5Silver and exact1 in the extract mean
;i. 9ade of unalloyed silver
;ii. Iery expensive and decorative
;iii. Absolutely river colored
;iv. <lear and accurate
;b. 56 have no preconcepts7 means mirror
;i. has no state and outdated ideas
;ii. is very imperfect
;iii. has no pre conceived ideas
;iv. reflects the image with a bias
;c. 5Bnmisted by love or disli'e1 refers
;i. to mirror1s love and disli'e for the old lady
;ii. to impartial reflection of the old lady
;iii. to love and disli'e towards the lady
iv. partial reflection of the old lady1s image
;66. The eye of a little 4od, four$cornered. 9ost of the time 6 meditate on the opposite all.
6t is pin', with spec'lesH 6 have loo'ed at it long 6 thin' it is a part of my heart.
;a. The literal meaning of the expression Pfour cornered1 as used in the poem is(
;i. in the eyes of 4od everybody is equal
;ii. mirror is unbiased
;iii. it is un partial
;iv. the mirror is rectangular and it has four corners
;b. The expression( Ppin' spec'les1 refers to
;i. a lady with a healthy pin' face and frec'les
;ii. spots made on the mirror with pin' paint
;iii. the mirror
;iv. the opposite wall that is pin'
;c. The expression( P6 meditate on the opposite wall1 mean
;i. the mirror 'eeps loo'ing at the wall
;ii. the woman 'eeps viewing herself in the mirror
;iii. the mirror is in love with the wall
;iv. the image of the wall is reflected in the mirror
;666. Searching my reaches for what she really is, than she turns to those liars, the candles
or the moon. 6 see her bac', and reflect it faithfully.
;a. The lady is reaching for ^^^
;i. her lost ring
;ii. her lost youth and beauty
;iii. her lost friend
;iv. her strength and vitality
;b. ,hat she really is ^^^
;i. She is very beautiful and charming
;ii. The truth that she is no more young and charming
;iii. She has become extremely old and ugly
;iv. She can not hold her youth anymore.
;c. The spea'er is different from those liars
;i. 6t does not show false images
;ii. 6t does not tell lies
;iii. 6t faithful unable those has
;iv. 6t does not mislead the woman
;6I. 6 have loo'ed at it so long 6 thin' it is a part of my heart but it flic'ers faces and
dar'ness separate as over and over.
;i. 61 and it Pin the above extract are^^^
;a. the young girls TTT. the la'e
;b. the mirror TTT the wall
;c. the woman TTT the mirror
;d. the wall T.. the light
;ii. The expression( Pa part of my heart1 means
;a. 6t lives in her heart
;b. 6t is a part of her existence
;c. 6t is very dear to her
;d. 6t is a part of her body.
;iii. The expression( P3ut it flic'ers1 means
;a. The light flic'ers because of the wind
;b. The image in the mirror flic'ers because of the light
;c. The candle flame flic'ers because of the lives
;d. The girl1s image flic'ers in the la'e water
$II) S,or" A+swer 80es"io+s i+ a(o0" .09/0 Words $ 2 #arks ea*,)
3ased on your reading of the poem, answer the following questions briefly. :ach
question carries ) mar's.
;a. ,hat is the significance of the expression Punmisted1 in the first stage of the
poemM
;b. 2ow does the Pmirror1 swallowM ,hat is the poetic device used hereM
;c. The expression K P6 have no pre$concepts Phas a deeper meaning in context to
human beings. :xplain.
;d. ,hat is the other name the mirror calls itself byM ,hy do you1 thin' this
comparison has been madeM
;e. ,hat are the different qualities of mirror highlighted by the poetM
;f. 2ow is the mirror typically different from most of the human beingsM
;g. :xplain$ The mirror li'e a four cornered god and a la'e.
;h. ,hy do you thin' the poet refers to the fish in the last lineM
;i. 6t is not the mirror but the woman herself who is responsible for the pain that
she experiences, comment.
;=. ,hat is personificationM Quote some examples of personification from the
poem K P9irror1.
III) Lo+4 A+swer 80es"io+ i+ a(o0" -009-50 words $< Marks)
Q&. 4ive the characteristics of the mirror as conceived by Slyvia /lath in her
poem 59irror7.
Q). ,hy does the poetess refer to the woman as a 5terrible fish7 in the poem
P9irror1M
Q*. ,hat 'ind of relationship does the woman share with the mirrorM
Q+. The woman in the poem searches for something in the depth of the mirror.
,hat is itM
Q". ,hy has been the mirror called Pa four cornered 4od1 K ,hat are its qualitiesM
T7E RIME OF T7E ANCIENT MARINER
S.T.COLERIDGE
INTRODCTION
The poem is an extract ta'en from <oleridge1s famous poem. 2ere he brings out the torment
and the strong feeling of guilt faced by an ancient mariner ;an old sailor. who, in a moment
of cruelty, 'illed an innocent albatross. 6n order to overcome his pain and guilt, he often
stopped strangers and told them the story of the troubles faced by the crew as a result of his
cruelty.
GIST
/ART &
>6N:S &$)#
An old grey bearded 9ariner with hypotonic, bright eyes stopped a wedding guest, who
along with his two companions, was going to attend a wedding. The wedding guest said that
he was a close relative of the bridegroom and that he had to go as the feast was ready. The
ancient mariner held the young man by his arm but he ;the young man. ob=ected to it angrily
calling him an old mad man. The old sailor held him by the power of his hypnotiEing eyes
and the wedding guest had no choice but to listen to him li'e a small child.
>6N:S )&$+#
The ancient mariner narrated his story that when he was a sailor, their ship sailed southwards
on a bright sunny day. 6t reached the equator where the sun was directly overhead at noon. At
this point, the wedding guest heard the loud music of the bassoon and was frustrated. 2e
visualiEed that the bride must have entered the hall as beautiful as rose and the merry singing
will be around to welcome the bride. The wedding guest stood helpless and annoyed as he
had to listen to the mariner1s story.
>6N:S +&$!#
The ancient mariner continued his story stating that a dreadful storm struc' his ship, pushed
it at high speed towards the south direction. The storm was li'e a hunter chasing its prey ;the
ship. following it closely. The ship was moving fast ma'ing lot of noises as if it was followed
by an enemy. The ship reached a place where there was lot of mist and snow. 6t was
extremely cold as both mist and snow surrounded the ship. The ice was flowing as high as
the ship loo'ing li'e as green as emerald. The snow cliffs created a very sad loo'ing shine, as
there was no life around. The ice crac'ed, growled, howled and roared as it moved heavily,
holding the ship at one place.
>6N:S !&$%&
The crew of the ship was disturbed with the cold weather but it was a great relief for them
when they were eventually greeted by the arrival of an albatross which came through the fog.
6t was welcomed by the sailors. As it flew around the ship for food and play, the ice crac'ed
and split. A good south wind propelled the ship out of the icy region into the sea. The
albatross followed the ship everyday but at one point the ancient mariner in a fit of anger shot
dead the innocent bird with his crossbow. 2e confessed this to the wedding guest.
1ART 2
>6N:S %#$&#"
The south wind continued to propel the ship northwards. The sailor soon realised that he had
done a hellish thing by 'illing the bird that had brought the change in the breeEe. 3ut when
the glorious sun rose after days of mist and snow, they all agreed that he had done a right
thing to 'ill the bird that had earlier brought the fog and the mist. The ship sailed on until the
winds brought it to a silent sea. Suddenly the winds died down and they were once again
stranded in the middle of the sea.
>6N:S &#!$&)&
,hen the breeEe stopped to blow, the sails dropped and the ship was becalmed. 6t was so
quiet that the sailors spo'e only to brea' the silence. All day long the red sun shone in the hot
s'y. 0ay after day for many days there was no breeEe and the ship remained still motionless
li'e a painting. The crew had no water to drin' in spite of the vast sea around them. :ven the
boats of the ship began to shrin'.
>6N:S &)#$&+#
The condition of the crew was pathetic as the sea loo'ed terrible. Soon the bright and slimy
creatures crawled out of it and wal'ed on the surface. At night, the water loo'ed li'e oil of
witch ma'ing it change its colours. The sailors had no peace even in their ship and were
constantly hunted by the presence of the dead Albatross1s spirit. 0ay after day they had no
water to drin', their tongues dried up and they were unable to even spea'. The guilt of 'illing
held the mariner responsible for their woes and a constant reminder of 'illing an innocent
bird, the sailors hung the dead albatross around his nec'.
1OETIC DEDICE $ A 2ew e!a#%&es)
&. Alliteration
a. 2e holds him with his s'inny hand.
b. ,ater, water everywhere.
). 9etaphor
a. All in a hot and copper s'y.
*. /ersonification
a. And now the ST-R9$3>AST came and he was tyrannous and strong.
b. The sun came up upon the left, out of the sea came he8
+. Simile
a. As green as emerald.
b. And listens li'e a three years child.
MLTI1LE C7OICE 8ESTIONS
Read the following extract and choose the correct option. :ach question carries three mar's
and sub part carries one mar'.
;A. The ship was cheered, the harbour cleared,
9errily did we drop.
3elow the Lir', below the hill,
3elow the lighthouse top.
Questions($
&. The ship was cheered by$
a. the people on the shore
b. the captain of the ship
c. the ancient 9ariner
d. the young sailors
). :xplain( 9:RR6>O 060 ,: 0R-/.
a. The sailors slowed down their ship
b. The sailors anchored their ship
c. The sailors happily sailed away
d. The sailors stopped ma'ing merry and began steering the ship away
*. The poetic device used in line * and +
a. Alliteration
b. Repetition
c. 9etaphor
d. Simile

;3. 2e holds him with his s'inny hand,
PThere was a ship1 quoth he,
P2old off8 Bnhand me, grey$bearded loon8
:ftsoons his hand dropt he.
Questions($
&. 2e holds him with his s'inny hand
a. to test the power of his hand
b. to en=oy his company
c. to narrate to him his story
d. to play a pran' on him
). the second spea'er reacted to the first spea'ers action very
a. angrily
b. respectfully
c. mildly
d. indifferently
*. The old mariner was spea'ing to
a. The bridegroom
b. The bride
c. The wedding guest
d. An old man
;<. And i had done a hellish thing,
And it would wor'1em woe(
Gor all averred, 6 had 'illed the bird
That made the breeEe to blow.
Ah wretch8 Said they, the bird to slay,
That made the breeEe to blow8
Questions($
&. The Phellish thing1 that the spea'er had done was(
a. 2e had 'illed his friend
b. 2e had 'illed the albatross
c. 2e had cheated his fellowmen
d. 2e had drowned a sailor in the sea
). 2ere the spea'er is(
a. The wedding guest
b. The albatross
c. The captain of the ship
d. The Ancient 9ariner
*. 2e made the breeEe to blow(
a. The Ancient 9ariner
b. The Albatross
c. The poet
d. The wedding guest
;0. All in a hot and copper s'y,
The bloody Sun, at noon.
Right up above the mast did stand,
No bigger than the 9oon.
Questions($
&. 2ow did the s'y loo'(
a. 3lue as sea
b. 0ar' as clouds
c. 3right as sun
d. <opper coloured
). The sun was(
a. behind the dar' clouds
b. behind the fog and mist
c. right below the mast of the ship
d. right above the mast of the ship
*. The poetic device used in this line is(
a. 2yperbolic and simile
b. Simile and metaphor
c. Alliteration and personification
d. Assonance and alliteration
a. &:
S,or" A+swer 80es"io+s i+ a(o0" .09/0 words $2 #arks ea*,)
&. 2ow did the Ancient 9ariner stop one of the wedding guestsM ,hat was his
reactionM
). ,hat happened when the Ancient 9ariner1s ship reached the equatorM
*. ,hy did the sailors hang the dead albatross round the nec' of the marinerM
+. 3riefly describe how the ship was caught in the stormM
". 2ow was the arrival of the albatross a good omenM
!. 2ow does the ancient mariner describe the land of mist and snowM
@. ,hat painful experience did the mariner undergo in 5the silent sea7M
%. ,hat impression do you form of the ancient mariner from the poemM
F. ,hat consequences did the mariner have to face as a result of 'illing the albatrossM
Lo+4 A+swer 80es"io+s i+ a(o0" -009-50 words $< Marks)
&. P,ater, water, everywhere........ Nor any drop to drin'1. ,hen does the ancient
mariner ma'e this commentM
). 0escribe the ship1s =ourney from harbour to the southern sea full of mist and
snow.
*. The ancient mariner1s shipmates are a bunch of fic'le minded sailors. <omment.
+. P/unishment does catch up with the sinner although it may be delayed
sometimes1. <omment on this statement with the reference to the poem.
". ,rite a short note on the element of supernaturalism in the poem.
DRAMA
A C7RISTMAS CAROL B3 C7ARLES DIC>ENS
INTRODCTION
CA <hristmas <arolC is one of the most famous novels of <harles 0ic'ens. The novel is set in
the time of <hristmas and celebrates the =oy of CgivingC and ClovingC.
The novel has been re$written and presented as a play. Oou must remember that this was
written as a Radio /lay. CA <hristmas <arolC is a one$act play that is supposed to be CenactedC
over the radio. So, the play depends largely on the language and the sounds.
Gis! o" !he #lay
Pa6e $0-$2
-ut of the dar', cold fog of the street came ScroogeCs nephew. Gred boldly wished his
Bncle Scrooge a merry <hristmas.
?3ah8? said Scrooge, ?humbug8? 2e had a lot to say about <hristmas, and none of it
was good.
Gred tried to reason with him, but it was no use. 2e even invited Scrooge to
<hristmas dinner. ScroogeCs response was very rude. At last Gred left, his <hristmas
spirit still intact
As Gred went out, two other men came in. They had boo's and papers in their hands,
and they bowed to Scrooge. They were collecting money for the poor. They felt that
9r. Scrooge would want to share his wealth.
Scrooge let them 'now that he would not share one penny. The poor could go to =ail
or the wor' houseH he did not care. -ne man protested that many would rather die.
Scrooge said that if they died, they would decrease the surplus population. 6t would
reduce the number of poor people left to beg for help. This response shoc'ed the two
men, and they left.
Scrooge1s underpaid cler', 3ob <ratchit as'ed Scrooge for a day1s off on <hristmas
day. Scrooge saw it as a poor excuse for pic'ing a man1s poc'et every )" th day of
0ecember. Quite unwillingly Scrooge granted his cler'1s wish
1a4es A29A.
That evening, Scrooge sat in his dreary upstairs room and had his cold supper. 2e
slept uneasily, tossing from side to side. Suddenly, Scrooge heard a clan'ing noise, as
if someone were dragging a heavy chain up the stairs to his door.
Through the heavy door and into the room came a ghost8 Startled, Scrooge
recogniEed 9arley, his dead business partner. 9arley wal'ed towards Scrooge, a grey
dim figure of a man .2e sat and tal'ed to Scrooge.
Though Scrooge was afraid, it was hard to tell. 2e did not believe that the ghost was
real.. 2e was sure something he ate had caused this bad dream.
9arley went on to warn him of what was coming. >ove of money would doom him
to ghostly regret. >ove of man'ind was what life was about. 6f Scrooge did not learn
to care about others, his spirit would wander the earth forever. 2is eyes would see all
the chances to do good that he had missed. 2e would never be at peace.
9arleyCs ghost warned Scrooge that he would be haunted by the visit of three spirits(
first at one o1 cloc', second at the stro'e of two and the third at * a m.
,ith those words, he floated out the window and up into the night.
Bnsure of what had happened, Scrooge put himself to bed and fell asleep in an
instant.
1a4es A/9A5
,hen the cloc' struc' one, a light flashed. Scrooge wo'e up. The first spirit had
come. 6t was the 4host of <hristmas /ast, a strange figure li'e a child with white hair
and holding in its hand a sprig of fresh green holly.
Scrooge had a vision of three scenes relating to his past life. 6n the first scene,
Scrooge found himself alone and sad in his school. 2e was crying as there was no one
to love him. This scene reminded Scrooge of the boys who had come singing
<hristmas carols to him and whom he had driven away.
Next he saw his sister Gan who came to him to ta'e him home.. She had been good to
himH she had loved him. Gred was her only child. She had died, but Scrooge saw her
young and alive again.
Then the ghost showed him as a young man brea'ing up with the girl whom he
wanted to marry. She saw that he loved money more than he loved her. Scrooge had
been uncaring then, but now loo'ing on the scene made his heart brea'.
?Spirit, show me no more8? he cried. 2e struggled with the ghost. Suddenly he was in
his bedroom. The ghost vanished. Scrooge fell asleep once more.
1a4es A<9AA
Again, a bright light made Scrooge wa'e up. The second ghost had arrived. 6t was the
Spirit of <hristmas /resent. 2e was a giant, but had a young face.. Scrooge had never
seen anything li'e it. The 4host too' Scrooge first to a church and then to a ship where
the people were happily celebrating <hristmas.
Their next stop was at the <ratchit house. The <ratchits were poor in wealth but rich in
love. Scrooge saw his cler', 3obCs wife, and all their children as they laughed and
cheered.
-f the five children, the youngest one caught ScroogeCs eye. Tiny Tim loo'ed sic'ly.
2e wal'ed with a little crutch. Scrooge heard 3ob say that Tiny Tim hoped the people
saw him in the church, because he was a cripple, and prayed for him on <hristmas day,
Scrooge watched them en=oy <hristmas dinner, small though it was. Scrooge as'ed the
Spirit of <hristmas /resent about Tiny Tim. 2e was upset when told that the child
would die.
The spirit loo'ed at him intently and said, if Tiny Tim died, it would decrease the
surplus population.
The words were a blow to Scrooge. 2ad he really said such an awful thing that dayM 2e
wished he could ta'e it bac'
They left the happy noise of the <ratchit house. Scrooge learned the real meaning of
<hristmas that evening.

1a4es AH9H0
At the stro'e of three in the church cloc', the third 4host appeared. A heavy snow
was falling, blan'eting a sleeping >ondon. 6t was early <hristmas morning.
The 4host showed Scrooge two men conversing about a man who died last night. 2e
left not a friend behind him. The spirit would have shown Scrooge the dead manCs
body, but Scrooge begged to see something else, tenderness connected with death.
They went to the <ratchitsC house. Tiny Tim and 3ob were not there. 9rs. <ratchit
was sewing and crying. Then 3ob came home. 2e had been to the graveyard.
3ob told his wife. That it would have done her good to see how green a place it was..
2e had promised Tim that he would wal' there on a Sunday. Then 3ob covered his
face with his hands and cried
Scrooge could stand it no longer. ,ho was the unloved manM 0id Tiny Tim have to
dieM ,ithout spea'ing, the 4host of <hristmas Oet to <ome led Scrooge to a gloomy
churchyard. The Spirit stood among the graves and pointed at one as'ing Scrooge to
read the inscription on the tombstone. 6t read P:beneEer Scrooge1.
?No Spirit8? cried Scrooge. 2e promised that he would honour <hristmas in his heart,
and he would try and 'eep it alive all the year. 2e would not shut out the lessons that
all three Spirits had taught him.
1a4es AH9H0
2earing the ringing of church bells, Scrooge ran to loo' out his window. 6t was a
golden day. 6t was <hristmas 0ay8
2is first act of love and 'indness was to send the biggest tur'ey he could find to 3ob
<ratchitCs house$ a surprise as 3ob <ratchit would never guess who had sent that gift8
2e then went straight to GredCs house and surprised them all.
Scrooge was warmly welcomed and felt right at home. 2e gave his nephew1s wife a
gift. Scrooge also wished to celebrate <hristmas 0ay with them.
Scrooge became a saint from a sinner. Scrooge was better than his word. To Tiny Tim,
who did N-T die, he was a second father 2e raised his cler'1s salary second too'
Gred into business.
Scrooge is reclaimed. 2e is transformed into a soft hearted and compassionate human
being.
2e became as good a man as anyone had ever 'nown. Grom then on it was always
said that he 'new how to 'eep <hristmas well. No ghosts ever had reason to visit him
again. 2e was truly a changed man, and the more blessed for it.
C,ara*"er Lis"
E(e+eQer S*roo4e
The protagonist, Scrooge is a cold, miserly creditor whose redemption to 'indness and
selflessness forms the arc of A <hristmas <arol. Scrooge represents the Iictorian rich who
neglect the poor and thin' only of their own well$being. As a young boyH Scrooge was
neglected by his peers and, it appears, by his father. The young Scrooge seemed determined
to live only for himself as he aged
6a*o( Mar&e: $ 6n the living world, :beneEer ScroogeCs equally greedy partner. 9arley died
seven years before the narrative opens. 2e appears to Scrooge as a ghost condemned to
wander the world bound in heavy chains. 9arley hopes to save his old partner from suffering
a similar fate
Bo( Cra"*,i"
<ratchit is ScroogeCs overwor'ed employeeH a timid man afraid to stand up to his bossCs
demanding ways. The patriarch of a family poor in wealth but rich in love, he cares
especially dearly for his crippled son, Tiny Tim. <ratchit is a symbol for the Iictorian poor,
good$hearted and hard$wor'ing but unable to climb out the stifling conditions of poverty.
G,os" o2 C,ris"#as 1as"
The first ghost to visit Scrooge, the small, elderly figure represents memory. 2e ta'es
Scrooge on a tour of <hristmases in his past
G,os" o2 C,ris"#as 1rese+"
A giant clad in robes, this ghost has &%## brothers and a life span of one day. 2e represents
celebration and charity. 2e escorts Scrooge on a tour of his contemporariesC holiday
celebrations
G,os" o2 C,ris"#as 3e" To Co#e
This solemn, silent phantom represents death, but also the presents the possibility that the
future is not determined, but opens to the free will of humans. 2e presents Scrooge with an
ominous view of his lonely death.
Fred
ScroogeCs nephew, Gred embodies the =ollity and sharing of <hristmas. 2e refuses to let
ScroogeCs ?3ah8 2umbug8? attitude bring him down, and is over=oyed when his uncle
converts and attends his party.

Ti+: Ti#
<ratchitCs crippled son, Tiny Tim represents the overwhelming goodness of the <hristmas
spirit. Tiny Tim is a highly sentimentaliEed character who 0ic'ens uses to highlight the
tribulations of :nglandCs poor and to elicit sympathy from his middle and upper class
readership.
9elle
A beautiful woman who Scrooge loved deeply when he was a young man. 3elle bro'e off
their engagement after Scrooge became consumed with greed and the lust for wealth. She
later married another man.
Fa+ Scrooge1s sisterH GredCs mother. 6n ScroogeCs vision of <hristmases past, he remembers
Gan pic'ing him up from school and ta'ing him home.
Mrs. Cra"*,i" $ 3ob1s wife, a 'ind and loving woman
M0&"i%&e C,oi*e 80es"io+s. Ea*, =0es"io+ *arries . #arks a+d ea*,
s0(%ar" is 2or - #ark.
Read ",e e!"ra*"s a+d a+swer ",e 2o&&owi+4 =0es"io+s (: *,oosi+4 ",e #os" a%%ro%ria"e
o%"io+.
&. For seve+ :ears I ,ave (ee+ dead9"rave&&i+4 ",e w,o&e "i#e. No res"@ +o %ea*e@ o+&:
re#orse.
a. The spea'er is
;i.3usiness partner$Gred
;ii.3usiness partner of Scrooge$Dacob 9arley
;iii..:mployee of Scrooge
;iv. <lassmatea of Scrooge
b. The purpose of the spea'er1s visit was
;i.to warn Scrooge to mend his ways and to avoid a similar fate as his own.
;ii..to advise him to go to Gred1s house to celebrate <hristmas.
;iii. to tell him to donate money to the poor people.
;iv. to advise him to decrease the surplus population
c. The spea'er died
;i. seven years ago
;ii. eight years ago
;iii. nine years ago
;iv. ten years ago.
2. 7e sa:s :o0 are +ever *o#i+4 (a*k ,ere a+d 2ro# +ow o+ we wi&& (e a&wa:s "o4e",er.
a. The spea'er of this line is
;i. Gan
;ii. 3elle
;iii. Gred
;v. Tiny Tim
b. The first visitor travels with Scrooge through
;i. his memories.
;ii. his dreams.
;iii. his hopes.
;iv.. his nightmares.
c. This line has been spo'en when
;i..Scrooge1s father came home from his office.
;ii..3ob <ratchit came home from his office.
;iii. Third ghost visited him.
;iv. Scrooge1s sister came to ta'e Scrooge home.
.. I wis, I ,ad ,i# ,ere. ICd 4ive ,i# a %ie*e o2 #: #i+d "o 2eas" 0%o+ a+d I ,o%e ,e
wo0&d ,ave a 4ood a%%e"i"e 2or i".
a. 9rs. <ratchit1s opinion about Scrooge is that he is
;i. Social
;ii. Stingy
;iii. Lind
;iv. 4enerous
b. 3ob <ratchit
;i. did allow his wife to spea' ill of his boss.
;ii. thought it improper to spea' li'e that on the pious day of <hristmas.
;iii. thought that his boss would sac' him
;iv. =oined his wife to spea' ill of his boss.

c. The <ratchit family embraces the message that even in ^^^^^^^ there is hope.
;i. sorrow.
;ii. death.
;iii. poverty.
;iv. sic'ness.
5. I a# +o" ",e #a+ I was. I wi&& +o" (e ",e #a+ I #0s" ,ave (ee+ (0" 2or ",is &esso+ I
wi&& ,o+o0r C,ris"#as i+ #: ,ear".
a. After the visit of the three ghosts Scrooge would
;i. be compassionate, considerate and concerned about other1s miseries
;ii. 3e cruel, stingy and miserly
;iii. be caring and loving but would remain to be miserly
;iv. Not at all be heeding the advice of the ghosts.
b. Scrooge $$$$$$$$$$$$$$$$when he utters these lines.
;i. >aughs and cries
;ii. cries and prays
;iii. cries and screams
;iv.. Screams and prays
c. The visit of the Third 4host was a
;i. horrifying experience and a realiEation from within
;ii. pleasant experience of his childhood memories
;iii. memorable but a sad experience
;iv. reminder of Scrooge1s sins of the past.

A+swer ",e 2o&&owi+4 =0es"io+s i+ a(o0" .09/0 words $ 2 #arks ea*,)
- ;Des%i"e ",e (i""er&: *o&d wea",er@ a&& o2 Lo+do+ is i+ a 2es"ive #ood. B0" ",ere is +o
,a%%: e!%ressio+ o+ E(e+eQer S*roo4e;s &i+ed 2a*e.;
a. ,hat does this show of ScroogeM
b. ,hat would the reaction of the society be to such a manM
Answer( a. Scrooge is not religious. 2e hates <hristmas as he is expected to give charity and
give 3ob <ratchit a holiday.
b. Society will not respect such a man.
2 ;3o0;re +o",i+4 (0" a+ 0+di4es"ed (i" o2 #ea" B&o" o2 #0s"ard@ a *r0#( o2 *,eese.;
a. ,ho is being described hereM
b. ,hy has the person made his appearanceM
Answer( a. Scrooge describes Dacob 9arleyCs 4host.
b. Dacob 9arley is in <hains. These chains represent the selfish acts done in his days on
earth. Dacob has come to scrooge as a 4host to try and correct him.
.. ;A+o",er ido& ,as dis%&a*ed #e@ a 4o&de+ o+e.;
a. ,ho is spea'ing to whomM
b. ,hy does the spea'er say thisM
c. Reflect on the character of the person spo'en about.
Answer( a. 3elle to Scrooge.
b. 2e does not marry 3elle as a family involves expenditure and he hates spending money.
c. Scrooge is mean and cruel right from his youth. 2e holds gold and money in greater
importance than necessary. :motions and feelings mean nothing to him.
/. Ge+"&e#e+: Mos" o2 ",e# wo0&d ra",er die ",a+ do ",a".
S*roo4e: T,e+ &e" ",e# do ",a" a+d ,e&% de*rease ",e s0r%&0s %o%0&a"io+
a. ,hy has the gentleman comeM
b. ,ho are they tal'ing aboutM
c. ,hat character of Scrooge is revealed hereM
Answer( a. to collect some money for charity.
b. They are tal'ing about the poor and needy, who do not have a great income, but will not
ta'e any assistance from warehouses.
c. Scrooge is very proud of the money he has. 2e is cruel, hard hearted and unsympathetic
towards the poor.
5. I was 4&ad "o (e a(&e "o 4o "o *,0r*,. T,a";s (e*a0se I wa+"ed ",e %eo%&e "o see ",a"
I;# a *ri%%&e.
a. ,hy does Tim want the people to see himM
b. ,hat future does the second ghost predict for himM
c. ,hat actually happenedM
Answer( a. Tim wants the people in the church to pray for him.
b. Tim would die as 3ob has no money for his treatment.
c. Scrooge is transformed. Grom a cruel and selfish man he becomes 'ind. 2e pays for TimCs
treatment and Tim recovers.
<. T,ird G,os": T,a" "o#(s"o+e... Read ",e +a#e o+ i".
S*roo4e: Be2ore I do@ a+swer #e o+e =0es"io+. Are ",ese the shadows of the things that
may be onlyM
a. ,hose tombstone was itM
b. ,hat does Scrooge mean by this questionM 0oes this reveal a changeM
Answer( a. :beneEer Scrooge
b. Scrooge does not wish to die as a miser and a cruel man. 2e would li'e to 'now if it was
possible to change the course of the future. Oes, we see a change in Scrooge. /erhaps he has
realised his mista'es and would li'e to be a good <hristian.
F. Fro# +ow o+ I a# 4oi+4 "o (e o+e o2 :o0r #os" %ersis"e+" 40es"s.
a. ,hat character of the spea'er is revealed hereM
b. 6s Gred worthy of this attentionM
Answer( a. Scrooge has realised that he has done Gan ;his sister. a great wrong. 2e would
li'e to correct his mista'e. 2e therefore begins to renew his relationship with Gred his
nephew.
b. Gred has been respectful and affectionate towards Scrooge. 2e had come to invite him for
<hristmas. So he is worthy of this attention from Scrooge.
A. S*roo4e: Le" #e see so#e "e+der+ess *o++e*"ed wi", dea",.
&. ,ho is Scrooge spea'ing toM ,hat has Scrooge =ust seenM
). ,hat is Scrooge shown laterM
Answer( &. Scrooge is spea'ing to the third ghost. ,ho is showing him his futureM Scrooge
had =ust seen a lonely unclaimed dead body which he recogniEes as his own. 2e has seen two
men refusing to go for his funeral unless they get a free meal.
). >ater the third ghost shows scrooge 3ob <ratchitCs house where all of them are mourning
tiny Tim1s death. Tiny Tim is missed by everybody after his death. Then the ghost shows
scrooge a tombstone with his name written on it.
F. ,hat is the theme of the play ?The <hristmas <arol?M
&#. ,hy do you thin' Scrooge was what he wasM
Ialue points(
2e had a very harsh childhood $had not en=oyed the love of his family$ was sent away
to a boarding school by a very strict father$ result he became a miser$a man of no
compassion$ did not celebrate <hristmas at home.
Lo+4 A+swer 80es"io+s i+ a(o0" -009-50 words $< Marks)
&. 6n the play PA <hristmas <arol1, Scrooge says that a poor man has no right to be merry
whereas Gred says that a rich man has no right to be dismal. ,ho do you thin' is right and
whyM
). The first ghost showed two women figures to Scrooge. ,ho were they and why did their
memories disturb himM
* 3ring out the supernatural element in the play, PA <hristmas <arol1.
+. ,ho was Dacob 9arleyM ,hat role did his ghost play in changing 9r. ScroogeM
". 2ow did Scrooge help to let tiny Tim1s dream come trueM
!. Throw light upon the character of Scrooge before and after his encounter of the three
ghosts.
@. :beneEer Scrooge said that he feared the Spirit of <hristmas Oet to <ome more than all the
othersM ,hy do you thin' that wasM 2ow would you react if you got the chance to see your
futureM
%. Oou are :beneEer Scrooge. Oou have never done one 'ind thing or said one 'ind word to
your only living relative Gred. Now you are ashamed of yourself. The next time you saw your
nephew, what would you say to himM 2ow would you explain your past behaviourM

60&i0s Caesar
B: Wi&&ia# S,akes%eare
>e: 2a*"s
Ge+re
Tragic historical drama
Se""i+4 $Ti#e)
++ 3.<.
Type of wor'
/lay
Se""i+4 $1&a*e)
Ancient Rome, towards the end of the Roman republic
To+e
Serious, proud, virtuous, enraged, vengeful, idealistic, anguished
Author
,illiam Sha'espeare
T,e 1&a: 60&i0s Caesar is i+ 2ive a*"s. Give+ (e&ow is a (rie2 s0##ar: o2 ",e eve+"s ",a"
,a%%e+ (e2ore A*" II S*e+e II:99
Two Roman tribunes, Glavius and 9urellus, see the common people parading in the
streets instead of wor'ing in their shops. They demand to 'now why the men are not
wor'ing. A cobbler informs them that the people are celebrating <aesarCs victory.
9urellus is infuriated and tells them that <aesar has not defeated an enemy, but rather has
'illed the sons of /ompey the 4reat. /ompey previously ruled Rome along with <aesar
until their alliance fell apart, at which point they went to battle over the right to rule.
Dulius <aesar triumphantly returns to Rome on the festival of >upercalia, celebrated on
Gebruary &". 2e is followed by Antony and 3rutus and many followers.
A soothsayer approaches <aesar and calls out for attention. <aesar allows him to spea',
and the man tells <aesar to, ?3eware the ides of 9arch?. <aesar ignores this warning and
calls the man a dreamer.
3rutus remar's to <assius that he is afraid the people will crown <aesar 'ing.<assius
then tells 3rutus that ?3rutus? is =ust as good a name as ?<aesar?, and that both names
could =ust as easily rule Rome.
3rutus, afraid that <aesar will become a 'ing, struggles to decide whether to ta'e action
with <assius.
<asca remains onstage with 3rutus and <assius and tells them that the three shouts they
heard were because <aesar turned down the crown three times. Apparently Antony
offered him the crown three times, and <aesar turned it down three times.
<asca then adds that the people forgave <aesar and worshipped him even more for
turning away the crown.
<assius informs the audience in a soliloquy that he will fa'e several handwritten notes
and throw them into 3rutusC room in an attempt to ma'e 3rutus thin' the common people
want him to ta'e action against <aesar.
<assius then arrives and <asca tells him that the senators are planning to ma'e <aesar a
'ing the next morning. <assius draws his dagger and threatens to die before ever
allowing <aesar to achieve so much power. <asca sha'es hands with <assius and they
agree to wor' together to prevent <aesar from seiEing power.
<inna, a co$conspirator, arrives and together they then leave to go throw <assiusC
handwritten notes through 3rutusC window. <assius indicates that he is quite sure 3rutus
will =oin them within the next day.
3rutus is in his garden and has made up his mind that <aesar must be 'illed. 2is reasons
are that <aesar is abusing his power and that he has ascended far too quic'ly.
>ucius, 3rutusC servant, brings him a letter he has found in 3rutusC private room. 3rutus
interprets the letter as if it were from all of Rome, telling him to slay <aesar and restore
the republic.
<assius is further of the opinion that 9ar' Antony should be 'illed along with <aesar,
but again 3rutus is against the plan, calling it too ?bloody.?
They plan to commit their murder of <aesar at the Senate at eight oCcloc' that morning ;it
is only three in the morning at this point.. 2owever, they are worried that <aesar will not
show up because he has become so superstitious over the past few months.
0ecius tells them that he 'nows how to flatter <aesar, and assures them that he will
convince <aesar to go to the Senate. <assius and his followers then depart, leaving
3rutus alone.
Gis" o2 ",e e!"ra*" 2ro# C&ass E (ook
<aesar, still in his nightgown, is terrified by a dream his wife <alpurnia has had in which
she cried out, ?2elp, ho8 They murder <aesar8? 2e orders a servant to go to the priests
and have them sacrifice an animal in order to read the entrails for predictions of the
future.
<alpurnia arrives and tells him that he dare not leave the house that day. <aesar acts
brave and tells her that he fears nothing, and that he will die when it is necessary for him
to die. The servant returns and tells him that the sacrificed animal showed a very bad
omen, namely the beast did not have a heart.
<aesar insists on misinterpreting the omens, but <alpurnia begs him to blame her for his
absence from the Senate, to which he finally agrees.
2owever, 0ecius arrives at that moment in order to fetch <aesar to the Senate 2ouse.
<aesar tells him to inform the Senate that he will not come this day. 0ecius claims that he
will be moc'ed if he cannot provide a better reason than that. <aesar then tells him about
<alpurniaCs dream, which 0ecius reinterprets in a positive light.
0ecius then overwhelms <aesarCs resistance by as'ing him if the Senate should dissolve
until a better time when <alpurnia has more favourable dreams.
0ecius also tempts <aesar by saying that the Senate plans to give the crown to him and
they may change their minds if he does not go.
<aesar tells <alpurnia that he was acting foolishly, and agrees to go to the Senate.
<assius and the other conspirators arrive at that moment to accompany him to the Senate.
Antony also appears and =oins the group of men who then escort <aesar out of his house.
<aesar ta'es his seat in the Senate and proceeds to allow 9etellus <imber to petition
him. The man throws himself down at <aesarCs feet in order to beg for his brotherCs
release from banishment, but is ordered to get up.
<aesar tells him that fawning will not win him any favours. At this 3rutus comes forward
and pleads for the manCs brother. <assius soon =oins him.
<aesar tells them his decision is, ?constant as the Northern Star? and that he will not
remove the banishment. <asca 'neels and says 5Spea' hands for me7 .<asca first, and
then the other conspirators and 3rutus all stab <aesar who falls saying, ?:t tu, 3ruteM $
Then falls <aesar.
Antony arrives and laments the death of <aesar. 2e begs the murderers, specifically
3rutus, to tell him why <aesar had to be 'illed. 3rutus tells him that <aesar was
destroying the republic and had to be removed from power.
Antony pretends to be convinced by this and as's the conspirators to, ?>et each man
render me his bloody hand? 2e then sha'es hands with each of them, naming them as he
sha'es the hand.
Antony quic'ly recants his agreement with the murderers, and tells <assius that he almost
=oined them after sha'ing their handsH 2e as's them if he may have permission to ta'e the
body to the mar'etplace and show it to the crowds. 3rutus gives him permission to do
this, but immediately <assius pulls 3rutus aside and says, ?Oou 'now not what you do7
3rutus decides to give his speech first, and to allow Antony to spea' afterwards,
provided that Antony only says positive things about the conspirators. Antony agrees to
this.
>eft alone with the body of <aesar, Antony says, ?- pardon me, thou bleeding piece of
earth That 6 am mee' and gentle with these butchers? 2e continues, with his speech
becoming ever more violent, ?0omestic fury and fierce civil strife Shall cumber all the
parts of 6taly?
3rutus tells the masses that he loved <aesar more than any of them, but that he 'illed
<aesar because he loved Rome more.
3rutus then as's them if they want him to 'ill himself for his actions, to which the crowd
replies, ?>ive, 3rutus, live, live8? ;*.).++..
2e lastly begs them listen to 9ar' Antony spea' and to let him depart alone. 2e leaves
9ar' Antony alone to give his oration.
AntonyCs speech begins with the famous lines, ?Griends, Romans, countrymen, lend me
your ears?. 2is speech continually praises 3rutus as ?an honourable man? who has 'illed
<aesar for being ambitious.
2e then presents all of the images of <aesar in which <aesar has not been ambitious,
such as when <aesar thrice refused the crown on the day of >upercal, or when <aesar
filled the Roman treasury with ransom money from victories in war.
The plebeians slowly become convinced that <aesar was not ambitious and that he was
wrongly murdered.
Antony then pulls out <aesarCs will and tells them he should not read it to them. They beg
him to read it, and he finally agrees, but puts if off by descending into the masses and
standing next to the body of <aesar.
2e shows them the stab wounds and names the conspirators who gave <aesar the
wounds. The crowd starts to surge away in anarchy, crying, ?Revenge8 About8 See'8
3urn8 Gire8 Lill8 Slay8? Antony stops them and continues spea'ing.
2e finally reads them the will, in which <aesar has given every Roman citiEen seventy$
five drachmas. The plebeians react in a frenEy of anger against the men who 'illed
<aesar, and carry away the body.
Antony says, ?Now let it wor'. 9ischief, thou art afoot. Ta'e thou what course thou
wilt7. 2e has successfully instigated the mob to mutiny.
A Few I+"eres"i+4 Fa*"s A(o0" T,e 1&a: 60&i0s Caesar
Dulius <aesar was first published in the Girst Golio in &!)*. The playCs source was Sir
Thomas NorthCs translation of /lutarchCs >ife of 3rutus and >ife of <aesar.
To celebrate his victory over /ompey, Dulius <aesar gave a banquet at which &"#,###
guests were seated at )),### tables. 6t lasted for ) days. 2e also proclaimed a rent$free
year for every poor family in the :mpire.
Dulius <aesar was assassinated on the 6des of 9arch $ 9arch &", ++ 3.<.
A soothsayer tells <aesar who is already on his way to the Senate ;and his death.,
?3eware the ides of 9arch.? <aesar replies, ?2e is a dreamerH let us leave him.
/ass.?;W,a" a+ iro+:R.
Artemidorus offered a letter to <aesar warning him about the conspiracy in the street but
<aesar refused to read it ;Iro+:.
<aesarCs enemies assassinated him at the foot of /ompeyCs statue, where the Roman
Senate was meeting that day in the temple of Ienus. ;Iro+:)
The sixty conspirators, led by 9arcus Dunius 3rutus and <aius <assius came to the
meeting with daggers concealed in their togas and struc' <aesar at least 2. "i#es as he
stood at the base of /ompeyCs statue.
After his death, all the senators fled, and three slaves carried his body home to <alpurnia
several hours later.
3rutus and <assius finally committed suicide.
3oth Dulius <aesar and 9ar' Antony had affairs with the :gyptian Queen <leopatra.
<assius had a personal animosity and a ?peculiar bitterness? against <aesar. 3rutus was
pestered, by anonymous appeals calling upon him to rid the state of the tyrant.
<assius gathered a conglomerate of senators willing to assassinate <aesar but all agreed
that the *o+s%ira*: *o0&d +o" s0**eed wi",o0" ",e idea&is"i* 4&a#o0r ",a" Br0"0s;
%ar"i*i%a"io+ wo0&d (ri+4 "o i"K ,e was ",e esse+"ia& #a+ "o 4ive ",e e+"er%rise
%o&i"i*a& &e4i"i#a*:.
,hen <aesar saw that he was beset on every side by drawn daggers, he muffled his head
in his robe, and at the same time drew down its lap to his feet with his left hand, in order
to fall more decently, with the lower part of his body also covered. 2e uttered not a word,
but merely a groan at the first stro'e, though some have written that when 9arcus 3rutus
rushed at him, he said in 4ree', Cyou too, my childMC ?
,hen <aesarCs will was opened it was discovered that he had left a considerable legacy to
each Roman citiEen, and when the people saw his body, all disfigured with its wounds,
being carried through the forum, they bro'e through all bounds of discipline and order.
Co+"e!"
Dulius <aesar ta'es place in ancient Rome in ++ 3.<., when Rome was the centre of an
empire stretching from 3ritain to North Africa and from /ersia to Spain.
Rome suffered from constant infighting between ambitious military leaders and the far
wea'er senators to whom they supposedly owed allegiance.
The empire also suffered from a sharp division between citiEens, who were represented
in the senate, and the increasingly underrepresented plebeian masses.
A succession of men aspired to become the absolute ruler of Rome, but only Dulius
<aesar seemed li'ely to achieve this status. Those citiEens who favoured more
democratic rule feared that <aesar1s power would lead to the enslavement of Roman
citiEens by one of their own.
Therefore, a group of conspirators came together and assassinated <aesar. The
assassination, however, failed to put an end to the power struggles dividing the empire,
and civil war erupted shortly thereafter.
The plot of Sha'espeare1s play includes the events leading up to the assassination of
<aesar as well as much of the subsequent war, in which the deaths of the leading
conspirators constituted a sort of revenge for the assassination.
6n &"FF, when the play was first performed, Queen :liEabeth 6 had sat on the throne for
nearly forty years, enlarging her power at the expense of the aristocracy and the 2ouse of
<ommons. As she was then sixty$six years old, her reign seemed li'ely to end soon, yet
she lac'ed any heirs ;as did Dulius <aesar.. 6n an age when censorship would have limited
direct commentary on these worries, Sha'espeare could nevertheless use the story of
<aesar to comment on the political situation of his day.
Girst performed in &"FF. 6t was probably the first play performed in the 4lobe Theatre,
the playhouse that was erected around that time in order to accommodate Sha'espeare1s
increasingly successful theatre company.
2owever, the first authoritative text of the play did not appear until the &!)* Girst Golio
edition.
1LOT ODERDIEW
<aesar has defeated the Roman general /ompey, his archrival, in battle.
<aesar enters with his entourage, including the military and political figures 3rutus,
<assius, and Antony.
<assius hatches a plot to draw 3rutus into a conspiracy against <aesar.
That night, Rome is plagued with violent weather and a variety of bad omens and
portents.
<aesar prepares to go to the Senate. 2is wife, <alpurnia, begs him not to go,
describing recent nightmares she has had in which a statue of <aesar streamed with
blood and smiling men bathed their hands in the blood.
<aesar refuses to yield to fear and insists on going about his daily business.
Ginally, <alpurnia convinces him to stay home`if not out of caution, then as a
favour to her.
3ut 0ecius, one of the conspirators, then arrives and convinces <aesar that <alpurnia
has misinterpreted her dreams and the recent omens. <aesar departs for the Senate in
the company of the conspirators.
At the Senate, the conspirators spea' to <aesar, bowing at his feet and encircling
him. -ne by one, they stab him to death.
,hen <aesar sees his dear friend 3rutus among his murderers, he gives up his
struggle and dies.
The murderers bathe their hands and swords in <aesar1s blood, thus bringing
<alpurnia1s premonition to fruition.
Antony, having been led away on a false pretext, returns and pledges allegiance to
3rutus but weeps over <aesar1s body.
2e sha'es hands with the conspirators, thus mar'ing them all as guilty while
appearing to ma'e a gesture of conciliation.
,hen Antony as's why they 'illed <aesar, 3rutus replies that he will explain their
purpose in a funeral oration.
Antony as's to be allowed to spea' over the body as wellH 3rutus grants his
permission, though <assius remains suspicious of Antony.
The conspirators depart, and Antony, alone now, swears that <aesar1s death shall be
avenged.
3rutus and <assius go to the Gorum to spea' to the public.
<assius exits to address another part of the crowd.
3rutus declares to the masses that though he loved <aesar, he loves Rome more, and
<aesar1s ambition posed a danger to Roman liberty. The speech placates the crowd.
Antony appears with <aesar1s body, and 3rutus departs after turning the pulpit over
to Antony.
Repeatedly referring to 3rutus as 5an honourable man,7
Antony1s speech becomes increasingly sarcasticH questioning the claims that 3rutus
made in his speech that <aesar acted only out of ambition, Antony points out that
<aesar brought much wealth and glory to Rome, and three times turned down offers
of the crown.
Antony then produces <aesar1s will but announces that he will not read it for it would
upset the people inordinately. The crowd nevertheless begs him to read the will, so he
descends from the pulpit to stand next to <aesar1s body. 2e describes <aesar1s
horrible death and shows <aesar1s wounded body to the crowd.
2e then reads <aesar1s will, which bequeaths a sum of money to every citiEen and
orders that his private gardens be made public.
The crowd becomes enraged that this generous man lies deadH calling 3rutus and
<assius traitors, the masses set off to drive them from the city.
C,ara*"er Lis"
&. <aesar( pompous, fatalistic, easily flattered, feared
). <assius( =ealous, manipulative, unscrupulous, perceptive, realistic
*. 3rutus( honourable, ambitious, idealistic
+. Antony( revengeful, manipulative
". <rowd9ob <haracteristics( fic'leness, loss of individuality, inability to deal with
intellect and reason, emotionally hyped, capability to perform cruel and inhuman acts
Mar*0s Br0"0s 9
A supporter of the Republic who believes strongly in a government guided by the
votes of senators.
,hile 3rutus loves <aesar as a friend, he opposes the ascension of any single man to
the position of dictator
2e fears that <aesar aspires to such power.
3rutus1s inflexible sense of honour ma'es it easy for <aesar1s enemies to manipulate
him into believing that <aesar must die in order to preserve the republic.
,hile the other conspirators act out of envy and rivalry, only 3rutus truly believes
that <aesar1s death will benefit Rome.
Torn between his loyalty to <aesar and his allegiance to the state, 3rutus becomes the
tragic hero of the play.
3rutus emerges as the most complex character in Dulius <aesar and is also the play1s
tragic hero.
2e is a powerful public figure, but he appears also as a dignified military leader, and
a loving friend.
6n a moment of naave idealism, he ignores <assius1s advice and allows Antony to
spea' a funeral oration over <aesar1s body.
60&i0s Caesar 9
A great Roman general and senator recently returned to Rome in triumph after a
successful military campaign.
,hile his good friend 3rutus worries that <aesar may aspire to dictatorship over the
Roman republic, <aesar seems to show no such inclination, declining the crown
several times.
Seduced by the populace1s increasing idealiEation and idoliEation of his image, he
ignores ill omens and threats against his life, believing himself as eternal as the North
Star.;over confident.
<aesar declares( 56t seems to me most strange that men should fear, Seeing that
death, a necessary end, ,ill come when it will come7 ;66.ii.*"K*@.. 6n other words,
<aesar recogniEes that certain events lie beyond human controlH to crouch in fear of
them is to enter a paralysis equal to, if not worse than, death
Addressees himself in third person. Sees no difference between his powerful public
image and vulnerable human body`thin's of himself as the great <aesar who 'nows
no fear.
<ompares himself to the North Star which is permanent and immovable`shows his
arrogance
The conspirators charge <aesar with ambition, and his behaviour substantiates this
=udgment( he does vie for absolute power over Rome
At first, he stubbornly refuses to heed the nightmares of his wife, <alpurnia, and the
supernatural omens pervading the atmosphere. Though he is eventually persuaded not
to go to the Senate, <aesar ultimately lets his ambition get the better of him, as the
prospect of being crowned 'ing proves too glorious to resist.
Mark A+"o+:
A friend of <aesar. Antony claims allegiance to 3rutus and the conspirators after
<aesar1s death in order to save his own life.
>ater, however, when spea'ing a funeral oration over <aesar1s body, he spectacularly
persuades the audience to withdraw its support of 3rutus and instead condemn him as
a traitor. ,ith tears on his chee's and <aesar1s will in his hand, Antony engages
masterful rhetoric to stir the crowd to revolt against the conspirators.
Antony proves strong in all of the ways that 3rutus proves wea'.
/ersuades the plebeians of the conspirators1 in=ustice, thus gaining the masses1
political support.
Not too scrupulous to stoop to deceit and duplicity
Antony proves himself a consummate politician, using gestures and s'illed rhetoric to
his advantage.
6n his eulogy for <aesar, Antony is adept at tailoring his words and actions to his
audiences1 desires.
Cassi0s 9
A talented general and long$time acquaintance of <aesar
0isli'es the fact that <aesar has become godli'e in the eyes of the Romans.
2e slyly leads 3rutus to believe that <aesar has become too powerful and must die,
finally converting 3rutus to his cause by sending him forged letters claiming that the
Roman people support the death of <aesar.
6mpulsive and unscrupulous$$ shrewd opportunist$$ proves successful but lac's
integrity.
Cas*a 9
A public figure opposed to <aesar1s rise to power.
Ca&%0r+ia 9
<aesar1s wife. <alpurnia invests great authority in omens and portents.
She warns <aesar against going to the Senate on the 6des of 9arch, since she has had
terrible nightmares and heard reports of many bad omens.
Nevertheless, <aesar1s ambition ultimately causes him to disregard her advice.
De*i0s Br0"0s 9
A member of the conspiracy.
0ecius convinces <aesar that <alpurnia misinterpreted her terrible nightmares and
that, no danger awaits him at the Senate.
0ecius leads <aesar right into the hands of the conspirators.
C&eari+4 Co+*e%"s
-. Co#%are a+d *o+"ras" ",e #o"ives ",a" Br0"0s a+d Cassi0s ,ave 2or ki&&i+4
Caesar.
3rutus and <assius, both conspirators against <aesar both have dramatically different
views on the reasons why <aesar should be assassinated.
3rutus is a man of morals, reason and honour while <assius is deceitful and
manipulative.
3rutus see's to glorify Rome and avoid tyranny by assassinating <aesar.
<assius wants to destroy his enemy and severely disli'es being under the control of a
tyrant. As leaders in the Senate, 3rutus and <assius confederate to assassinate Dulius
<aesar.
3rutus has high standing in the senate and is revered by other members as a great leader
with outstanding ethics. 2e is always willing to place Rome above himself and says he
would give his life to save Rome from tyranny.
<assius, on the other hand, is more concerned about himself being ruled by a tyrant than
Rome. 2e is not a very honourable man but is very intelligent and contriving.
3rutus and <assius are ali'e in the fact that($$
They are both in the Roman senate.
They both also detest tyrants and in some part wish the best for Rome.
2owever, 3rutus feels that his actions are necessary to sustain free rule in Rome while
<assius sees freedom for Rome as an extra in the assassination.
2. Co#%are a+d *o+"ras" ",e *,ara*"ers o2 Caesar a+d Cassi0s
6n Dulius <aesar, Sha'espeare carefully moulds the characters of <aesar and <assius to
create rivalry and =ealousy.
<aesar, the emperor of Rome is vain, easily flattered and modestly superstitious. Those
characteristics leave him susceptible to his inevitable assassination.
<assius has the unique ability to interpret human nature and thought. <assius also envies
<aesar even though he has a severe disli'e for him.
These two characters compete in a struggle for power in Rome and their faults eventually
lead to their downfall.
<aesar is not content being emperor and desires to be crowned 'ing. 2is main fault is that
he feels that he is superior to others and not mortal, but divine and indestructible.
<assius also feels that <aesar is extremely powerful and that nobles have the
responsibility to stop <aesar.
As emperor, Dulius <aesar has ruled as a mild tyrant with qualities that <assius despises,
even though <assius is himself corrupt and over taxing.
3oth <aesar and <assius both confide in and show great companionship and camaraderie
in 'ey leaders, 9arc Antony and 3rutus, respectively.
<assius has a distinguished ability to understand the true meaning behind the statements
that his colleagues deliver.
<aesar, on the other hand, is easily flattered and not very perceptive of the evil that is all
around him.
<aesar is by no means consistent in his superstitions. 9eanwhile, <assius remains
constant in his disli'e of <aesar and plot to assassinate him.
6t is intriguing that Sha'espeare creates <aesar and <assius so that their flaws bring about
their demise.
<aesarCs inability to recogniEe his mortality and understand the foreshadowing of his
death lead to his assassination.
<assius hates <aesar and cites <aesarCs evil ambitions and actions as the need for
assassination.
2owever, <assius has much the actions and ambitions as <aesar did. 3oth <aesar and
<assius affiliate themselves with people who are both strong in character, leadership and
noble ;Antony and 3rutus..
<aesar and <assius complement each othersC style and are renowned for their rivalry.
The two leaders both exhibit extraordinary leadership abilities, but their inabilities and
uncertainties cost them their lives.
.. No"e o+ VE" T0 Br0"eC
?:t tu 3ruteM? <aesarCs simple statement sums up 3rutusC round character in the
development of The Tragedy of Dulius <aesar.
3rutus was thought no threat and an ingenious right$hand man due to his nobility and
his loyaltyH however, these qualities are precisely why the story is such a catastrophe.
,hat stemmed from these traits is the last expected outcome.
/erhaps the most famous three words uttered in literature, ?:t tu, 3ruteM? ;:ven you,
3rutusM.. This expression has come down in history to mean the ultimate betrayal by
oneCs closest friend. <aesarCs surprise was so immenseH he could only mutter these last
few words.
This scene, in which the conspirators in the Senate assassinate <aesar, is one of the
most dramatic moments on the Sha'espearean stage. The audience has =ust witnessed
the arrogance and hubris of a ruler who has sought, within a republic, to become a
monarch, comparing himself to the gods
3rutus, a friend whom <aesar had long trusted and treated as a son. and yet a man
who loved Rome ;and freedom. more, =oined the conspirators in the assassination, a
betrayal which is captured by the three words above
6n the accounts of the assassination by Roman historians, <aesar is said to have
resisted the onslaught of the assassins $ until he saw 3rutus among them, his dagger
drawn. <aesar then ceased to struggle and, pulling the top of his robe over his face,
cried( ?Oou too, 3rutus8M?
The words reflect the pain and surprise of <aesar because he suffered the greatest
wound of all in being attac'ed by his friend.
cThese words would have been said in 4ree' $ ?Lai su, te'non? ;Oou too, my sonM?. $
but are best 'nown in their >atin form( ?:t tu, 3ruteM?d
/. Br0"0s a+d A+"o+: are "wo di22ere+" %eo%&e@ a+d (e*a0se o2 ",is ",e: ,ave "wo
di22ere+" (e&ie2s. O+ ",e #a""er o2 Caesar@ ",e: disa4ree a+d "0r+ "owards ",e
%eo%&e "o ,e&% ",e# o0". Br0"0s;s s%ee*, is 2irs" a+d ",e+ A+"o+:;s. Bo", are
*o+vi+*i+4 (0" o+e wi&& %revai& over ",e o",er.
a) Br0"0sCs F0+era& Ora"io+:
3rutus, in a somewhat arro4a+"@ "o ",e %oi+", funeral oration, attempts to sway the
people. 2e ?0s"i2ies *o+s%iri+4 against <aesar by stating that <aesarCs ambition would
have hurt Rome. 2e tries to persuade the crowd by trying to tal' to them about ,o+o0r@
%a"rio"is# a+d ,is"or:.
The Romans go bac' and forth between leaders, responding emotionally, rather than
intellectually, to the orators.
3rutus see's to explain why he conspired against <aesar. 2e begins his speech with
?Romans, countrymen ...7 a%%ea&i+4 "o ",eir *o+s*io0s+ess as *i"iQe+s o2 Ro#e@ who,
he later says, will benefit as freeman with <aesarCs death. This shows that 3rutus k+ows
,ow "o &0re ",e *rowd, appealing to their better =udgement as Romans.
2is 0se o2 di*"io+ a+d *0++i+4 words help him to better express his point. 2e declares
that he is an honorable man, and tells them that he will let them =udge the validity of his
claims. That is, he wi&& a&&ow ",e "r0", "o s%eak 2or i"se&2. This encourages the crowd to
believe him, as an honourable man.
2e says that he wa+"s ",e# "o k+ow ",e 2a*"s. 2e says that the reason for 'illing
<aesar was his great love for Rome. 2e =ustifies this by saying that he loved <aesar but,
?Not that 6 loved <aesar less, but that 6 loved Rome more.?
2e then as's rhetorically if the people would want to live their lives as slaves under
<aesarCs rule or would they prefer to live as freemen with <aesar dead.
To anyone insulted by his speech he wonders if, as Romans who love their freedom, they
could be offended or re=ect what he, 3rutus, says. 2e poses the question, ?,ho is here so
base that would be a bondmanM? 2e stresses the point, repeating the line, ?6f any, spea',
for him have 6 offended.? ?6 pause for a reply.7
2e a&&ows ",e# "o res%o+d "o ,is r,e"ori*a& =0es"io+s, giving them an even greater
sense that he cares about them and their opinions. They can only respond, ? None, 3rutus,
none.? That is, none are offendedH they do not disagree or argue with his words or his
actions.
0irectly after the speech the crowd hail him as a hero and the next <aesar, w,i*, was
+o" ",e res%o+se w,i*, Br0"0s was &ooki+4 2or@ +ever",e&ess i" was %osi"ive@ it does
not mean much however as they havenCt heard Antony1s speech.
() A+"o+:;s 20+era& ora"io+
9ar' AntonyCs funeral oration uses a different approach. 2e manages to turn the mob
against the conspirators. 7ow does ,e do ",isB 2e 0ses #a+: r,e"ori*a& "ri*ks "o
%ers0ade ",e %eo%&e to go against the conspirators and support him and <aesar1s goals.
2e dra#a"i*a&&: e+"ers wi", CaesarCs (od: to win the sympathies of the crowd. 2e
first gathers everyone around <aesar1s body and points out the wounds that were made by
each conspirator`the crowd is moved to tears.
2e starts out by addressing the crowd as ?Griends? because he wa+"s "o *o#e "o ",e# as
a 2rie+d rather than a ruler trying to gain power.
2e then says, ?6 come to bury <aesar, not to praise him.?, thus he *a+ ease i+ %raises o2
Caesar without the crowd stopping him.
2e sounds very sincere when he says, ?The noble 3rutus hath told you <aesar was
ambitious.... Gor 3rutus is an honorable man.? Throughout his speech Antony continues
with his pledge to the conspirators by calling them ?honourable men? repeatedly, but the
*rowd 2ee&s a se+se o2 sar*as# each time he calls them that.
9arc Antony is a respectable man and is himself honourable, but most importantly ,e
,as #as"ered ",e ar" o2 r,e"ori*. Antony states in his speech that ?c3rutusd hath told
you <aesar was ambitious?, and then Antony retorts with ?6 thrice presented him c<aesard
a 'ingly crown which he did thrice refuse.? 3y doing that, Antony carefully re(0"s
Br0"0s; s"a"e#e+" ",a" Caesar was a#(i"io0s and starts turning the crowd against the
conspirators. This has an effect of *a0si+4 ",e *rowd "o (e&ieve Br0"0s is &:i+4 and
maybe everything he has said may be questionable.
2e then says ?Oou cthe crowdd all did love him once, not without cause. ,hat cause
withholds you then to mourn for himM? This rhetorical question goes against 3rutus by
questioning his speech in which he so greatly demoniEed and demeaned <aesar. Now the
crowd is starting to turn against the conspirators and follow Antony.
Antony tal's about <aesar very passionately to give the impression that <aesar is a hero
and causes the crowd to thin' about how they lost such a great man. 2e sa:s ,e wo+;" do
#a+: ",i+4s w,i*, ,e does do, ?6 will not stir you up,? when he does. ?6f it were so, it
was a grievous fault,? this gives the impression that Antony doesnCt agree, or doubts
3rutus, he tries to ma'e the crowd thin' that too.
Antony then "eases ",e *rowd wi", Caesar;s wi&&, which they beg him to read, but he
refuses. Antony tells the crowd to ?have patience? and expresses his feeling that he will
?wrong the honourable men whose daggers have stabbed <aesar? if he is to read the will.
The crowd yells out ?they were traitors7 and have at this time completely turned against
the conspirators and are inflamed about <aesar1s death.
To re20"e Br0"0s; *&ai# that <aesar was a heartless tyrant Antony recounts ?how dearly
he c<aesard loved him c3rutusd.
Next, Antony ,0#(&es ,i#se&2 as W+o ora"or@ as Br0"0s isW ,i+"i+4 ",a" Br0"0s 0sed
"ri*ker: i+ ,is s%ee*, "o de*eive ",e *rowd.
After that Antony deals his final blow by revealing to the crowd <aesar1s will, in which
?To every Roman citiEen he gives, to every several man seventy$five drachmas? as well
as land. 2e thus a%%ea&s "o ",eir se+se o2 4reed. 2e then as's the crowd, ?2ere was a
<aesar, when comes such anotherM7 which =0es"io+s ",e *o+s%ira"orsC a(i&i": "o &ead.
Ginally, Antony releases the crowd and utters, ?Now let it wor'. 9ischief, thou art afoot.
Ta'e thou course thou wilt.? After this the crowd riots and searches out the traitors in an
attempt to 'ill them
2e a(ides (: ,is a4ree#e+" wi", Br0"0s +o" "o %&a*e (&a#e o+ ",e *o+s%ira"ors.
Since the people responded positively to 3rutusC speech, Antony could not insult 3rutusC
honesty in a direct manner.
Oet, Antony disproves <aesarCs ambition with three examples. The people of Rome are
swayed to him by($$
his dramaticshistrionics
his way of ma'ing a point, his repetition
And compelling proof of <aesarCs concern. All of these style elements help him to ta'e
the crowd on 5his side.7
Antony1s style of %ara&&e&is# a+d s:+"a! helped him to get his point across better
than 3rutus did. 2e is a(&e "o e&o=0e+"&: #a+i%0&a"e ",e *rowd ",ro04,
re#arka(&e r,e"ori* ski&&s a+d "0r+ ",e# a4ai+s" ",e ;,o+o0ra(&e #e+.; 7is
s%ee*, was "r0&: o+e o2 ",e #os" re+ow+ed a+d %re#i0# e!a#%&es o2 ar"i*0&a"io+
a+d de&iver:.
6n conclusion, AntonyCs has an advantage over 3rutus because he bac's up statements
with examples while 3rutus leaves his statements more open$ended.
The people seem to find it easier to accept A+"o+:@ a+ e#o"io+a& a+d si+*ere s%eaker@
than Br0"0s who appears arro4a+" a+d 2or*e20&.
S%ee*,es a" a G&a+*e
A+"o+:$o22e+sive) Br0"0s$de2e+sive)
rea&is"i*IMa+i%0&a"ive$
Reads the crowdCs hostility and plays
upon it`underhand way of ma'ing a
point
Starts mildly$$ begins implanting
anger and #0"i+o0s ideas when he
'nows the mob agrees with him
Ba&a+*ed %,rases$ As's =0es"io+s
Re%e"i"io+ of phrases which leads to
an effective use of irony ?3rutus is an
honourable man?
0ramatic pauses, awaits for reply
To+e( soft and humble, emotional
;weeping., anger, sarcasm, increased
volume
Bse of s0s%e+se ;will.
Ta+4i(&e vis0a& e22e*"s( sealed will,
<aesarCs mantle, <aesarCs body
Lies( 0id <aesar really weep for the
poorM 2ow does Antony 'now whose
stab wound is where in <aesarCs
mantle
For#$ /oetry $ a form suitable to
appeal to emotions
Sa&0"a"io+ ?Griends, Romans,
countrymen...?
Idea&is"i*II#%er*e%"ive
Co&d @arro4a+"@ e#o"io+&ess ora"io+
:xpects the crowd, which is
emotionally stirred, to pay close
attention to his words
Ba&a+*ed se+"e+*es
R,:",#
As's =0es"io+s@ a&&ows #o( "o
a+swer
Bses vis0a& %ro% $da44er)
7aQ: 4e+era&iQa"io+ that if <aesar
lived Romans would all be bondmen
3andwagon technique playing upon
the idea that we are all good Romans
2is speech fails when a citiEen says,
5let him be <aesar7
For#$$$ /rose $ a form more fitting
to an appeal to reason
Sa&0"a"io+ ?Romans, countrymen,
and lovers8?
III. M0&"i%&e C,oi*e 80es"io+s
Read ",e 2o&&owi+4 e!"ra*"s a+d a+swer ",e =0es"io+s (: *,oosi+4 ",e #os" a%%ro%ria"e
o%"io+. Ea*, =0es"io+ *arries ",ree #arks a+d ea*, s0(%ar" is 2or o+e #ark.
-. W,a" *a+ (e avoided
W,ose e+d is %0r%osed (: ",e #i4,": GodsB
3e" Caesar s,a&& 4o 2or",: 2or ",ese %redi*"io+s
Are "o ",e wor&d i+ 4e+era& as "o Caesar
a. The extract shows that the spea'er is
i. a believer in destiny
ii. arrogant and fearless
iii. confident
iv. obstinate
b. P<aesar shall go forth1 refers to <aesar1s plan(
i. to go to the battlefield
ii. to meet the senators
iii. to attend the meeting of the senate
iv. to go ahead
c. According to <aesar the predictions
i. are very general in nature
ii. made for the world
iii. bode ill for the whole world and not him alone
iv. are a reflection of 4od1s purpose
2. T,e drea# is a&& a#iss i+"er%re"ed
I" was a visio+ 2air a+d 2or"0+a"e
a. The spea'er of these lines is(
i. <assius
ii. 9arcus 3rutus
iii. 0ecius 3rutus
iv. 9ar' Antony
b. The spea'er aims to(
i. guide <aesar
ii. mislead <aesar
iii. give <aesar a false sense of security
iv. reveal the reality behind the dream
c. Pamiss interpreted1 means that the dream(
i. 6s inappropriate
ii. 6s incomplete
iii. 6s wrongly interpreted
iv. 6s differently construed
.. P<owards die many times before their death
The valiant never taste of death but once1
a) The quality of <aesar highlighted through these lines is
i. optimism
ii. bravery
iii. pride
iv. arrogance
() P<owards die many times before their deaths1 means
i. cowards die several times in their lifetime
ii. cowards are terrified and don1t live their life properly
iii. cowards lead a death li'e existence
iv. cowards waste their precious life being afraid of and anticipating death
*) <aesar is spea'ing to(
i. 9arcus 3rutus
ii. <alpurnia
iii. 0ecius 3rutus
iv. 9ar' Antony
d) PThe valiant never taste of death but once1 means
i. the brave are never faced with death
ii. the brave never die
iii. the brave accept death fearlessly
iv. the brave die only once in a lifetime
/. I do0(" +o" o2 :o0r wisdo#
Le" ea*, #a+ re+der #e ,is (&ood: ,a+d
Ge+"&e#a+ a&&@9 a&as w,a" s,a&& I sa:B
M: *redi" +ow s"a+ds o+ s0*, s&i%%er: 4ro0+d
a) T,e s%eaker o2 ",ese &i+es is
i. 9arcus 3rutus
ii. <aesar
iii. <assius
iv. 9ar' Antony
() The spea'er1s tone is
i. deceptive and guileful
ii. flattering and indecisive
iii. Sad and troubled
iv. Appreciative
*) The spea'er thin's he stands on slippery ground because(
i. he may be regarded as deceitful
ii. he may be considered a coward or flatterer
iii. he is standing on <aesar1s blood
iv. he may be thought of as a fic'le$minded person
5. Now &e" i" work@ #is*,ie2@ ",o0 ar" a2oo"@
Take ",o0 w,a" *o0rse ",o0 wi&"
a. The spea'er of these lines is
i. 9arcus 3rutus
ii. 9ar' Antony
iii. <aesar
iv. A citiEen of Rome
b. P9ischief is afoot1 because
i. <aesar has been murdered
ii. The citiEens have been instigated to rebel
iii. The conspirators are playing a pran'
iv. A plot is being hatched against Roman citiEens
c. The final course ta'en by mischief was
i. 3rutus became the 'ing
ii. 9utiny bro'e out in Rome
iii. Roman citiEens turned against Antony
iv. Antony too' revenge for <aesar1s death
<. No" ",a" I &oved Caesar &ess (0" ",a" I &oved Ro#e #ore.
7ad :o0 ra",er Caesar was &ivi+4 a+d die a&& s&aves@
",a+ ",a" Caesar were dead @"o &ive a&& 2ree #e+B
a. Through these words the spea'er is(
i. <ondemning <aesar1s murder before Roman mob
ii. Dustifying <aesar1s death to Roman masses
iii. 6nstigating Roman crowds
iv. Advising the masses
b. The main aim of the spea'er is to prove that
i. <aesar was his friend
ii. <aesar was a tyrant
iii. <aesar1s death has saved them from slavery
iv. 2is loyalties are towards Rome
c. These lines are addressed to(
i. 9ar' Antony
ii. <alpurnia
iii. Roman masses
iv. <assius
<. T,is was ",e 0+ki+des" *0" o2 a&&
For w,e+ ",e +o(&e Caesar saw ,i# s"a(
I+4ra"i"0de@ #ore s"ro+4 ",a+ "rai"orCs ar#s@
80i"e va+=0is,Cd ,i#:
a. The Pun'indest cut1 refers to(
i. The blow struc' by 3rutus1 ingratitude
ii. The attac' by 3rutus
iii. The cruelty of the conspirators
iv. The bloody attac' on <aesar
b. The spea'er of these lines wants to(
i. <ondemn the traitors
ii. <reate public opinion against 3rutus
iii. :xpress his anger
iv. 9ourn <aesar1s death
*) T,e s%eaker is
i. addressing the traitors
ii. tal'ing to himself
iii. addressing the Roman mob
iv. tal'ing to <aesar1s body
II. S,or" a+swer 80es"io+s i+ a(o0" .09/0 words $ 2 #arks ea*,)
&. ,hat commotion is noticed by <alpurniaM ,hat is it suggestive of according to herM
). 2ighlight two most important qualities of <aesar1s character and substantiate them
from the text.
*. P2ow foolish do your fears seem now, <alpurniaM1 3ring out the irony behind
<aesar1s statement.
+. ,hat does <aesar mean when he says P<owards die many times before their death,
the valiant never taste of death but onceM
". 2ow does 0ecius persuade <aesar to go to the senate in spite of <alpurnia1s best
efforts to dissuade himM
!. ,hy does <assius ob=ect to Antony1s spea'ing to the Roman 9obM 2ow do his fears
come trueM
@. :xplain why Antony calls <aesar a Pbleeding piece of earth1M
%. ,hat were the contents of <aesar1s willM ,hy did Antony elaborate upon themM
F. 2ow does 3rutus =ustify the assassination of <aesarM
&#. 2ow was Antony able to provo'e the Roman mob through his speechM
&&. Antony disproves the conspirators1 claim about <aesar1s ambition with three
examples. ,hat are theyM
Lo+4 A+swer 80es"io+s i+ a(o0" -009-50 words $ < #arks)
&. 3ring out the significance of the words$ P:t Tu 3rute1.
). <ompare and contrast the funeral orations of 9ar' Antony and 9arcus 3rutus.
*. ,rite the character s'etch of Dulius <aesar.
+. 0o you thin' 3rutus is close to being an honourable manM ,hyM ,hy notM
". 9ar' Antony emerges as the true friend to <aesar. <omment with reference to the
play
SMMATIDE ASSESSMENT I
SAM1LE 1A1ER I
<>AS Q
:N4>6S2 <-99BN6<AT6I:
MAEIMM MAR>S A0
The question paper is divided into four sections.
Section A Reading comprehension )#
Section 3 ,riting )#
Section < 4rammar )#
Section 0 >iterature )#
S:<T6-N A
;Reading $)# mar's.
Q & Read the following passage and answer the questions that follow choosing from the
options provided to you ;" 9ar's.
9edia experts and social scientists have been wrestling with this questions for decades and
thousands of studies have been have been done on it. 9ost of them reached the same
conclusion`media violence is responsible for aggressive behaviour in children.
Research has found that the more violence children watch on television the more li'ely they
may act in aggressive ways towards others. Also, they become less sensitive to others pain
and are less li'ely to help a victim of violence.
A study of violence on 6ndian television and its impact on children, commissioned by
BN:S<-, accuse the idiot box 5bombarding young minds with all 'inds of violent image,
cutting across channels, programmes and viewing times.7 Not only studies but also incidents
go to prove that children who watch violent episodes show increased li'elihood of behaving
aggressively.
There have been reports from all over the country of children hurting themselves while trying
to are the superhuman feats of Sha'timan, the superhero of 6ndian TI. Then there was the six
year old child of >uc'now who leapt off the balcony of his second floor flat trying to imitate
a bungee =umping drop shown in soft drin's commercial.
There1s no doubt that media is a powerful teacher and contributes greatly to the way we act
and behave. 6n some cases li'e these the effects are immediate and in others there is a
5sleeper effect7 where the results show up much later.
;&.Aggressive behaviour in children is because they are
;a. trained so by friends ;b.aggressive from birth
;c. influenced by media violence ;d. trained at Larate classes
;).The idiot box in the passage refers to
;a. radio ;b. stage shows
J street plays ;d. television
;*.Social scientists say that media is the ma=or source of inspiration for children in the field
of
;a.fashion ;b.education
;c. sports ;d. aggressive behaviour
;+.9edia is a powerful teacher because
;a. it guides us to act and behave appropriately ;b. it entertains us
J it trains us ;d. it ma'es us violent
;". The term sleeper effect means
;a. sleeping ;b. dreaming
J affecting the subconscious ;d. identifying
Q). Read the poem carefully and choose the correct option ;&Q"]".

<B>TBR:
<ulture, an ardently admired elegance
2as been aptly compared to the bee
,hich pic's honey from tender blooms
Sweet in taste and toothsome as manna
Soft wax obtained from the honeycomb
A source of light when dar'ness falls
The bee thus gives sweetness and light.
0ar'ness often illustrates ignorance
,hereas light is a symbol of 'nowledge
Sweet words or righteous deeds
Render a person lovable and refined
6gnorance, a grave sin as well vice
Spoils the grace, pollutes the mind
A man of polish 6 guest of 'nowledge
6s in right earnest to imbibe culture
/rof 2aEara Singh ;abridged.
&. <ulture has been compared to a bee because
;a. 3oth are beautiful ;b. both are sweet
J 3oth are li'e honey ;d. culture is pic'ed up from places
li'e bee pic'ing honey from different
flowers

). A bee is a source of sweetness and light as
;a. it is light in co lour ;b. it is slender
J it glows li'e a glowworm ;d. it gives honey and beehive wax is used
for ma'ing candles
*. A refined person is 'nown by his
;a. ,ealth ;b. social status
J /oliteness and manners ;d. friends
+. The most deserving person to imbibe culture is the one
;a. ,ho is hardwor'ing ;b. who is well connected
J ,ho wishes to 'eep adding to his 'nowledge ;d. none of the above

" Pmanna1 refers to
;a. Toffees ;b. desserts
J candy ;d. sweets
*. Read the following passage and answer the questions that follow, choosing from the
options given. ;&Q"]".
6n order that all parts of the body wor' properly without getting strained or tired, we must
be aware of our body posture while wor'ing. 3ad posture can result in early fatigue,
deformation of the body and defective vision. ,hile wor'ing our shoulders should be up and
spine erect. Stooping or bending over our wor' will cause cramping of the lungs,
deformation of the spine and defects of the vision.
,hile standing, wal'ing or running not only should the spine be erect but the body
weight should be resting on the balls of feet not on the heels. Sleeping in curled up posture
interferes with circulation, breathing and causes strain to the muscles. 2igh pillows are bad
for the nec', chin, spine and for circulation of blood to the head and face.
Regular exercise done preferably in the morning and before meals, 'eeps our body strong,
fit and in good wor'ing condition. 6t has a special effect on the respiratory, circulatory,
muscular and nervous systems. ,e remain active, alert and relaxed in mind.
,e do not tire easily while doing wor'. Ta'ing part in regular sports and outdoor games
helps us to relax, ma'e friends, cooperate and share things with others. :xercises which are
strenuous and done irregularly are harmful and should be avoided.
&. Gor effective functioning of the body we must be aware of our
;a. nature of wor' ;b. environment
J right posture ;d. body parts
). 3ad posture can result in
;a. fatigue ;b. defective vision
J deformation of the body ;d. all the above
*. ,or'ing with stooped shoulder can cause
;a. nausea ;b. giddiness
J headache ;d. deformation of spine
+. Sleeping in curled up position can result in
;a. insomnia ;b. 'nee pain
J poor blood circulation ;d. in=ury to spine
". The term Pstrenuous1 in the passage means
;a. effective ;b. irregular
J vigorous ;d. unstable
Q +. Read the given passage and answer the questions that follow by choosing the correct
option. ;&Q"]".
SAI6N4 <-R3:TT1S T64:RS
At the ripe age of !+ 3ri=indra Singh continues pursuing his passion K to preserve Dim
<orbett National /ar' and to protect its inhabitants. Gor him life without an association with
the Animals would be li'e being a fish without water. 2e is candid and blunt when he tal's of
the failures and remedies in the programs and policies related to the par'. -ne can expect this
from him, given the fact that he was appointed honorary
warden of Dim <orbett National /ar' in &F%), when, at gunpoint he had arrested the entire
top brass of the district administration while it was poaching right in the heart of the area.
The <ommissioner of the district, the 0istrict 9agistrate, the Superintendent of /olice, the
Sub divisional 9agistrate and the S2- were arrested single handedly by him.
This set an unforgettable example for all poachers K high and mighty$ as far as the par' was
concerned as an arrest of this magnitude made by a mere citiEen was unheard of.
Today, when the census of tigers is underway at the par' amidst frequent reports of tiger
deaths and various other controversies, he says 5 the first concern must be to protect the tiger.
-ther issues can be dealt with later.7 2e is particularly pained about the growing instance of
the man Kanimal conflict and says that a lot needs to be done at the political, social and
administrative levels if the tiger population is to be 'ept thriving.
&. The warden wanted to pursue his passion in order to
;a. preserve the National /ar' and protect the inhabitants ;b.preserve the National /ar'
J protect the inhabitants ;d. none of the above
). 2is attitude towards plans and their failures is
;a. pre=udiced ;b. biased
J dishonest ;d. fran'
*. 2e was appointed the honorary warden of Dim <orbett /ar' as
;a. he encouraged poaching ;b. he favoured the administration
;c. he arrested the top brass of the
0istrict administration ;d.he did everything at gunpoint
+. 2e thin's that to protect the tiger species a lot needs to be done at the
;a. political level ;b. social level
; c. administrative level ;d. all the above
". The phrase Pa fish out of water1 means being in$$
;a. favorable conditions ;b. suitable situations
; c. a situation unsuited to ;d. a deadly situation

S:<T6-N 3
;,R6T6N4`)# 9ARLS.
". -n the basis of the value points given below, write a biographical s'etch in about %#
words. ;+ mar's.
Na#e( R.L.Narayan
1ro2essio+( ,riterH Novelist
80a&i2i*a"io+s( 3achelor1s 0egree from 9ysore Bniversity
S%e*ia& areas o2 e!%er"ise: began his writing career with YSwami and 2is friendsY in &F*" at
the age of )F
O",ers( writing style was mar'ed by simplicity and subtle humour
Fa#o0s (ooks$ The 3achelor of Arts, The 0ar' Room, The Ginancial :xpert, The 4uide and
many more
Awards: Sahitya A'ademi Award, /adma 3hushan
A*,ieve#e+"s( nominated for the Ra=ya Sabha in &F%F
!. Oou are an avid viewer of a new progamme related to teenagers on TI. 2owever, you have
=ust discovered that the TI station has shifted its telecast to late night. 6t would not be
possible for you to watch it because of this change. ,rite a letter to the 0irector of the
<hannel explaining why you li'e the said programme and what you would li'e the 0irector
to do.
W 0escribe why you li'e the programme
W :xplain why you believe it should not be on at a later time
W Say what action you would li'e the TI station to ta'e ;% mar's.
Q@. Oou are 0eepa'0eepi'a of St. Doseph1s School 3ombay. ,rite an article for the school
magaEine on the importance of the education of the girl child and the steps ta'en by the
government to encourage her education. Oou may use the hints given. ;% mar's.
7INTS
&. 6mportance given to girl child education by the 4overnment
). /rovisions for compulsory and free education under the Right to :ducation
*. Suggestion on infrastructure and other facilities required to improve girl child
education
a. Gree text boo's
b. Separate toilets for girls
c. 3ac' to school camps for out$of$school girls
d. 3ridge courses for older girls
e. Recruitment of "#S women teachersH among others.
+. -ther initiatives for improvement $ the National /rogramme for :ducation of 4irls at
:lementary >evel ;N/:4:>. and the Lasturba 4andhi 3ali'a Iidyalaya ;L43I.
S:<T6-N K<
;4RA99AR$)# 9ARLS.
%. <omplete the passage given below by writing one suitable word, choosing out of the
choices given. ;+ 9ar's.
.;a. ^^^ far as the export of wildlife is concerned this is luc'ily a sub=ect on ;b. ^^^opinion
does not stop at political boundaries. ;c.^^^^^^ people of :urope are =ust as ;d. ^^^^^^by
the ;e. ^^^^^^^ of 6ndian wildlife, as we are. ,hile we have stopped the ;f. ^^^^$and export
of animals, they have helped us ;g. ^^^^banning the import of those animals or s'ins which
are ;h.^^^ our endangered list.
a. ;i. the ;ii. as ;iii. to ;iv. if
;b. ;i. which ;ii. who ;iii. when ;iv. where
;c. ;i. thoughtful ;ii. needy ;iii. thin'ing ;iv. need
;d. ;i. disturbing ;ii. trouble ;iii. troubling ;iv. disturbed
;e. ;i. depleting ;ii.depletion ;iii. reduce ;iv. reduction
;f. ;i. 'ills ;ii. 'ill ;iii. 'illing ;iv. 'illed
;g. ;i. in ;ii. one ;iii. by ;iv. for
;h. ;i. for ;ii. on ;iii. over ;iv. above
F. Read the following conversation. <omplete the conversation by choosing the correct
option from the ones given below( ;+ 9ar's.
Ramesh ( ,hat are you doing this wee'end ShyamM
Shyam ( 6 do not have any specific plan
Ramesh ( 2ow do you li'e the idea of ;a) OOOOOOOOOOOOOOOOOB
Shyam ( 4reat, but 6 $() OOOOOOOOOOOOOOOOOOOO of my parents
Ramesh ( 6 can also request them on your behalf.
Shyam ( That1s a good idea. 2ow about $*) OOOOOOOOOOOOOOOB
Ramesh ( So what time do 6 drop inM
Shyam ( 6 thin' $d) OOOOOOOOOOOOOOO as they are at home then.
Ramesh ( 4ood, then 61ll see you all in the evening.
;a. ;i. having to go for a trip ;ii. tre''ing on the hills
;iii. tre''ed expedition ;iv. tre''ing to the hills
;b. ;i. need to as' ;ii. needed the permission
;iii. need to ta'e permission ;iv. need the permission
;c. ;i. coming over to my house ;ii. come over to my house
;iii. reaching my house ;iv. reaching my parent
;d. ;i. you should come in the evening
;ii. you can come in the evening
;iii. you may come in the evening
;iv. you would come in the evening
&#. Read the following conversation and then write it in reported speech. <hoose the most
appropriate option from those given below ;+ 9ar's.
9ohit ( 2ello Letan, where have you been all these yearsM
Letan( 6 was posted in 3angalore.
9ohit( Are you still wor'ing with the Tata 4roupM
Letan( Oes, in fact, 6 have been promoted and transferred bac' to 0elhi.
9ohit( <ongratulations8 Oour promotion calls for a party.
9ohit, an old classmate of Letan, met him by chance one day and as'ed him where he
had been all these years. Letan replied $a)OOOOOOOOOOOOOOOOOO . 9ohit further as'ed
him $()OOOOOOOOOOOOOOOOO . Letan replied in affirmative and told him that in fact he
$*)OOOOOOOOOOOOOOOOO bac' to 0elhi. 9ohit congratulated him and added
$d)OOOOOOOOOOOOOOOOOOOOOOOOO for a party.
;a. ;6. that he was posted in 3angalore.
;66. that he had been posted in 3angalore.
;666. he was posted in 3angalore.
;6I. he had been posted in 3angalore.
;b. ;6. if he was still wor'ing with the Tata 4roup.
;66. whether he wor'ed with the Tata 4roup.
;666. if he wor'ed with the Tata 4roup.
;6I. whether he wor's with the Tata 4roup.
;c. ;6. has been promoted and transferred.
;66. had been promoted and transferred.
;666. have been promoted and transferred.
;6I. was promoted and transferred.
;d. ;6. that his promotion calls.
;66. that his promotion called.
;666. that his promotion call.
;6I. that his promotion will call.
&&. The following is the instruction for the process of sedimentation. <omplete the
paragraph that follows, by choosing the correct option from those given below( ;+
mar's.
Ta'e a bea'erH half fill it with water
9ix salt and sand
Allow it to stand for sometime till sand settles
/our the solution into another =ar
Bse a cloth filter to sieve the mixture
A bea'er is ta'en. 6t $a)OOOOOOOOOOOOOOOO with water sand and salt is mixed in it.
The mixture $()OOOOOOOOOOOOOOOOOO for some time. >et the sand settle down. The
solution $*) OOOOOOOOOOOOOOO into another =ar. A cloth filter
$d)OOOOOOOOOOOOOOOOOOO sieve the mixture.
;a. ;i. are half filled ;ii. is half fill
;iii. is half filled ;iv. are half fill
;b. ;i. are allowed to stand ;ii. is allowed to stand
;ii. is allow to stand ;iv. are allow to stand
;c. ;i. is poured ;ii. is pour
;iii. are poured ;iv. are pour
;d. ;i. are use ;ii. are used
;iii. is use ;iv. is used
&). <omplete the following news items choosing from the options provided to you.
;+ 9ar's.
A) 1over": de*&i+ed a2"er re2or#s i+ I+dia: 1M
The economic reforms^^^^^^^^^^^^^^^^^^ in reducing poverty in 6ndia, /.9. 9anmohan
Singh said today.
6. 2ave helped
66. 2as helped
666. ,as helped
6I. 6s helped
$B) Ido&s i##ersed i+ river
The ten$day 0usshera festival came to an end yesterday with ^^^^^^^^^^^^^^^ in the
4anga
6. The immersion of the idols
66. The immersed of the idols
666. The immerse of the idols
6I. The immersing of the idols
$C ) Lo""er: 2ever 4ri%s Ger#a+:
The 4erman News Agency K 0/A$ has reported that ^^^^^^^^^^^^^^^^^ the nation in
disproportionate numbers.
$i) >ottery fever has gripped
$i) >ottery fever had gripped
$ii) >ottery fever have gripped
$iii) >ottery fever will have gripped
$D) 9aid wins 9aruti car as priEe
The fortune of a house maid today changed when ^^^^^^^^^^^^^^^^^^^ a 9aruti car in the
local mela raffle
$a) she won
$() she win
$*) she wins
$d) she will win

S:<T6-N 0
;>6T:RATBR:$ )# 9ARLS.
Q &*. Read the extracts and answer the following questions by choosing the most appropriate options.
Attempt any two. ;! 9ar's.

&. M: #o",er o+&: said
T,a+k 4od ",e s*or%io+ %i*ked o+ #e
a+d s%ared #: *,i&dre+.
a. The scorpion had pic'ed on the mother as
i. She wanted to hurt it
ii. She was moving in the unlit room
iii. 6t wanted to hurt her
iv. She was the only one available
b. The above lines indicate
i. 9other was ungrateful to the villagers
ii. She was grateful to the scorpion
iii. 9other1s selfless concern for her children
iv. 9other disregarded what father had done for her
c. 9other1s reaction to the scorpion bite
i. is similar to that of the villagers
ii. Dustifies the father1s reaction
iii. is that of submitting to the will of 4od
iv. is of relief that her children were saved from the pain
). O,@ &i2" #e as a wave@ a &ea2@ a *&o0dR
I 2a&& 0%o+ ",e ",or+s o2 &i2eR I (&eedR
A ,eav: wei4," o2 ,o0rs ,as *,ai+ed a+d (owed
O+e "oo &ike ",ee9 "a#e&ess@ a+d swi2"@ a+d %ro0d

a. The mood of the poet is
;i..sad
;ii.happy
;iii..aggressive
;iv..frustrated
b. The spea'er appeals to the ,est ,ind
;i.to infuse power to him
;ii.. to chain him with irons
;iii..to bow to him
;iv.to ta'e out the strength from him.
c. Thorns of life means
;i. the happiness of human life
;ii. the misfortunes of human life.
;iii. the bad ways of a person in his life
;iv. power of the west wind.
.. FIRST MAN: W,e+ did ,e dieB
SECOND MAN: Las" +i4,"@ I (e&ieve
FIRST MAN: I ",o04," ,eCd +ever die.
a. The two men are discussing the death of ^^^^^^^^^^
;i. Tiny Tim
;ii.Dacob 9arley
;iii. Scrooge
;iv. Scrooge1s father.
b. They express feelings of
;i. regret
;ii. contempt
;iii. affection
;iv. blame
c. The above scene was shown by the
;i. Girst 4host
;ii. Second 4host
;iii. Dacob 9arley1s ghost
;iv. Third 4host
Q &+. Answer any four of the following questions in *# to +# words each
;% 9ar's.
;a. ,hy did the people on earth find <utie /ie so adorableM
;b. ,hy does the second ghost say, 56 see an empty seat in the chimney corner and a
crutch$$$ if these shadows remain unaltered by the future the child will die7M
J 0escribe the character of the peasants in the Night of the Scorpion.
;d. ,hy does the 9editerranean sea sleep and what does it dream ofM
;e. ,hat did 3abuli1s last letter reveal about himM
Q&". Answer any one of the following ;! 9ar's.
0escribe <h$tsal1s life in <. 2. ,inter1s house. ,hat were the things that made him stay 6n
the houseM
-R
6magine 3abuli writes a diary daily. ,hen he comes bac' from home he writes a diary page
recording his feelings about his elder brother and the reason why he gave his share of family
property. ,rite that diary page for 3abuli.


ANS,:R L:O
S:<T6-N A ;)# 9ar's.
&. & c )d *d +a "c
). &d )d *c +c "b
*. &c )d *d +c "c
+. &a )d *c +d "c
S:<T6-N 3 ;)# 9ar's.
Q" Bio Ske"*, o2 R. >. Nara:a+
R.>. Nara:a+ was a 2a#o0s I+dia+ Wri"er@ (e""er k+ow+ as a +ove&is". 7e "ook ,is
Ba*,e&orCs De4ree 2ro# M:sore +iversi":. 7e (e4a+ ,is wri"i+4 *areer a" ",e a4e o2 2H
:ears i+ -H.5. 7is +ove&s were &iked (: *o##o+ %eo%&e (e*a0se o2 ,is si#%&e &a+40a4e
a+d ,is s0("&e ,0#o0r ,e 0ses. So#e o2 ,is 2a#o0s (ooks are9 T,e Ba*,e&or o2 Ar"s@ ",e
Dark Roo#@ T,e Fi+a+*ia& E!%er"@ T,e G0ide. 7e was awarded wi", Sa,i":a A*ade#:
Award a+d 1ad#a B,0s,a+I+ -HAH@ ,e was a&so +o#i+a"ed 2or Ra?:a Sa(,a.
8 <
9anas 4upta
2ouse No. +*"&, Ar=un Iihar
New 0elhi
#"
th
August )#&#
The 0irector
National 4eographic <hannel
9andi 2ouse New 0elhi
0ear Sir,
S0(?e*": Broad*as" S*,ed0&e o2 TTee+a4ersC Ta&kU o+ Na"io+a& Geo4ra%,i* C,a++e&
1ro4ra##e
6 want to write to you about the recent change in the broadcast schedule of the programme $
?Teenagers1 Tal' on your channel from its earlier slot of F(## pm to the present slot of &&(##
pm.
>et me first compliment you for the excellent content and presentation of this bi$wee'ly
programme. 6t addresses all the teenage problems and pressures in a wonderful manner. 9y
friends and 6 relate quite well to it.
2owever, the shifting of the broadcast time to such a late hour is an inconvenience. 6 am sure
6 echo the sentiments not only of friends but also of the other teenager fans of the programme
who have to get up early next morning. 6 appreciate your rationale behind the decision to
shift the programme. 2owever, perhaps you have underestimated the fan$following that this
programme has acquired over such a short period of time.
4iven its popularity, 6 request you to revert to the original broadcast schedule.
Than' you for your attention regarding this matter.
Oours faithfully
9anas 4upta
8 F Wa:s "o I#%rove Gir& C,i&d Ed0*a"io+
:ducation of girls has been a high priority with the 4overnment of 6ndia. The
national commitment to provide free and compulsory education to all children in the !$&+
years age group is now a Gundamental Right of every child in 6ndia after the passing of the
<onstitution ;%!th Amendment. Act in 0ecember, )##).
Reaching out to the girl child is central to the efforts to universaliEe elementary
education. Sarva Shi'sha Abhiyan, or P:ducation for All1 programme recogniEes that
ensuring girl1s education requires changes not only in the education system but also in
societal norms and attitudes. A two$pronged gender strategy has therefore been adopted, to
ma'e the education system responsive to the needs of the girls through targeted interventions
which serve as a pull factor to enhance access and retention of girls in schools and on the
other hand, to generate a community demand for girls1 education through training and
mobilisation.
The targeted provision for girls under Sarva Shi'sha Abhiyan include(
e Gree textboo's to all girls upto class I666
e Separate toilets for girls
e 3ac' to school camps for out$of$school girls
e 3ridge courses for older girls
e Recruitment of "#S women teachers
e Teachers1 sensitiEation programmes to promote equitable learning opportunities
e 4ender$sensitive teaching$learning materials including textboo's
e 6ntensive community mobilisation efforts
6n addition, to target poc'ets where girls education is lagging behind, the
4overnment of 6ndia has launched two focussed interventions for girls K the National
/rogramme for :ducation of 4irls at :lementary >evel ;N/:4:>. and the Lasturba
4andhi 3ali'a Iidyalaya ;L43I. to reach out to girls from marginalised social groups
in over *,### educationally bac'ward bloc's in the country where the female rural
literacy is below the national average and the gender gap in literacy is above the national
average.
S:<T6-N <
;4RA99AR )# 9ar's.
Q% a ii b i c i d iv e ii f iii g iii h ii + 9
QF a iv b iii c i d i + 9
Q&# a ii b i c ii d ii + 9
Q&& a iii b ii c i d iv + 9
Q&) a i b i c i d i + 9
S:<T6-N 0
;>6T:RATBR: )# 9ar's.
Q &*. &. a ii b iii c iv * 9
). a i b i c ii * 9
*. a iii b ii c iv * 9
; Answer any two of the above .
Q &+. a. small, alien, different from the inhabitants of /lanet :arth, covered with iridescent
feathers, liquid eyes, bushy hands, lovely sil' whis'ers.
b. reference to Tiny Tim, 6f Scrooge doesn1t provide help Tim may not live
c. peasants Kignorant ,superstitious, belief in suffering due to sins of the previous
births.
d. lullaby li'e coils made by the clear water stream falling on its bay, dreaming of old
palaces and towers trembling because of the power of the west wind.
e. reciprocation of all the sacrifices his elder brother had made, ac'nowledgement of the role
played by the elder brother.
; any four of the above. %m
Q &". escapes from prison, lonely, befriends ! wee's and * days old ,inters,
same siEe , completely na'ed, not ashamed of each others company, thic' wave of loving
emotions between ,inters and his mother remembers his mother, his childhood,
communicates with ,inters, when ,inters + months old <h$tsal1s whis'ers grow and he is
able to communicates with his mother and he leaves
-R
Fpm
&+.#F.&#
<hildhood memories, dutiful brother$,role played, show of care and concern,
Tribute$share of land Kfrom whom he had harvested everything in life
SMMATIDE ASSESSMENT II
SAM1LE 1A1ER II
<>AS Q
:N4>6S2 <-99BN6<AT6I:
MAEIMM MAR>S A0
The question paper is divided into four sections.
Section A Reading comprehension )#
Section 3 ,riting )#
Section < 4rammar )#
Section 0 >iterature )#
S:<T6-N A
;Reading $)# mar's.
Q & Read the following passage and answer the questions that follow choosing from the
options provided to you ;" 9ar's.
NOW A CAR WIT7 IN9BILT ELECTRIC SCOOTER
Traffic snarls in cities need not cause much worry as a new car with an in$built electric
scooter that flips and folds into the boot will allow commuters to Eip through the congested
streets.
Iol'swagon is wor'ing on a bi'e, 3i'.e that compacts into the car boot and can be recharged
by plugging it into the car1s battery outlet or a conventional power point.
The 3i'.e wor's in tandem with the car. 6t loo's li'e a traditional push$bi'e, but there are no
pedals, and it1s more li'e a folding electric scooter.
2onda in Dapan has already sold a version of its <ity 2atch in the &F%#1s with a bi'e in the
boot. The concept will be a blessing for commuters who are increasingly frustrated with
thic' traffic and hefty par'ing charges.
The 3i'.e1s top speed is )# 'mph and it has a range of about )# 'm. ,ith such a slight range
it1s hardly going to replace your everyday drive, but it could well be handy if you par' your
car a long distance from your wor'place, or even for getting to and from the train station.
Iol'swagon head of research and development, Blrich 2ac'enberg, said, 5,e thin' far
beyond the car, as such, with a focus on mobility in general.7
A. Traffic snarls in the city do not need to cause much worry as
a. <ommuters will Eip through streets in cars
b. <ars will fold into the boot
c. <ars can be recharged on the move
d. <ommuters can Eip through congested streets on an electric scooter that flips and
folds into the boot of the car
3. 3i'.e which has a top speed of )# 'mph has
a. A spare wheel
b. No pedals
c. A range of +# 'm
d. 2efty par'ing charges
<. The electric 3i'.e be a blessing to commuters
a. ,ho li'e to travel long distances
b. ,ho li'e conventional forms of transportation
c. ,ho are frustrated with the thic' traffic
d. ,ho do not li'e to drive cars
0. Iol'swagen designed the 3i'e
a. As a conventional form of transportation
b. To wor' in tandem with the car
c. To replace cars
d. Gor going to and from the railway station
:. Blrich 2ac'enberg, Iol'swagen head of development said
a. Iol'swagen designed 3i'e to wor' in tandem with the car
b. ,e thin' far beyond the car, as such, with a focus on mobility in general
c. 2onda has already sold a bi'e in the boot in the &F%#1s
d. 3i'e can replace your everyday drive
Q ) Read the following passage and answer the questions that follow choosing from the
options provided to you ;" 9ar's.
H&6+-)ec+ De()&)r'
,elcome to the brave new world of high$tech dentistry. 2ere, teeth stained by coffee, tea and
tobacco are bleached considerably white in a single sitting, and crowned and laminated in a
single visit. 6nfected gum tissue can be vaporiEed bloodlessly in a few minutes with lasers.
:ven though restorative dentistry dates bac' to ancient civiliEations, it is, in a sense, a new
art and one that is increasingly in demand. 6ndeed, restorative dentistry has become a multi$
billion$dollar industry. 9ore and more people are clamouring for dentistry that not only
ensures good oral health, but also helps attract admiring glances and raises self$esteem.
6f you need a restoration $ either an inlay that replaces part of a tooth or a crown that covers it
entirely, some dentists offer a space$age tic'et called C<erecC. This laser scanner outlines and
preserves the genetic design of your tooth with precision. 6t calculates the toothCs exact siEe
and shape, creates a three$dimensional computer image and sends it to a milling unit where,
in minutes diamond burrs transform a tiny bloc' of ceramic that is coloured to match the
enamel of your other tooth into a natural loo'ing crown.
$$$$$ Readers 0igest
A. ,ith the high tech dentistry
a. Teeth can be polished in single sitting
b. They can not be crowned and laminated in single visit
c. 6nfected gum tissue can be vaporiEed with lot of blood loss
d. Single sitting is not enough for teeth polishing
3. Restorative dentistry is in demand because
a. 6t is a multi$billion dollar industry
b. 0ates bac' to ancient civiliEation
c. 6t1s a new art
d. 6t ensures good oral health and raises self$esteem
<. <erec is
a. An inlay that replaces part of a tooth
b. A gadget that scans the genetic design of the teeth
c. A milling unit that transforms a tiny bloc' of ceramic
d. An outdated gadget
0. Gind one word from the following that means a continuous demand
a. Transform
b. Restore
c. <lamour
d. :nsure
:. The area of high tech dentistry in which lasers are not used is
a. IapouriEation of infected gum bloodlessly
b. <reating *$dimensional computer images of tooth
c. Transforming of tiny bloc' of ceramic by diamond burrs
d. /olishing of teeth
Q * Read the following passage and answer the questions that follow choosing from the
options provided to you ;" 9ar's.
Da++:@ ",e C,a#%io+ o2 ",e Wor&d
6nside the caravan, 6 stood on a chair and lit the oil lamp in the ceiling. 6 had some wee'end
homewor' to do and this was as good a time as any to do it. 6 laid my boo's on the table and
sat down. 3ut 6 found it impossible to 'eep my mind on my wor'.
The cloc' showed half past seven. This was the twilight time. 2e would be there now. 6
pictured him in his old navy blue sweater and pea'ed cap, wal'ing soft$footed up the trac'
towards the wood. 2e told me he wore the sweater because navy$blue hardly showed up in
the dar', blac' was even better, he said. The pea'ed cap was important too, he explained,
because the pea' casts a shadow over oneCs face. Dust about now he would be wriggling
through the hedge and entering the wood. 6nside the wood, 6 could see him treading carefully
over the leafy ground, stopping, listening, going on again and again and all the time
searching and searching for the 'eeper who would be standing somewhere, as still as a post,
behind a big tree with a gun under his arm. Leepers hardly move at all when they are in a
wood watching for poachers, he had told me. They stand dead still right up against the trun'
of a tree and its not easy to spot a motionless man in that position at twilight.
6 closed my boo's. 6t was no good trying to wor'. 6 decided to go to bed instead. 6 left the
lamp burning. Soon 6 fell asleep.
,hen 6 opened my eyes again, the oil$lamp was still glowing and the cloc' on the wall
showed ten minutes past two.
6 was out of my bun' and loo'ed into the bun' above mine. 6t was empty. 2e promised he
would be home by ten thirty at the latest and he never bro'e promises.
At that moment, a frightful sense of doom came over me. Something really had happened to
him this time. 6 felt quite certain of it.
$ Roald 0ahl
A. The narrator is
a. A detective
b. A 'eeper
c. A young school$going boy
d. A poacher
3. The narrator could not concentrate on his homewor' because
a. 2e was not interested in doing it
b. 2e was thin'ing constantly of someone and waiting for him
c. 2e wanted to go to the wood
d. 2e was feeling sleepy
<. ,ho do you thin' Phe1 is
a. The narrator1s father
b. The narrator1s classmate
c. An adult male with whom he shared a close relationship
d. <ould be any one of them
0. ,hy was Phe1 wearing a navy blue sweater
a. 2e li'ed the navy blue colour
b. 2e wanted to camouflage himself and evade the 'eepers
c. 2e was feeling cold
d. None of the above
:. ,hen Phe1 did not come at the promised time, the narrator became
a. /ensive
b. -ffensive
c. Apprehensive
d. <omprehensive
Q + Read the following passage and answer the questions that follow choosing from the
options provided to you( ;" 9ar's.
T,e Bi4 R0+
9ary li'es to run. :ver since she was in third grade she has been training to run long
distances. Now that she was in eighth grade, her parents agreed to let her run in the 3anana
<lassic. The 3anana <lassic is a ten 'ilometer run. 6t ta'es place every year in 9attydale.
The winners get a lifetime supply of peanuts and fruit =uice from /aulCs /eanut Stand.
9ary practiced by running for ten 'ilometers twice a day. -n the average run, she would
finish in thirty minutes. >ast yearCs winner finished the race in twenty$six minutes and ten
seconds.
-n the day of the race, 9ary ran faster then she ever had in past. As she came across the last
'ilometer she was in fourth place. She pic'ed up the pace and quic'ly leaped into second
place, =ust as she could see the finish line. She ran fast as should in a mad dash to the finish
line, but she =ust wasnCt fast enough to get ahead of leader.
Answer the questions(
A.. 9ary too' training for long runs for
a. one year
b. three years
c. five years
d. eight years
3. 3anana <lassic is
a. one 'ilometer run
b. five 'ilometer run
c. seven 'ilometer run
d. ten 'ilometer run
< 3anana <lassic is held in
a. New Oor'
b. Rosendale
c. [illdiale
d. 9attydale
0. The average amount of time that too' 9ary to complete a run the same distance as the
3anana <lassic
a. fifteen minutes
b. thirty minutes
c. forty minutes
d. one hour
:. The place at which 9ary finished in the 3anana <lassic was
a. first
b. second
c. fourth
d. sixth
S:<T6-N 3
;R:A06N4 $)# 9ar's.
Q". -n the occasion of 2->6 <:>:3RAT6-NS at your <olony <lub, all colony
members had participated. A short cultural programme was organiEed, with
some dances, songs and a s'it. Then, as the Secretary of the club, you applied a
holi tila' on everyone1s forehead. This was then followed by a lunch. ,rite a
factual description of the event in about %# $ &## words to be published in your
colony1s newsletter ;+9ar's.
Q!. Oour friend met with an accident. Oou rushed himher to the nearby 4ovt.
2ospital. 3ut the 'ind of treatment and facilities available at the hospital left you
disillusioned about the health care available to the common man. Send an email to the <hief
9edical -fficer highlighting the condition of 4ovt. 2ospitals and the need to bring changes
into the system. ;% 9ar's.
2ints(
W 0escribe why heshe was in hospital
W <omplain about the poor service heshe received
W Say what action you would li'e the hospital staff to ta'e
Q@. Oou are the captain of your school football team. Oou have to lead your team in a
match against a stronger team. 0raft a speech to address your fellow players to encourage
them play hard and win. ;%9ar's.
Notes
&. Iictory is not always for the strong. 6t is for the well$prepared.
). Iictory is important, but an honest effort is more important.
*. 4reat teams give great performances, but ordinary teams that give extra$ordinary
performances.
+. Leep in mind all that is at sta'e $ our hard$wor', our sacrifice, and the expectation of
the whole school.
S:<T6-N <
;4RA99AR$)# 9ar's.
Q%. <omplete the passage given below by writing one suitable word, choosing out of the
choices given. ;+ 9ar's.
. Tourism has grown in the last fifty years in ;a. ^^^^^^^areas of the world as an unplanned
activity. The results ;b. ^^^not been fairly up to the mar'. They have been haphaEard. ;c.
^^^example, in Thailand, large hotels have been ;d. ^^^^^at a beach resort ;e.
^^^^^^/attuya Kabout &"# 'm from 3ang'o'. The occupancy of the hotels ;f. ^^^^^^yet is
not even +#S a year. ;g. ^^^^^^who has visited ;h.^^^^ few popular holiday resorts of the
world will be aware of this.
a. ;i. most ;ii. total ;iii. many ;iv. all
;b. ;i. have ;ii. had ;iii. do ;iv. does
;c. ;i. to ;ii. for ;iii. one ;iv. many
;d. ;i. built ;ii. building ;iii. made ;iv. ma'e
;e. ;i. calling ;ii.name ;iii. naming ;iv. called
;f. ;i. as ;ii. for ;iii. today ;iv. day
;g. ;i. who ;ii. anyone ;iii. whoever ;iv. whomsoever
;h. ;i. some ;ii.most ;iii. a ;iv. many
F. Read the following conversation. <omplete the conversation by choosing the correct
option from the ones given below( ;+ 9ar's.
Travel Agent ( 4ood morning, S'ylar' Travels. 2ow may 6 help youM
9r. 4arg. ( 4ood 9orning. <ould you please $a)OOOOOOOO to 9umbaiM
Travel Agent ( <ertainly Sir. ,hen $()OOOOOOOOOOOOOOOOOO M
9r. 4arg ( ,ell, Tuesday morning, in fact.
Travel Agent ( 6 see. ,hat time $*) OOOOOOOOOOOOOOOOO for you SirM
9r. 4arg ( 6 have a @ -1cloc' appointment in evening at my 2Qs in
9umbai.
Travel Agent ( ,ell then Sir, 61ll boo' you $d)OOOOOOOOOOOOO to 9umbai.
6 hope that suits you.
9r. 4arg ( 6t does, in fact. Than' you very much
;a. ;i. boo'ed a tic'et for me ;ii. boo' a tic'et for me
;iii. boo's a tic'et for me ;iv. boo'ing a tic'et for me
;b. ;i. did you plan to travel ;ii. have you plan to travel
;iii. do you plan to travel ;iv. had you planned to travel
;c. ;i. would be suitable ;ii. shall be suitable
;iii. can be suitable ;iv. may be suitable
;d. ;i. from an afternoon flight ;ii. on an afternoon flight
;iii. for an afternoon flight ;iv. in an afternoon flight
&&. Read the following conversation and then write it in reported speech. <hoose the most
apropriate option from those given below ;+ 9ar's.
Gather ( <hildren , did you help your mother todayM
Son ( Gather, 6 washed all the dirty dishes.
0aughter ( And 6 hung out all the clothes to dry.
Son ( 6 also dried the dishes and 'ept then on the shelves.
The father as'ed his children $A). 2is son told him that he $B).The daughter said that she
$C) .The son, then added, that $D) and 'ept them on the shelves.

;a. ;6. whether they helped their mother today.
;66. if they helped their mother today.
;666. whether they have helped their mother today that day.
;6I. if they had helped their mother that day.
;b. ;6. had washed all the dirty dishes.
;66. has washed all the dirty dishes.
;666. washed all the dirty dishes.
;6I. have wash all the dirty dishes.
;c. ;6. has out all the clothes to dry.
;66. have out all the clothes to dry.
;666. had out all the clothes to dry.
;6I. hung out all the clothes to dry.
;d. ;6. dried the dishes.
;66. had dried the dishes.
;666. have dried the dishes.
;6I. has dried the dishes
.
&&. The following is the instruction for the process of sedimentation. <omplete the
paragraph that follows, by choosing the correct option from those given below( ;+
mar's.
7ow "o %ai+" a wa&&
<lean the wall with a soft broom
/ull out the nails
Gill in holes using white cement
Sand paper the wall, then sub with soft cloth
9ix the paint wellH dip paint brush in paint and wipe off excess paint
/aint the walls with a wide brush using neat and even stro'es
The wall is cleaned with a soft broom. Then all the nails $a)OOOOOOOOOOOOOOOOOOO out.
The holes are filled in using white cement. The wall $()OOOOOOOOOOOOOOOOOOOOOOOOO
and then rubbed with a soft cloth. The paint is mixed well, the brush ;c.
^^^^^^^^^^^^^^^^^^^^ in paint and the excess paint is wiped off. The walls
;d.^^^^^^^^^^^^^^ using neat and even stro'es, with a wide brush.
;a. ;i. is pulled ;ii. is pull
;iii. are pulled ;iv. are pull
;b. ;i. is sand $ paper ;ii. is sand papered
;ii. are sand papered ;iv. are sand paper
;c. ;i. are dipped ;ii. is dip
;iii. is dipped ;iv. are dip
;d. ;i. is painted ;ii. are painted
;ii. was painted ;iv. were painted
&). <omplete the following news items choosing from the options provided to you.
;+ 9ar's.
$A) B0d4e" +eeds "o (e do0(&ed "o a*,ieve -00G ed0*a"io+
6ndia ^^^^^^^^^^^^^^^^^^ on elementary education by over &##S if it has to achieve free
and compulsory education for all.
$a) Needed to double its budget
$() Needs to double its budget
$*) Need to double its budget
$d) ,ell need to double its budget
$B) 7os%i"a& %a:s Rs. - Lak, "o 4ir& 2or +e4&i4e+*e
A &) year old girl, who got exposed to septicemia due to a technical mista'e of a
surgeon,^^^^^^^^^^^^^^^ as compensation by the hospital.
$a) ,ill been paid Rs & la'h
$() ,ill pay Rs & la'h
$*) ,ill be paying Rs. >a'h
$d) ,ill be paid Rs & la'h
$C) Ta? Ma,a& is o0rs@ sa:s WA>F Board
Oou1ve always thought that the exquisite white marble monument in Agra was a National
Treasure. 3ut, on Saturday, a trust claimed that ^^^^^^^^^^^^^^^^^^^^^ to the ,ALG
board
$a) The Ta= belong
$() The Ta= belonged
$*) The Ta= is belonging
$d) The Ta= belongs
$D) Two A0"o9&i2"ers +a((ed@ %o&i*e *ra*k .0 *ases
The South 0istrict /olice claims to have ^^^^^^^^^^^^^^^^^^^ the arrest of two
persons on
Sunday
$a) <rac' *# cases of auto$lifting with
$() <rac'ed *# cases of auto$lifting with
$*) <rac's *# cases of auto$lifting with
$d) <rac'ed *# cases of auto$lifting with
S:<T6-N 0
; >6T:RATBR:$ )# 9ARLS.
Q &*. Read the extracts and answer the following questions by choosing the most
appropriate options. Attempt any two. ;! 9ar's.
5. We&&@ %oor (ird9 s,e s,o0&d ,ave k+ow+
T,a" :o0r so+4 #0s" (e :o0r ow+
T,a"Cs w,: I si+4 wi", %a+a*,e
a. The nightingale is referred as Ppoor bird1 means
i. She was short of money
ii. She deserves our sympathy
iii. She was in debt
iv. She owed a lot of money to frog
b. 6t is evident from the lines that the spea'er was
i. 2aughty
ii. <ondescending
iii. /atroniEing
iv. Superior
c. POour song must be your own1 implies that
ii. Oou must write your own songs
iii. Oou must believe in yourself
iv. -ur self image should be guided by others
iv. Oou should not lead your life by other1s standards
:ach morning it is her face that replaces the dar'ness. 6n me she has drowned a young girl
and in me an old woman. Rising towards her day after day li'e a terrible fish
;i. The expression K P6n me she has drowned a young girl mean($
;a. A young girl drowned in la'e
;b. Since her childhood the woman has been coming to the la'e
;c. The la'e no longer shows her lovely face
;d. The woman has been loo'ing into the mirror ever since she was
young. She feels her youth and beauty is dissolved in the mirror.
;ii. 2er face that replaces the dar'ness means
;a. The first thing the woman does in the morning is to view herself in the
mirror
;b. >ady1s face removes the dar'ness of the mirror
;c. >ady loo's bright and fair
;d. The dar'ness of her face
;iii. 2ere P a terrible fish1 is ^^
;a. The thought of death
;b. The thought of youth
;c. The thought of becoming very ugly in due course of time
;d. The thought of childhood
P4od save thee, ancient 9ariner8
Grom the fiends, that plague thee thus8$
,hy loo'1st thou soM ,ith my cross bow
6 shot the A>3ATR-SS.
&. The spea'er of the first dialogue is (
a. Ancient 9ariner
b. The wedding guest
c. Albatross
d. 3oth b R c
). The mariner 'illed the albatross with his (
a. 4un
b. <ross$bow
c. Lnife
d. None of the above
*. The ancient 9ariner loo'ed (
b. horrified
c. li'e a ghost
c. tensed and had an anxiety$ ridden expression on his face
d. both a R c
Q &+. Answer any four of the following questions in *# to +# words each
;% 9ar's.
a. 4ive two examples of personification from the poem K P9irror
b. 2ow did the frog become the unrivalled 'ing of the 3ogM
c. 0escribe the role played by the letter in the lesson The >etter
by 0huma'etu
d. ,hat reasons did the conspirators give to =ustify the 'illing of <aesarM
e. -f the two grandparents, who was strongerM 4ive two instances in support of your
answer. ;The Bltimate Safari.
Q&". Answer any one of the following ;! 9ar's.
2ow does 9ar' Antony turn the mob in favour of dead <aesar without offending the
conspiratorsM
-R
After reaching he refugee camp, the guide writes a page of his diary describing hi
experiences of the =ourney through the Lruger /ar'. ,rite his diary entry.
RE!IE/ COMMITTEE MEM9ERS
A%:&-r 7 Dr; E; T Ara8
Dire&tor
KI*, Mysore
G8&%a(ce 7 Mr; R8.' H8r&a
Asst *du&ational "###i&er
KI*, Mysore
S8.Aec) C-()r&.8)-r7 -ita Sengu9ta ,., *nglish
B! MEGC Ce()re
K! Vasa/adatta ,., *nglish
B! NAL

Você também pode gostar